Renal Exam Master

अब Quizwiz के साथ अपने होमवर्क और परीक्षाओं को एस करें!

A 25-year-old man developed hematuria and hemoptysis. Immunofluorescent studies on a renal biopsy demonstrated linear immune globulin deposits along glomerular basement membranes. What is the most likely diagnosis?

: Anti-glomerular basement membrane disease

A 5-year-old boy presents with history of low-grade fever, headache, and intermittent colicky abdominal pain localized mainly around the umbilicus since yesterday. He has vomited once. His symptoms are also accompanied by a purplish-red maculopapular rash more confluent over the lower extremities and the buttocks. There is no itching. Both knees and ankles are swollen and tender, and there is edema of the hands and feet mainly in the dependent areas. Examination of cardiovascular, respiratory, and abdominal are essentially normal. Laboratory investigations: Hemoglobin 10 gm Platelet count 550,000/mm3 WBC 11,000/mm3 Serum IGA 500 mg /dL (normal 14-159 mg/dL) UrineProteinuria++, RBCs++ StoolRBC+ What is the most likely diagnosis?

: Henoch-Schönlein purpura

Previously healthy 15-year-old female patient has serum sodium 231 mmol/L and serum osmolality 280 mOsm/kg. She denies urinary or GI symptoms. Blood pressure is low. There is poor skin turgor. Her BMI is extremely low. Oropharyngeal exam does not reveal abnormalities. Bowel sounds are normal. Which of the following do you expect to find in the urinalysis? A)Elevated sodium levels B)Low sodium levels C)Red blood cells D)Leukocyturia E)Oval fat casts F)Albuminuria

? Hypovolemia and hyponatremia. This patient is young and has no previous history of disease. Her BMI is very low so we should suspect diuretic abuse. Why not option 2? Low sodium may appear if vomiting or diarrhea (bulimia, laxatives) but oropharyngeal exam and abdominal exam do not suggest this.?

A 38-year-old male presents to the emergency department (ED) with pulsatile right-sided back and side pain that has been radiating to his groin for the past three hours. He was sitting on his couch at home when the pain began, and it has gotten progressively worse. He denies any recent trauma. A review of systems is significant for constipation, feelings of depression, and pain in each of his limbs. His temperature is 98.6°F (37°C), pulse is 78 beats/min, respirations are 16 breaths/min, and blood pressure is 138/84 mm Hg. The patient rates the pain as a 9 on a 10- point scale. Complete metabolic panel shows a serum calcium of 11.2 mg/dL and phosphate of 12 U/L. Urine analysis shows microscopic hematuria. An abdominal CT scan shows calcification within the kidney parenchyma bilaterally in addition to nephrolithiasis. Which of the following is the most likely cause of this condition? A)Hyperthyroidism B)Hypoparathyroidism C)Hypothyroidism D)Primary hyperparathyroidism E)Secondary hyperparathyroidism

?Nephrocalcinosis is a condition characterized by the calcification of the renal tubules that leads to renal insufficiency. This occurs via metastatic calcification, which means that calcium is deposited in normal tissues, rather than those that have been injured and are degenerating (which is called dystrophic calcification). Nephrocalcinosis most commonly occurs due to primary hyperparathyroidism, in which the parathyroid glands secrete more parathyroid hormone (PTH) than what is needed to maintain a normal serum calcium level. The most common cause of primary hyperparathyroidism is a parathyroid adenoma, and other causes include parathyroid carcinoma and sporadic parathyroid hyperplasia. In addition to signs of renal dysfunction, patients may exhibit signs of hypercalcemia, such as constipation, bone pain (due to osteoclasts being indirectly stimulated by PTH to resorb calcium), kidney stones, depression, and seizures. Treatment is via surgically removing the adenoma or parathyroid glands that are responsible. ?

A 67-year-old male presents with a subacute onset of lower urinary tract symptoms. He cannot discuss his past medical history or current medications. An initial genitourinary workup is started, and microscopic urinalysis reveals granular and waxy casts. What condition are these results most closely associated with? A)Chronic renal disease B)Nephrotic syndrome C)Glomerulonephritis D)Pyelonephritis E)Diuretic use

A)Casts are cylindrical structures consisting of clumps or clusters of cells or material that can form in the renal distal and collecting tubules of the kidney. Casts form when the pH of the urine is acidic and when the urine is concentrated. Granular casts are the second most common type of cast, and they result from the breakdown of cellular material. They are most often indicative of chronic renal disease, but they can also be seen with vigorous exercise and acute tubular necrosis. Waxy casts are also indicative of chronic renal disease. Fatty casts are the result of the breakdown of lipid-rich epithelial cells. These casts with proteinuria are pathognomonic for nephrotic syndrome. Nephritic syndromes, urinary tract injury, glomerulonephritis, and vasculitis all result in red blood cell casts. Whenever there are red blood cells in a cast, there is a strong indication for glomerular damage from a number of different disease processes. White blood cells seen within a cast indicate inflammation (acute interstitial nephritis) or infection of the kidney (pyelonephritis).Diuretic use results in the presence of hyaline casts.

A 52-year-old male with a history of type I diabetes mellitus presents to the ER with increasing fatigue. Two days ago, he ran out of insulin. He denies fevers or chills. His temperature is 37.2 degrees Celsius, blood pressure 84/56 mmHg, heart rate 100/min, respiratory rate 20/min, and SpO2 97% on room air. His physical exam is otherwise within normal limits. An arterial blood gas analysis shows the following: pH 7.25, PCO2 29, PO2 95, HCO3- 15.Which of the following acidbase disorders is present? A)Metabolic acidosis with appropriate respiratory compensation B)Respiratory acidosis with appropriate metabolic compensation C)Mixed metabolic and respiratory acidosis D)Metabolic alkalosis with appropriate respiratory compensation E)Respiratory alkalosis with appropriate metabolic compensation

A)Incorrect Answers: Answer B: In primary respiratory acidosis, the PCO2 would be elevated, not decreased. Answer C: A mixed acidosis would be diagnosed if the PCO2 was higher than expected based on Winters formula. Answer D: Metabolic alkalosis is diagnosed when the pH is higher than 7.4 in the setting of a primary elevation of HCO3-. Answer E: Respiratory alkalosis is diagnosed when the pH is higher than 7.4 in the setting of a primary decrease of PCO2.

A 64-year-old man presents with progressive weakening of his urinary stream, nocturia, post void dribbling, and the sensation that he is not emptying his bladder. Annual screening blood work recently revealed a serum prostate-specific antigen of 4.3 ng/mL. What is the most important component to your physical examination of this patient?

Abdominal exam and DRE (BPH/Prostate cancer)

A 10-year-old boy developed pharyngitis due to β-hemolytic streptococci. It subsided after a few days of penicillin therapy. 2 weeks later, his eyelids became puffy, and he had a fever, elevated blood pressure, and gross hematuria. What is the most likely diagnosis?

Acute proliferative glomerulonephritis

28-year-old woman receives a lung transplant due to cystic fibrosis. 2 months after the transplant day, she is readmitted with paralysis of the left body. A basal ganglia stroke is diagnosed by computed tomography (CT). After 2 weeks of hospital stay, she shows signs of cardiac failure and is intubated. A posterior mitral valve leaflet vegetation with severe regurgitation is identified by transesophageal echocardiography (TEE). She is referred for mechanical mitral valve replacement. Colonies of Aspergillus fumigatus are detected in the excised mitral valve leaflets. Intravenous antifungal therapy with amphotericin B is started. A week later, she presents with fever, shortness of breath, and edema. A rapid increase in creatinine levels is observed. Fraction of excretion of sodium (FENa) is increased (>3%), and urine analysis reveals epithelial casts. What would most likely result in return of normal morphology and function of kidney tissue?

Acute tubular necrosis

A 75-year-old African American woman presents for follow-up of osteoarthritis, diet-controlled type 2 diabetes, and hypertension. She has taken ibuprofen PRN x 10 (osteoarthritis) and lisinopril (hypertension) for 15 years. Her last urine albumin to creatinine ratio was 180 mg/g. Her osteoarthritis is currently asymptomatic, and she has not taken ibuprofen for 2 months. In review of her laboratory results from earlier this week, you note that her most recent serum potassium was 5.8 mEqdL. Her blood pressure is 147/90. How should you treat her hyperkalemia?

Addition of diuretic to lisinopril and follow potassium

A 59-year-old man with a past medical history of hypertension, benign prostatic hypertrophy, type II diabetes mellitus for the past 15 years, and chronic back pain for the past 4 years presents to the hospital with gross hematuria. The patient states he noticed blood in his urine last night along with mild intermittent flank pain. He further states that his diabetes and blood pressure are well controlled with medications and he has managed his chronic back pain with 2 aspirin/day for the past 4 years. Vital signs are T 98.6°F, BP 124/82 mm Hg, pulse 88/min, and RR 14/min. Blood work is notable for HbA1c level of 5.6%. Fasting glucose is 140 mmol/L. His PSA value is 2.0 ng/mL. A renal CT demonstrates papillary calcifications and cortical atrophy. What is the most likely etiology of hematuria in this patient?

Analgesic nephropathy

A 41-year-old man presents to the clinic complaining of a chronic cough over the past 4 months which has now accompanied by hemoptysis. He denies smoking or any past medical history. On physical exam, his head and neck examination is normal. Laboratory finding reveal a sodium of 142 mEq/L, a potassium of 4.3 mEq/L, a chloride of 110 mEq/L, a bicarbonate of 24 mEq/L, a BUN of 39 mEq/L, and a creatinine of 2.9 mEq/L. Urinalysis reveals microscopic hematuria and 4+ proteinuria. Which of the following serologic tests would most help confirm the suspected diagnosis?

Anti-glomerular basement membrane antibodies

A 7-year-old boy is brought to the emergency department by his mother because of " tea colored urine" for the last several days. He ahs had some nausea and vomiting, and his eyes appear swollen from when he wakes up in the morning. The eye swelling tends to resolve over the course of the day. He is generally very healthy and there is no family history of any chronic disease. His temperature is 36.7 C ( 98.F), blood pressure is 130/90 mmHg, pulse is 96/min, and respiratory rate 16/min. Physical exam is unremarkable. A urinalysis shows red cell casts. At this time the most appropriate study to confirm the diagnosis is .

Antistreptolysin O Antibody

Patient with history of vitiligo and rheumatoid arthritis has serum sodium 231 mmol/L and serum osmolality 280 mOsm/kg. Blood pressure is normal and there are no remarkable findings on physical exam. Which of the following is the best next step? A)Urinary cortisol B)Thyroid function tests C)Kidney biopsy D)Urinary sodium E)IV insulin

B) Euvolemic patient and hyponatremia. The best next step in the workup is to assess thyroid or cortisol levels to rule out hypothyroidism or hypocortisolism. Given the history of autoimmune diseases, TFTs are the best option.

A 60-year-old male with a history of recurrent sinus infections presents with hemoptysis and hematuria. Physical examination shows a temperature of 101°F, a blood pressure of 145/85 mm Hg, decreased breath sounds on his right lower lobe, and palpable purpura on his bilateral lower legs. No warm or swollen joints were noted. What is the most likely diagnosis? A)Systemic lupus erythematosus (SLE) B)Granulomatosis with polyangiitis (GPA, Wegener's granulomatosis) C)Rheumatoid arthritis (RA) D)Anti-GBM antibody disease (Goodpasture's disease) E)Immunoglobulin A vasculitis (IgA vasculitis, Henoch-Schönlein Purpura, HSP)

B)This patient most likely has granulomatosis with polyangiitis, formerly known as Wegener's granulomatosis. While many of the answer choices can have palpable purpura, hemoptysis, hematuria, and signs of glomerular disease, granulomatosis with polyangiitis is most likely to have this combination of symptoms and lab results. Granulomatosis with polyangiitis is a small vessel vasculitis of unknown pathogenesis that causes inflammation and necrotizing granulomas in the upper and lower respiratory tract and kidneys along with rash; 15-50% of patients have cutaneous involvement. Palpable reddishpurple lesions on the lower extremities should raise suspicion of vasculitis; macules and nodules may also erupt. 50-95% of patients with this disease will eventually have renal involvement, which may range from mild findings to fulminant dialysis-requiring nephritis. On renal biopsy, focal segmental necrotizing glomerulonephritis is noted; crescents may be seen. Immune staining is generally negative. Treatment involves immunosuppressive therapy, as is the case with SLE, and dialysis as needed. In some resistant cases, plasmapheresis may be helpful. Patients with granulomatosis with polyangiitis/Wegener's are most commonly anti-neutrophil cytoplasmic antibody (ANCA)-positive (90% of patients).Lack of arthritis makes rheumatoid arthritis (RA) and systemic lupus erythematosus (SLE) much less likely. Lack of antidsDNA further reduces the likelihood of SLE. Patient's adult age makes IgA vasculitis/Henoch-Schönlein Purpura less likely. Lack of anti-GBM antibodies rules out anti-GBM antibody disease (Goodpasture's disease).

A 65-year-old man presents with gradual increase of urinary frequency over the past few months. Most recently, he has difficulties starting to urinate and the stream seems slow. Despite waking up several times a night to release his bladder, he continues to have a sensation of incomplete emptying. He has no history of sexually transmitted diseases, surgeries, or urinary tract infections. Upon examination, he is slightly overweight and his vitals are normal. His abdomen feels soft, the genitals are benign, and his prostate is non-tender and moderately enlarged (35-40 g). Lab tests are negative for blood presence in urine and indicate a protein-specific antigen (PSA) of 1.3 ng/mL. What is the most likely diagnosis?

BPH

A 71-year-old Caucasian man has been "peeing red" for 3 days and has come in to see you after constant urging of his wife. He is not having any pain with urination. Pertinent history includes a 100 pack-year smoking history, quitting 5 years ago after having a successful heart catheterization. He also has a history of benign prostatic hyperplasia, for which he is taking tamsulosin 0.4 mg once daily. The patient denies any acute trauma, injury, urinary frequency, or urgency. What is the most likely diagnosis?

Bladder cancer

A 65-year-old male presents with flank pain, blood in his urine, and an unexplained weight loss. His past medical history is significant for numerous infections, kidney stones, cigarette use, and alcohol use. On physical exam, there is a palpable abdominal mass, as well as a low-grade fever. What puts this patient at the greatest risk for the development of the most likely diagnosis? A)Escherichia coli infection B)Urolithiasis C)Smoking D)Interstitial nephritis E)Schistosoma haematobium infection

C)This patient has renal cell carcinoma. Smoking is a risk factor for many neoplasms, including renal cell carcinoma. There is a classic triad associated with renal cell carcinoma: hematuria, flank pain, and a palpable abdominal mass. The classic triad is not seen in most patients, however. Other presenting symptoms include a fever and weight loss. Cells from the proximal convoluted tubule are the most common cells from which renal carcinoma arises. There is an increased incidence of renal cell carcinoma with von Hippel-Lindau disease. Escherichia coli infection, urolithiasis, and interstitial nephritis are not known risk factors. Schistosoma haematobium infection is associated with bladder tumors, not renal cell carcinoma.

35-year-old man presents with right flank pain that has progressively worsened over the past 12 hours and is now radiating into his right groin and testicle. He currently rates the pain as an 8/10 and reports some nausea but no vomiting. He denies ever having this type of pain previously. He states that he thinks he had some blood in his urine at his last void, but he did not notice any before that. The patient is unable to sit still during the interview and refuses the portion of the physical exam where CVA tenderness is assessed; he also refuses any palpation of the abdomen or testicle on the right side. Vital signs include a temperature of 99.2°F, BP is 156/84 mm Hg, RR is 12, and oxygen sat 99% on room air. What is the best study to assess this patient's condition?

CT without contrast

A 45-year-old woman presents as a new patient. She was recently seen in the emergency department for right flank pain, and a CT scan revealed a right-sided ureteral stone. The stone was 4 mm, a passable size, and she was sent home with analgesics and advised to hydrate well and strain her urine. She was straining her urine and noticed a small dark fleck. She brought the sediment to the urologist's office to undergo a stone analysis, as this is her first episode of a renal or ureteral stone. What is the most likely composition of this stone

Calcium (m/c)

A 25-year-old sexually active man notices that he has burning and pain while urinating. He also notices some urethral discharge. He sees you in your office for a consultation, and you order several laboratory tests. One of the tests that you order is a Gram stain and culture on a sample of the discharge. The results are negative, and gonorrhea is ruled out. Based on the most likely diagnosis, what is the most common etiology of his symptoms?

Chlamydia trachomatis

A 27-year-old man presents with burning pain during urination and a 5-day history of urethral discharge. He sees his family physician because of his symptoms. A Gram stain and culture on a sample of the urethral discharge is performed; the results are negative. What is the most common cause of his condition?

Chlamydia trachomatis

A 67-year-old man presents with a subacute onset of lower urinary tract symptoms. He is unable to discuss his past medical history or current medications. An initial genitourinary workup is started, and a microscopic urinalysis reveals granular and waxy casts. What disease process are these results closely associated with?

Chronic renal disease

A 35-year-old woman presents with a 24-hour history of fever, right flank pain, vomiting, dysuria, and hematuria. A urinalysis reveals large numbers of red and white blood cells, as well as leukocyte esterase, and a subsequent urine culture was positive for >100,000 Escherichia coli. The patient denies having a history of renal calculi, and a bedside renal ultrasound does not show any stones or hydronephrosis bilaterally. She is diagnosed with a right-sided pyelonephritis and admitted for pain control, hydration, and IV antibiotic therapy. Her symptoms, including her fever, abate quickly, and she is discharged after 48 hours of being afebrile. Which prescription would she most likely receive upon discharge?

Cipro

A 33-year-old woman presents with fever, vomiting, severe irritative voiding symptoms, and pronounced costovertebral angle tenderness. Laboratory analysis reveals leukocytosis with a left shift; blood cultures indicate bacteremia. Urinalysis shows pyuria, mild hematuria, and gram-negative bacteria. What is the best treatment for this patient?

Ciprofloxacin (E coli)

A 2-year-old boy has a solid mass of tumor on the lower pole of his right kidney. Histopathological studies show whorls of mesenchymal cells and clusters of disorganized epithelial cells rather than renal corpuscles and renal tubules, which confirm your suspected diagnosis. What is true regarding treatment and/or prognosis of this diagnosis?

Combination chemotherapy is appropriate after surgery. (wilms tumor)

A 53-year-old woman presents for an annual examination. She has a history of asthma for which she takes inhaled steroids and ß-agonists. She has no history of bone fractures and no family history of osteoporosis. She exercises regularly. Her menses used to be regular, but have just started to show some irregularity. She believes she might be entering menopause and asks for advice to prevent osteoporosis. Question What treatment will be recommended?

Correct answer: 1200 mg of calcium and 800 IU vitamin D daily Explanation The correct response is the administration of 1200 mg of calcium and 800 IU vitamin D daily. The patient has no risk factors for osteoporosis. Even though she has noticed some irregularity in her menses, she does not have any other symptom characteristic to entering menopause. She takes inhaled steroids, which are not recognized as a cause of increased bone loss in adults (as is the case for oral steroids). These are the recommended daily calcium and vitamin D daily doses for adults, and regular exercise is recommended. A bone mineral density (BMD) screening test is incorrect because this screening test is recommended for women over 65 (or over 60 if they present other risk factors for osteoporosis). Administration of a vertebral imaging test is recommended for women 65 or older if the BMD screening test T-score is ≤−1.5 at the spine, total hip, or femoral neck, and for women older than 50 if other risk factors for osteoporosis are present. Administration of a 75 mg tablet of risedronate weekly is incorrect because risedronate is recommended for the treatment and prevention of osteoporosis in postmenopausal women. Administration of 200 units of nasal salmon calcitonin daily is incorrect because calcitonin is approved for treatment of osteoporosis only, not prevention.

At a yearly history and physical examination of a male patient, digital rectal exam reveals an enlarged prostate. You would more likely suspect benign prostatic hypertrophy, rather than prostate cancer, if the patient's history also included what presentation?

Correct answer: 72-year-old Caucasian who complains of a poor urinary stream Explanation The male patient most likely to have benign prostatic hypertrophy (BPH) is the 72-year-old Caucasian who complains of a poor urinary stream. BPH occurs in 90% of men >70 years old. It occurs in the periurethral zone of the prostate and usually presents with lower urinary symptoms (LUTS) that suggest obstruction (i.e. hesitancy, weak stream, intermittent stream, straining, incomplete emptying, post-void leaking) or irritation (i.e. nocturia, frequency, urgency). Risk factors for prostate cancer include men >50 years old or African American men >45 years old or a first-degree relative with prostate cancer. Prostate cancer most often develops in the peripheral zone of the prostate. Early prostate cancer is usually asymptomatic, but locally advanced prostate cancer may encroach on the central transition zone of the prostate and present with irritative urinary symptoms (e.g.,. nocturia, frequency, urgency). Laboratory findings suggestive of prostate cancer/BPH include: An elevated prostate specific antigen (PSA) >4 ng/mL: sensitivity of this value for prostate cancer is 57-79% In BPH, level of PSA is generally below 10 ng/dl, A rise in PSA >0.75 ng/mL per year would suggest prostate cancer

A 64-year-old man presents with progressive weakening of his urinary stream, nocturia, post void dribbling, and the sensation that he is not emptying his bladder. Annual screening blood work recently revealed a serum prostate-specific antigen of 4.3 ng/mL. Question What is the most important component to your physical examination of this patient?

Correct answer: Abdominal examination, including digital rectal exam Explanation The portion of your examination on this patient that would be the most crucial is the abdominal examination, including digital rectal exam (DRE), because a male patient in this age group is at risk for both benign prostatic hyperplasia (BPH) and prostate cancer. The lower urinary tract symptoms listed are indicative of BPH, as is the minimally elevated prostate-specific antigen. It would be important to rule out prostate cancer prior to treatment of the BPH. Prostate cancer screening includes a DRE, which is one reason that abdominal examination, including digital rectal exam (DRE), is the correct answer. A healthcare provider is also able to determine whether a patient's prostate is enlarged by doing a DRE, which is an extension of the abdominal examination. Suprapubic palpation of the bladder during the abdominal examination may indicate a large amount of retained urine secondary to BPH. Percussion of costovertebral angle tenderness (CVAT) is typically used to detect discomfort related to hydronephrosis. Hydronephrosis could be present secondary to obstruction of the urinary tract by various causes. It is true that extreme cases of BPH can cause hydronephrosis and therefore CVAT. This does not, however, make percussion for CVAT a more important component than abdominal examination and DRE. The latter is used for diagnosis, and the former can help diagnose a complication. Scrotal examination should be part of any genitourinary examination of a male patient, but it is not needed to make this patient's diagnosis. The lower urinary tract symptoms described are not symptoms of any scrotal abnormality. They are also not symptoms of any hepatic abnormality, so liver percussion would not be imperative in this case.

A 25-year-old woman is on a 2-week honeymoon trip with her new husband. At the end of the first week she experiences a burning sensation on micturition and finds herself going to the bathroom every 15-20 minutes. As the day progresses, she notices an ache in the lower abdominal area and seeks treatment at an urgent care center nearby. She has no fever, nausea, or diarrhea. On exam, her vitals are normal, and other than mild lower abdominal tenderness, there are no physical findings of significance. Baseline labs show serum: Hb 13g/dl, WBC 6500/uL, platelets 340000/ul; urine: WBC 12 and RBC 2. Question In all likelihood, this patient has which one of the following?

Correct answer: Acute cystitis Explanation The symptoms of high fever with chills, nausea, vomiting, and back pain with tenderness in the renal angle are classic for pyelonephritis. It is a common condition in young women. The common organisms are gram negative, for example E.coli, klebsiella, proteus, enterobacter, and pseudomonas. Gram positive bacteria, like staphylococcus aureus and enterococcus fecalis, may also be seen. The usual mode of infection is ascent from the lower urinary tract, except for staphylococcus aureus, which is hematogenously spread. Leukocytosis with a left shift and abnormal urine with pyuria and bacteriuria confirm the condition. Absence of pyuria should be an indication to look for an alternative diagnosis. Hematuria may also be present. Blood and urine cultures should be done. Imaging may be needed in complicated cases, in which scenario an ultrasound may reveal hydronephrosis due to obstruction from a calculus or other causes. It is generally recommended that all males with acute pyelonephritis undergo imaging with ultrasound or CT scan, since such an infection is usually associated with an anatomical abnormality like enlarged prostate, etc. A long urethra and absence of organisms residing in vagina makes it unusual for men to have a urinary infection with a normal anatomy. Treatment should be started empirically without waiting for culture results, since they are usually not available immediately, and as they become available, antibiotics may be changed accordingly. Urine gram stain, which is available right away, may be a useful tool to direct antibiotic treatment. Indications for hospitalization include vomiting, pregnancy, HIV disease, diabetes, impending septic shock with unstable vitals, and other comorbidities like renal failure, post transplant, etc. It should be treated with oral fluoroquinolone or trimethoprim-sulfamethoxazole for mild to moderate disease and IV ceftriaxone or a fluoroquinolone for hospitalized patients, to be substituted with oral antibiotics after improvement in symptoms. Total duration of antibiotics should be 10-14 days. Prognosis is usually good if diagnosis is prompt, treatment appropriate, and complications absent. Acute cystitis is a milder disease, which is more common in women than men due to a short urethra and proximity to vagina with its abundance of micro-organisms. About 50-60% adult women have had a urinary tract infection in their lives at some point. 10% postmenopausal women also have been found to get these infections. Coitus seems to be a predisposing factor, and symptoms quite often arise after sexual intercourse (honeymoon cystitis). The offending organisms include gram negative bacteria, such as E.coli, in 80-85% cases in women; it is also common in men. Most of the other cases in women are due to staphylococcus saprophyticus, though this is uncommon in case of males. This is a coagulase negative staphylococcus, which is normally considered benign but is actually a true urinary pathogen and should not be ignored. Rarely, klebsiella, proteus, enterococci, etc. may be isolated. Symptoms include low grade fever, dysuria, urgency, increased frequency of urination, and suprapubic abdominal pain. Occasionally women may have gross hematuria. There is suprapubic tenderness on examination without costovertebral angle tenderness. Urinalysis shows pyuria, bacteriuria, and hematuria. Hematuria is absent in female patients with urethritis and vaginitis, which can cause similar symptoms and can be used to differentiate the conditions. Urine culture is usually positive for the causative organism. Treatment is based on culture reports. Uncomplicated cystitis in women can be treated with a 3-day course of trimethoprim-sulfamethoxazole, trimethoprim alone, fluoroquinolone, or cephalexin. A 7-day course of nitrofurantoin is also adequate. Men should be evaluated for underlying conditions since uncomplicated cystitis is uncommon in males. A 7-day course is recommended even for uncomplicated cases in men. Acute gastroenteritis, or food poisoning, has a similar picture but without costovertebral angle tenderness. Diffuse abdominal pain and watery diarrhea are the predominant symptoms. Fever may be low grade or high grade. UA is usually normal. Acute salpingitis, or pelvic inflammatory disease, is characterized by lower abdominal pain and tenderness, abnormal vaginal discharge and/or bleeding, dyspareunia with adnexal tenderness, and cervical motion tenderness on a pelvic examination. An acute episode may present with high fever and chills, profuse vaginal discharge, and severe lower abdominal pain. Leukocytosis is found in less than 50% patients; UA is mostly normal, and culture of the vaginal fluid should be done. Treatment is with broad spectrum antibiotics. Acute diverticulitis is usually left sided and manifested by left lower quadrant abdominal pain and tenderness with diarrhea and occasionally low grade fever without chills. Leukocytosis may be present with sterile pyuria sometimes. The patient does not have back pain or costovertebral angle tenderness and seems well hydrated. Treatment is with ciprofloxacin and metronidazole for 7-10 days.

A 75-year-old African-American woman presents for follow up of osteoarthritis, diet-controlled type II diabetes, and hypertension. She has taken ibuprofen PRN x 10 (osteoarthritis) and lisinopril (hypertension) for 15 years. Her last urine albumin to creatinine ratio was 180 mg/g. Her osteoarthritis is currently asymptomatic, and she has not taken ibuprofen for 2 months. In review of her laboratory results from earlier this week, you note that her most recent serum potassium was 5.8 mEqdl. Her blood pressure is 140/88. How should you treat her hyperkalemia?

Correct answer: Addition of diuretic to lisinopril and follow potassium Explanation The correct response is addition of diuretic to lisinopril and follow potassium. This patient has not recently taken ibuprofen, making it an unlikely cause of her hyperkalemia. Ibuprofen and other non-steroidals can cause hyperkalemia by decreasing glomerular filtration rate, decreasing distal delivery of sodium (Na), thereby decreasing Na for potassium (K) exchange and flow for K excretion via flow-activated K channels. Sodium polystyrene sulfonate sorbitol is an exchange resin combined with a laxative. It exchanges potassium and sodium; it can be removed from the body in 4 - 24 hours if it produces a bowel movement. Many patients find it unpleasant tasting and do not like the laxative effect. It is possible that some patients would not want to take it daily. It is uncertain if bicarbonate administration would lower the potassium level. Lisinopril can elevate potassium by disrupting the renin-aldosterone-angiotensin system and inhibiting aldosterone-mediated potassium secretion. Presumably, this patient had a compelling indication (i.e., diabetes, albuminuria) to use lisinopril as a first-line blood pressure medication; African Americans without diabetes and or proteinuria might use diuretics or calcium channel blockers as first line antihypertensives. Goal blood pressures for patients >60 are generally less than 150/90 mm hg; the presence of proteinuria tightens this goal to less than 140/90 mm hg. The addition of a diuretic should increase distal delivery of sodium and assist with potassium excretion.

Question Glomerular damage inflicted in a patient with Goodpasture's disease is best described by which of the following?

Correct answer: Auto-antibodies are produced against glomerular basement membrane Explanation In Goodpasture's disease, autoantibodies (IgG) are produced against basement membrane antigen (type IV collagen). Simultaneous pulmonary hemorrhage and glomerulonephritis due to autoantibody deposition in pulmonary and glomerular basement membrane is known as Goodpasture's syndrome.

An 8-year-old boy presents to your office with his parents for an evaluation of his bedwetting. He is being teased about his bedwetting on overnight outings and his parents are concerned about his self-esteem. He has never had a prolonged period of dryness at night. He has no wet accidents during the day and has not had difficulties with stool incontinence. His parents report that his mother wet the bed until age twelve. They state that he appears to be a very sound sleeper. He is on no medications and his past medical history is unremarkable. The physical examination is within normal limits. Question What is considered first line intervention due to proven evidenced success rates?

Correct answer: Bed-wetting alarm system Explanation Bedwetting, or enuresis, is a common problem in the pediatric age group. Prevalence at the age of 5 years is 7% in boys and 3% in girls. At the age of 10 years the prevalence is 3% for males and 2% in females. Enuresis is divided into primary enuresis, where the child has never been dry at night, and secondary enuresis, where the child who has been continent for at least 6 months starts to wet the bed again. Primary nocturnal enuresis is associated with a smaller bladder capacity, abnormal arousal patterns during sleep, and inappropriate or inadequate toilet training. Secondary enuresis is generally precipitated by situations of psychological stress, such as a recent move, marital conflict, or a new sibling in the household. Only rarely does enuresis have an organic component; usually urological procedures are not warranted. General treatment guidelines that patients and especially the parents should be educated on include voiding before retiring, limiting fluids prior to bedtime, and expecting older children to clean their own bedding. More often than not, the first line intervention is considered bed-wetting alarms. These have been found in multiple studies to be extremely effective in producing a long-term cure. The bed-wetting alarms are moisture-sensing devices that are placed near the child's genitals, and are activated to trigger an alarm when the child voids in bed. These alarms go off at the initial first few drops of voiding, thereby awakening the child to get out of bed and finish voiding in the toilet or hold urine until later. This evokes a conditioned response of waking and inhibiting urination. They are generally recommended in children older than seven years. Therapy is recommend for at least three months and used every night. Parents must get up with the child to ensure the child does not just turn off the alarm and go back to sleep. If used appropriately, bed-wetting alarms are successful in 2/3 of the patients that utilize them. Alarm therapy requires a cooperative, motivated child and family. Parental involvement plays an essential role when using alarm devices due to the consistency that is necessary. Treatment must involve education and avoidance of being judgmental and even shaming the child due to most children feeling ashamed; the goal of treatment is to help the child establish their continence and at the same time maintain or gain self-esteem. It is recommended that children use these alarm devices until they experience three weeks of complete dryness. For complete resolution of nocturnal enuresis, the bed-wetting alarm may be needed to be used for up to 15 weeks. Relapse rates are higher when the alarm system is discontinued after shorter dry periods. Studies have shown that compared with other skill-based or pharmacologic treatments, the bed-wetting alarm has a higher success rate (75%) and a lower relapse rate. Desmopressin and imipramine are the primary drugs used in the treatment of nocturnal enuresis but should not be considered first line interventions. Imipramine administration is only slightly less effective than the alarm systems in bringing about dryness, usually within 2 weeks. Long-term results are less promising, as the enuresis tends to return while off the medication. Desmopressin acetate typically has an excellent response over the short-term, but again, the enuresis tends to recur when the child is taken off the medication. Fluid restriction and wearing diapers at night may resolve the consequences of the issue but will not create the behavior which will lead to the actual bedwetting from stopping; behavioral modification/training is a main key to treatment.

A 71-year-old Caucasian man presents because he is "peeing red". He states this has been occurring for 3 days and has come in to see you after constant urging of his wife. Pertinent past history includes being a former smoker, with 100 pack years and quitting 5 years ago after having a successful heart catheterization. He also has a history of benign prostatic hyperplasia, for which he is taking tamsulosin 0.4mg once daily. The patient denies any acute trauma, injury, urinary frequency, or urgency; there is no history of a urinary tract infection or kidney stones. Question What is the most likely diagnosis?

Correct answer: Bladder cancer Explanation Any time a patient's chief complaint resembles frank hematuria, bladder cancer must be assumed until proven otherwise. In fact hematuria in general, whether it is gross or microscopic, chronic or intermittent, is the presenting symptom in up to 85-90% of patients diagnosed with bladder cancer. It occurs more commonly in men than women and the mean age at diagnosis is around 73 years of age. Cigarette smoking and exposure to industrial dyes or solvents are known risk factors for this type of carcinoma. The patient above fits many of these components. Patients may also potentially have irritated voiding symptoms, although not always. A patient with a urinary tract infection (UTI) is extremely rare in men. Because this patient is not having any other symptoms other than the painless hematuria, one may still keep the diagnosis of a UTI on the differential diagnosis list. Symptomatic nephrolithiasis typically will cause the patient excruciating pain, usually in the flank areas. Nausea and vomiting may also be present. The location of this pain may correspond with the movement of the stone through the urinary tract. This is also inconsistent with the presenting patient in the above scenario. The pathology of diabetes mellitus itself does not cause hematuria, but rather the increase incidence of urinary tract infections in diabetics is what may increase this potential. Either acute or chronic prostatitis is not as likely due the fact that this gentlemen is not experiencing any irritating voiding symptoms fever, or even perineal or suprapubic pain.

A 62-year-old woman presents with bright red gross hematuria for the past 2 months. She states it is painless but persistent. She denies other symptoms. She has no chronic medical problems. Upon further questioning, she admits to a 50 pack-year smoking history, and she states she is currently retired from her job in a rubber factory. Vital signs are within normal limits, and physical exam is normal. Urine dipstick only shows red blood cells that are too numerous to count and urine cultures are negative. Question What is the most likely diagnosis?

Correct answer: Bladder cancer Explanation Hematuria is the most common presenting sign of urinary tract cancer, and bright red gross hematuria is usually of lower urinary tract origin. Silent or painless hematuria suggests tumor or renal parenchymal disease, so the clinical picture points to cancer, with bladder cancer as the most likely diagnosis. Smoking and exposure to industrial dyes or solvents (like in a rubber plant) are risk factors for bladder cancer. Bladder cancer is the second most common urologic cancer, and the mean age at diagnosis is 65. It is more common in men than women (2.7:1), and 98% of primary bladder cancers are epithelial malignancies (majority urothelial cell carcinomas). Ordering cytology of the urine sample is often helpful with higher grade and stage bladder cancers. Patients can become anemic with chronic blood loss, so a CBC is justified. Diagnosis is made by cystoscopy with biopsy. Acute cystitis typically presents with irritating voiding symptoms (frequency, urgency, dysuria), suprapubic discomfort, and possible hematuria. Urinalysis will show pyuria, bacteriuria, and varying degrees of hematuria. Urine cultures will show specific organisms. Renal cell carcinoma can present with flank pain, hematuria, persistent back pain, and an abdominal mass; also, it can be found incidentally on CT scan. RCC is more common in men than women (2:1), and it has a peak incidence in ages 50-60. This could be a possible option for diagnosis, but the significant history of smoking and previous work history points more toward bladder cancer. Urethritis is inflammation of the urethra that presents with urethral discharge, dysuria, and itching. Urethritis is most often caused by an STD. This patient does not have a history of unprotected sexual intercourse or any other symptoms that would indicate this diagnosis. Ureteral calculi can present with hematuria, but it also typically presents with flank or abdominal pain. If the stone is in the ureter, it often causes some hydroureter with or without hydronephrosis, both of which cause some pain or discomfort. The patient has no past history of forming stones.

A 45-year-old woman presents as a new patient. She was recently seen in the emergency department for right flank pain, and a CT scan revealed a right-sided ureteral stone. The stone was 4mm, a passable size, and she was sent home with analgesics and advised to hydrate well and strain her urine. She was straining her urine and noticed a small, dark fleck. She brought the sediment to the urologist's office to undergo a stone analysis, as this is her first episode of a renal or ureteral stone. Question What is the most likely composition of this stone?

Correct answer: Calcium Explanation The correct answer is calcium. Calcium stones are the most common type of renal calculi and can be further characterized as either calcium oxalate or calcium phosphate stones. Calcium oxalate stones make up about 60% and calcium phosphate stones make up about 20%, which results in calcium stones being about 80% of stones overall. Geography, fluid intake, and diet can all influence stone formation, but metabolism and genetics can also play a role. Absorptive hypercalciuria, renal hypercalciuria, and resorptive hypercalciuria can all result in calcium stone formation. Hyperuricosuria, gout, hyperoxaluria, and hypocitraturia are all other causes of calcium calculi. Uric acid is the not the correct answer, as this is not the most common type of renal calculus. Uric acid stones make up about 10% of stones in the United States. Hyperuricosuira and/or a urinary pH less than 5.5 are the 2 most common causes of uric acid stones. Gout, increased turnover of nucleic acids (such as in polycythemia or psoriasis), increased purine intake, and alcohol consumption are all causes of hyperuricosuria and can therefore put a patient at risk for uric acid stones. Struvite is not the correct answer, as this is not the most common type of renal calculus. Struvite stones make up about 7% of total renal calculi. Urinary tract infections secondary to the presence of urea splitting organisms can result in the formation of struvite stones. Klebsiella, Proteus, Staphylococcus, and Pseudomonoas produce urease. Urease breaks down urea and aids in the formation of ammonia. The ammonia then undergoes hydrolysis, which results in alkaline urine and reduced solubility of struvite, as well as urine that is supersaturated in struvite. Cystine is not the correct answer, as this is not the most common form of renal calculi. Cystine stones make up about 3% of total renal calculi. Cystine stones form due to an autosomal recessive disorder in the metabolism of cystine, which leads to cystinuria. Once the urinary saturation of cystine is more than 250 mg/L, cystine stones can start to form. Magnesium is not the correct answer, as this is not a type of renal calculi. In fact, magnesium has been known to be preventative of stone formation and is, therefore, a component in a lot of renal calculi prevention measures. Hypomagnesuria, usually dietary in nature, is a known risk factor for renal calculi formation. Maintaining urine magnesium about 50 mg/day is preventative.

Hypocitraturia is associated with:

Correct answer: Calcium oxalate nephrolithiasis Explanation Various substances have been proposed to inhibit renal stone formation, and in this regard, citrate is well studied. Hypocitraturia (<470mg/24 hours) is present in up to 60% of patients with nephrolithiasis. Hypocitraturia may have secondary causes, including chronic diarrheal states, renal tubular acidosis, hypomagnesemia, urinary tract infection, and thiazide-induced hypokalemia. Hypocitraturia is associated with idiopathic calcium oxalate stone formation, and after its correction, there is a notable reduction in stone forming events.

A 2-year-old boy has a solid mass of tumor on the lower pole of his right kidney. Histopathological studies show whorls of mesenchymal cells and clusters of disorganized epithelial cells rather than renal corpuscles and renal tubules, which confirm the diagnosis of Wilms tumor. Question What is true regarding treatment/prognosis of this diagnosis?

Correct answer: Combination chemotherapy is appropriate after surgery. Explanation Wilms tumor is one of the earliest examples where adjuvant systematic chemotherapy and surgical reduction of the tumor is employed with successful outcome even for advanced disease. Even with metastases to liver and lungs, or with bilateral tumors, patients still have better than a 50% chance of survival. Some cases of Wilms tumor are associated with mutations in the WT-1 gene, a tumor suppressor gene that is also involved in the development of the urogenital system. Consequently, Wilms tumor is commonly associated with developmental defects in the gonads. The tumor is radiosensitive, and advanced cases can be treated with radiotherapy, albeit with a high complication rate in young children.

An 83-year-old woman presents with vague periumbilical and epigastric pain for the past 7 days. The patient has a percutaneous gastrostomy tube that was placed 4 months ago for dysphagia following a cerebrovascular accident. She is able to eat now and has not used the tube for the past 2 months. The patient's pain is unchanged after eating but has limited her food intake. Blood gas results are pH = 7.44, pO2 = 92 mm Hg, pCO2 = 16 mm Hg; serum HCO3 is 12 mmol/L. Other electrolytes are as follows: Na 137 meqL, Cl 100 meqL. Question This arterial blood gas is consistent with what?

Correct answer: Combined metabolic acidosis and respiratory alkalosis Explanation The patient's bicarbonate level and pCO2 are low, but because this patient's pH is normal, a combined metabolic acidosis and respiratory alkalosis most likely exists. Anion gap is given by the formula [(Na)] -[(Cl) +(HCO3)] = 137−[100+12] = 25 meqL The normal anion gap is 8-16 meqL. The marked elevation in the patient's serum anion gap to 25 meq/L provides strong evidence of a severe metabolic disorder. The expected ventilatory compensation for metabolic acidosis is a decrease in PCO2. If this patient had simple metabolic acidosis, the expected PCO2 would be 21-25 mm Hg, but the marked decrease in this patient's PCO2 to 16 mm Hg confirms the presence of a concurrent respiratory alkalosis.

Case A 47-year-old Caucasian woman presents for evaluation of acute abdominal pain. She was brought in by her son, who reports the patient had not been eating or drinking well for several days. Further history, exam, and imaging studies were performed. The patient was pre-hydrated with sodium bicarbonate, had an abdominal CT with IV contrast, and was later admitted to the medical floor with a provisional diagnosis of diverticulitis. Her past medical history is significant for diabetes mellitus, which was diagnosed 12 years ago, and hypertension. Both conditions were reported to have been under good control. Her medications include regular and long-acting insulin and hydrochlorothiazide/lisinopril 25/20 mg QD. She has recently been taking 800mg ibuprofen BID-TID for her abdominal pain during the last week. She has no known allergies. While in the hospital, the patient's laboratory results are followed daily. 2 days after the CT with contrast, it is noted that her serum creatinine has risen to a level of 3.5 mg/dL. Records from 1 month ago at her family physician showed her labs to include a hemoglobin A1C of 6.8%, creatinine of 1.2 mg/dL, GFR of > 60 mL/min/1.73 m2, and blood pressure of 127/78 mm Hg. Question Which of the following is a major risk factor for this patient's sudden decline in renal function?

Correct answer: Diabetes mellitus Explanation Diabetes mellitus is a major risk factor for renal impairment after administration of IV contrast. In one study, DM had an odds ratio of 5.47 for development of contrast nephropathy. A baseline GFR > 60 mL/min/m2 estimates nearly normal creatinine clearance. Individuals with significantly reduced GFRs are at high risk for contrast nephropathy. Because creatinine clearance is inversely related to the creatinine level, this patient's lower creatinine level also supports less risk for the contrast nephropathy. However, it was not enough to offset the risk from her diabetes. Caucasian race is not considered a major risk factor in developing contrast nephropathy. Race is not a major factor in the development of contrast nephropathy. However, if any race is implicated for higher risk, it would be African Americans. Pre-hydration with sodium bicarbonate is a means of reducing risk, rather than a major risk factor, for contrast nephropathy. Pre-hydration seems to confer a protective effect, preventing hypotension and decreased renal blood flow. The sodium bicarbonate has been suggested in many studies to be more effective than traditional saline (sodium chloride). Younger age is not a risk factor for contrast nephropathy. Elderly age is a risk factor, with particular concern arising in patients 75 years of age and older.

You are called by the local Emergency Room regarding a 56-year-old white female patient of yours who has scleroderma (systemic sclerosis) and end stage renal disease. Her family brought her to the ER because she states that she no longer wants dialysis and has missed both dialysis today and two days ago. This patient has an advanced directive kept on file at her dialysis center. How should you approach this case?

Correct answer: Discuss with patient, nephrologist, and family. If she gives informed refusal of dialysis and knows her prognosis, withdraw dialysis, offer palliative care Explanation Dialysis is a procedure for which patients (or those making their medical legal decisions on their behalf) must consent. The patient in this vignette has apparently opted to withdraw from dialysis. She understands her prognosis and is making an informed decision. She will eventually die without dialysis support. Leaving her out of the decision making process is inappropriate, as she is apparently able to make an informed cognizant decision. Involving her in the decision but still ordering dialysis against her will is also inappropriate. Not every patient will want to initiate or continue to pursue dialysis therapy, for a variety of reasons. The American Society of Nephrology and Renal Physicians Association addressed guidelines for dialysis withdrawal a few years back. These guidelines focus on addressing: Shared decision making Informed consent or refusal Estimating Prognosis Conflict Resolution Advanced Directives Withholding or Withdrawing Dialysis Addressing Special Patient Groups for whom the medical condition precludes the process of dialysis Consideration of offering a time-limited trial of dialysis for patients with uncertain prognosis and Addressing Palliative Care.

A 60-year-old man presents with difficulty initiating voiding, incomplete emptying, and increasing urinary frequency over the past few months. He has no history of stones, cancer, surgery, diabetes, or AIDS; he takes no medications. His physical exam shows a temperature of 98.6°F, a blood pressure of 128/78 mm Hg, suprapubic fullness, an enlarged prostate, and no peripheral edema. The remainder of his exam is normal. Urinalysis No protein, no blood, pH 6, unremarkable sediment, no glucose Post void bladder scan 225 mL Renal ultrasound No hydronephrosis, normal cortex, normal size Serum creatinine 0.9 mg/dL Serum sodium, serum potassium 139 meq/L, 3.9 meq/L Serum bicarbonate 24 meq/L Total white blood cell count 5x103 cells/mm3 Question What will help prevent the loss of renal function that may accompany his primary disorder?

Correct answer: Doxazosin Explanation This patient most likely has obstructive uropathy secondary to an enlarged prostate and will benefit from doxazosin, which will both relieve symptoms of urinary retention and prevent the renal damage it may cause. His enlarged prostate and elevated post-void residual (>200 mL) are consistent with the diagnosis. Obstructive uropathy can occur from compression of any portion of the urinary tract; prostatic enlargement or bladder stones may impede flow through the urethra, and oncologic and gastrointestinal processes may obstruct ureters. Neurologic diseases and medications may impair bladder emptying and lead to urinary retention. Prolonged acute obstruction and/or chronic obstruction may cause declining glomerular filtration rates, inability to concentrate urine, inability to properly acidify the urine, and dysfunctional sodium and potassium excretion. The patient currently has no disorders of bicarbonate or potassium, so he will likely not benefit from their administration. As above, he should be monitored for them because they do occur with chronic obstruction. Likewise, he currently has no evidence of infection (e.g., fever, leukocytosis, pyuria). He is predisposed to urinary infections given his impaired flow; if he does develop chronic urinary infections, they may contribute to intrinsic renal disease and scarring. The patient should continue normal intake of fluids, but with increased intake, his kidneys are at risk for failure if he cannot control his outlet flow obstruction.

A 26-year-old African American woman presents to her physician with complaints stemming from the symptoms of a urinary tract infection. Patient states that the infection began approximately 2 weeks after giving birth. As you begin your investigation, as part of your disease and disorder considerations, which of the following would be a likely cause of her type of condition?

Correct answer: E. coli Explanation Approximately 80% of urinary tract infections are caused by E. Coli, 15% by Klebsiella pneumoniae, and 5% by group B streptococci or staphylococci. Urinary tract infections during pregnancy are most common in black, multiparous patients with the sickle cell trait. The treatment of choice is nitrofurantoin (Macrobid) 100 mg twice a day for 3 days. Patients who have a poor response to this short course of therapy require extended duration of treatment for up to 3-4 weeks. A urinary tract infection is quite common during pregnancy and puerperium as a result of: 1. Trauma to the bladder from a normal vaginal delivery 2. A hypotonic bladder from conduction anesthesia 3. The use of catheterization 4. Untreated UTI during pregnancy The use of uterine contracting agents such as oxytocin or IV ergonovine is not associated with UTI.

A 23-year-old presents with frequent, painful urination and lower back pain. History and physical exam reveals suprapubic pain, no evidence of systemic illness or fever, and no history of hospitalization. A presumptive diagnosis of an urinary tract infection (UTI) is made, and a urine culture yields lactose-fermenting beta-hemolytic Gram-negative rods. Question What is the most likely etiology of the UTI?

Correct answer: Escherichia coli Explanation The most common cause of an uncomplicated UTI is Escherichia coli, which is responsible for up to 85% of outpatient cases. Proteus sp. and Pseudomonas sp. are most often the etiologic agents of UTIs in hospitalized patients. Pseudomonas does not ferment lactose. Klebsiella pneumoniae and Enterobacter spp. are rarely beta-hemolytic.

A 77-year-old right-handed woman presents with a history of right-side hemiparesis and global aphasia. She has long standing diabetes, for which she takes Metformin; she also has a history of well-controlled hypertension. CT shows large left hemispheric infarction. On examination, she is alert, and you notice that she appears queasy. The nurse suspects her nausea is due to the urine incontinence that was noted after the admission. Serum analysis shows elevated glucose. Glomerular filtration rate shows mild renal insufficiency; dipstick urinalysis is glucose positive, and post-void residual volume is 80 ml. Question What is the most likely diagnosis?

Correct answer: Functional incontinence Explanation This patient most probably has functional incontinence secondary to her CVA. With functional incontinence, a person is usually aware of the need to urinate, but because of a physical or mental reason are unable to get to a bathroom. This patient cannot walk to the toilet independently because of her reduced mobility, and she cannot communicate the need to urinate because of the impairment of both expressive and receptive skills. Diabetic cystopathy is a form of autonomic neuropathy. It is a chronic condition caused by selective damage to autonomic afferent nerves, leaving motor function intact, but impairing the sensation of bladder fullness; therefore, it results in decreased urinary frequency, overdistension of the bladder, and overflow incontinence. It is unlikely that your patient has overflow incontinence; her incontinence started after the admission, and post-void residual volume is less than 100 ml. Overactive bladder (also known as urge incontinence) is a chronic condition characterized by the involuntary loss of urine preceded by a strong urge to void; it occurs whether or not the bladder is full. This patient had no incontinence before the admission; therefore, it is unlikely that she has an overactive bladder. In general, renal insufficiency is an unlikely cause of urinary incontinence. Normotensive hydrocephalus is a chronic condition characterized by gait disturbance, urinary incontinence, and dementia. She had no such symptoms before the admission.

An 18-year-old male high school baseball player has been hospitalized with a severe throat infection, fever, and possible pneumonia. He had been taking a number of antibiotics, and his physician noted lower extremity edema and an elevated blood pressure. Ten days after being discharged, he began to note blood in his urine. You order a urinalysis, and the dipstick results are positive for blood and protein. Microscopic results are positive for RBCs and RBC casts. Question What is the most likely diagnosis?

Correct answer: Glomerulonephritis Explanation The clinical picture is suggestive of glomerulonephritis. Signs and symptoms of glomerulonephritis include hematuria, proteinuria, edema, and hypertension, usually occurring 7 to 10 days after the onset of acute pharyngitis. Chronic renal failure is most commonly caused by untreated or poorly-controlled diabetes mellitus, and untreated or poorly-controlled hypertension. There is no indication of untreated or poorly controlled diabetes mellitus, or untreated or poorly-controlled hypertension in this patient. Nephrolithiasis, or kidney stones, would present with severe flank pain, hematuria, and oliguria. There is no indication of flank pain or oliguria in this patient. Cystitis, or bladder infection or inflammation, would present with suprapubic pain, dysuria, nocturia, odd- or foul-smelling urine, an increase in urinary frequency, and no fever. On urinalysis, a cloudy appearance and white blood cells would be seen on microscopic examination. These symptoms are not present in this patient. Nephrotic syndrome presents with massive proteinuria (> 3.0 grams per 24-hour urine), hypoalbuminemia, peripheral edema, and hyperlipidemia. Oval fat bodies may be seen on urinalysis. These symptoms, with the exception of edema, are not present in this patient.

The mother of a 2-year-old Caucasian boy explains that the boy has had a 4-day history of fever and cries during urination. She has been giving him acetaminophen, but the fever keeps coming back. She also says that his urine "smells funny." Family history is positive for a 4-year-old brother with Down syndrome. His temperature is 39°C. Urinalysis and culture reveal urinary tract infection. A VCUG and a renal bladder ultrasonography reveal vesicourethral reflux (VUR). Question What is a risk factor for VUR in this boy?

Correct answer: His ethnicity Explanation Ethnicity is a significant risk factor for vesicourethral reflux (VUR) because it is 10 times more common in Caucasians than African-Americans. VUR is the retrograde flow of urine from the bladder towards the kidneys due to a dysfunctional vesicoureteric junction. It may occur alone or in association with other urological conditions like posterior urethral valves. Although prevalence is more common in male neonates, VUR is 5 times more common in female children compared to male children after the age of 1 year. Acetaminophen treatment and family history of Down syndrome have no association with VUR.

A 33-year-old African American woman develops a loss of appetite. She reports constipation and lethargy. She thinks that her urine output has increased. Her history is significant for the presence of sarcoidosis. On physical examination, some muscle weakness is present. Laboratory tests are ordered and are pending. An EKG is done; it shows a shortened QT interval. Question Based on the history and EKG changes, what electrolyte abnormality would be expected?

Correct answer: Hypercalcemia Explanation A shortened QT interval is seen with hypercalcemia. The time from the beginning of the QRS complex to the ending of the T wave is the QT interval. Sarcoidosis can be associated with hypercalcemia. Symptoms of hypercalcemia include anorexia, vomiting, constipation, and polyuria. Muscle weakness can be seen. None of the other electrolyte abnormalities listed are associated with sarcoidosis. Hyperkalemia can result in peaked T waves on EKG. The T wave is a reflection of ventricular repolarization. There can be widening of the QRS complex as well. The QRS complex is seen when the ventricle depolarizes. There would be a prolonged QT interval with hypocalcemia. The time from the beginning of the QRS complex to the ending of the T wave is the QT interval. Hypokalemia can result in a depressed ST segment and the presence of U waves. The time from the end of the QRS complex to the beginning of the T wave is the ST segment. Hyponatremia is not typically associated with EKG changes.

A 9-year-old boy presents with headache and dark cola colored urine that appeared 2 days after a respiratory tract infection, for which he was given amoxicillin. He has microscopic hematuria and proteinuria. The rest of his past medical history and family history are non-contributing. Physical examination reveals elevated blood pressure (120/80), and the rest of the examination is normal. Urinalysis shows hematuria (the presence of erythrocytes and erythrocytes casts) and protein loss of 2.8 grams. Question What is the most probable diagnosis?

Correct answer: IgA nephropathy Explanation Both gross and microscopic hematuria a couple of days after nonspecific upper respiratory tract infection and hypertension in male patients are highly suggestive on IgA nephropathy (Bergers disease). IgA nephropathy is the most common chronic glomerular disease worldwide. It may also be associated with gastroenteritis, acute or chronic renal failure, or may be asymptomatic when erythrocytes (RBCs), RBC casts, and proteinuria are discovered on urinalysis. Some patients also have hypertension. Pathophysiological mechanisms are subendothelial deposits of amorphous material that lead to vascular occlusions, mechanical RBC, and platelet damage, resulting in prothrombotic state. Alport syndrome is hereditary X-linked dominant hereditary nephritis that will also present with hematuria (asymptomatic or gross) 1-2 days after upper respiratory infection. This progressive hereditary nephritis will, however, be accompanied with bilateral sensorineural deafness and visual problems (patognomonical extrusion of central part of lenses into anterior ocular chamber). Amoxicillin side effects are not probable. Amoxicillin side effects include nausea, vomiting, rashes, antibiotic-associated colitis, and diarrhea, in addition to more rare side effects such as mental changes, lightheadedness, insomnia, confusion, anxiety, sensitivity to lights and sounds, and unclear thinking. Even allergy to amoxicillin presents with a change in mental state initially, followed by itching skin rash, fever, nausea, and vomiting any time during the treatment up to a week after treatment has stopped. Acute overdose of amoxicillin may manifest with renal dysfunction, lethargy, and vomiting, but this usually happens in very young children. Acute poststreptococcal glomerulonephritis can present with the same clinical picture: sudden hematuria, edema, and hypertension, usually together with non-specific constitutional symptoms. However, there is always a latent period between the streptococcal infection and the onset of signs and symptoms of acute glomerulonephritis. Latent period is 1-2 weeks after a throat infection and 3-6 weeks after a skin infection. Hemolytic-uremic syndrome (HUS) is acute renal failure associated with non-immune (Coombs-negative) microangiopathic hemolytic anemia and thrombocytopenia. It is the most common cause of acute renal failure in children (though it may occur in adults as well). In HUS, there is usually a prodromal gastroenteritis, fever, or bloody diarrhea for 2-7 days before the onset of renal failure, sometimes with central nervous system signs (irritability, lethargy, even seizures). Acute renal failure with anuria follows. Physical findings may reveal hypertension, edema, fluid overload, and severe pallor.

A 37-year-old man has a 5-hour history of left-sided flank pain. He was previously healthy and takes no medications or supplements. He passed several grainy pieces in his urine 2 hours ago and brought them to the emergency room. Calcium oxalate crystals were identified. What dietary modifications will help reduce chances of future episodes of nephrolithiasis?

Correct answer: Increased intake of fluids Explanation This patient has passed calcium oxalate stones in his urine. Increasing his intake of fluids will help prevent future episodes of nephrolithiasis. At least 2 liters/day of fluid intake is suggested to prevent recurrent nephrolithiasis. Increased intake of water, coffee, and beer will all lead to decreased urine concentration of stone-forming solutes; alcoholic beverages inhibit the action and secretion of anti-diuretic hormone, leading to decreased urine solute concentration. In addition, reducing the intake of oxalate-rich foods, such as rhubarb, green leafy vegetables, chocolate, tea, liver, nuts, and seeds, is recommended. Crystals form in urine when the urine is supersaturated with crystal-forming solutes such as calcium, phosphate, and uric acid. Some patients over-excrete solutes, while others drink inadequate amounts of fluids to keep solutes dissolved. Stones also occur when the urine is infected with urea-splitting bacterium (Proteus). Here, urea is broken down into ammonia and bicarbonate, which then forms ammonium hydroxide and bicarbonate, the components of struvite stones. Struvite stones consist of a triple phosphate of calcium, magnesium, and ammonium. Certain stone inhibitors, such as pyrophosphate, citrate, and magnesium, prevent crystal growth. In patients who have low levels of these inhibitors, stones are more likely to form. We do not know if this patient is deficient in urinary stone inhibitors, only that he passed calcium oxalate stones. 24-hour urine collections, preferably 2 samples drawn 6 weeks apart, are recommended to assess urinary citrate excretion. Decreased intake of oranges (being a type of citrus fruit) will likely decrease urinary citrate levels. This modification will be of little benefit if his urine citrate levels are normal and may be detrimental if his levels are low. Increased intake of liver is not recommended; liver is an oxalate-rich food. Increased sodium is not advised in a patient who is prone to developing urinary calcium stones. Increased sodium intake leads to increased calcium excretion, which may promote further calcium stone formation. Increased protein intake is not recommended for patients with risk or recurrent renal stones and should be limited to less than 80 g/day. Nephrolithiasis is a common problem, affecting some 2-9% of the population. Without preventative treatment, 50% of patients having an episode of nephrolithiasis will likely have recurrent episodes over the next 10 years. Because of its likely recurrence, measures to prevent recurrent nephrolithiasis (such as increasing water intake, decreasing sodium intake, and moderating calcium intake) are all important in patients who have documented calcium phosphate stones.

Question A 24-year-old woman presents with severe diarrhea that she has been experiencing for 3 days, with no medical issues before then. She now feels dizzy upon standing, her tongue is dry, and her eyes appear glazed. Her serum sodium concentration is 130 mEq/L. What finding is most likely?

Correct answer: Increased serum ADH concentration Explanation The patient has obvious symptoms of dehydration. She also has a low serum sodium concentration, which shows that the body's drive to conserve water supersedes that of maintaining an adequate sodium concentration. ADH makes the normally impermeable collecting tubules permeable to water. ADH promotes water reabsorption in the collecting tubules, so urine volume is decreased and urine osmolality is increased. The patient's condition can be readily explained in view of an increased level of serum ADH. Serum aldosterone will be increased, as the body will also be trying to conserve sodium. Atrial natriuretic peptide will be decreased for the same reason.

A 24-year-old man presents with fever and 1-week history of a transient maculopapular rash. His serum creatinine and blood urea nitrogen (BUN) are elevated. The urinalysis is significant for hematuria, pyuria, white blood cell casts, and eosinophiluria. Question What is the most likely diagnosis?

Correct answer: Interstitial nephritis Explanation The clinical picture includes fever, recent etiological exposure (e.g., drugs, infection), and maculopapular rash; coupled with with serum and urinalysis findings of elevated creatine, elevated BUN, hematuria, pyuria, white blood cells casts, and eosinopiluria, the picture is suggestive of interstitial nephritis. Interstitial nephritis typically occurs following medication administration, but can also occur in response to viruses or bacterial infections. It is an allergic reaction of the kidney that results in fevers, rash, arthralgias, hematuria, and eosinophilia. In acute tubular necrosis, the BUN and creatinine are elevated, but the urinalysis may show a brown color. On microscopic examination, muddy brown casts (pigmented granular casts), epithelial cell casts, and renal tubular cells would be seen, which is not the case in this patient. There is no history of diabetes or hypertension in this patient; therefore, diabetic nephropathy and hypertensive nephrosclerosis are ruled out. Lupus nephritis is a complication of systemic lupus erythematosus. It is an autoimmune inflammatory disorder that affects many organs. About 85% of patients are women. On urinalysis, hematuria and/or proteinuria are common findings.

Upon examining the genital area of a 22-year-old patient, small firm testes measuring 1 cm are palpated. Physical exam also reveals a small penis, sparsely distributed pubic, armpit, and facial hair, and enlarged breasts. His weight is normal for his height. He is 6'11" with long legs and a short trunk. What is the most likely diagnosis?

Correct answer: Klinefelter's syndrome Explanation Klinefelter's syndrome should be highly suspected when small firm testes that measure less than 2 cm are palpated, along with the following findings: sparsely distributed pubic, armpit, and facial hair, enlarged breasts, and tall stature with long legs and a short trunk. Although small testes may be indicative of myotonic dystrophy, hypopituitarism, estrogen use, orchitis, and cirrhosis, the testes are soft in these conditions as opposed to the hard testes found in Klinefelter's patients. Cryptorchidism, a condition in which testicles remain undescended, results in an underdeveloped scrotum. Normal testes size in a male adult is between 2 and 3.5 cm.

A 43-year-old Caucasian woman presents with a 1-day history of burning sensation during urination and foul-smelling urine. She says that she has the habit of frequently voiding her bladder about once every 2-3 hours. She is hypertensive and on treatment with thiazides. She is sexually active with 1 partner (her husband); they use condoms for contraception. Fluid intake and blood glucose are normal. Leukocyte esterase dipstick test is positive, and urinalysis reveals 10 WBC/mL and bacteria. Question What is the most applicable risk factor for the patient's diagnosis?

Correct answer: Length of the urethra Explanation The correct answer is the length of the urethra. The diagnosis in this patient is a urinary tract infection (UTI). The female urethra is shorter, about 4 cm, compared to the male urethra, which is 22.3 cm (mean) (SD 2.4 cm) in length. The short length favors easy ascent of uropathogens to the bladder. Other risk factors for UTI in women include recent sexual activity, urinary obstruction, instrumentation, and uterine or bladder prolapse. Repeated and complete voiding of the bladder has been shown to reduce the risk of UTI. Diaphragm and spermicide use, not condom use, is associated with higher risk of UTI. Hypertension and thiazides are not known to be associated with UTI.

A 76-year-old African-American man with a past medical history of diabetes, hypertension, and hyperlipidemia is referred to a urologist for the evaluation of supranormal PSA readings associated with a reduction in urinary stream, back and hip pain, hematuria, and weight loss. He has not yet been managed for these symptoms. An x-ray was done in the office which revealed the following image. Question What is the most appropriate pharmacotherapeutic agent for this patient at this time?

Correct answer: Leuprolide (Lupron Depot) Explanation This patient is demonstrating manifestations suggestive of metastatic prostate cancer, suggested on the x-ray as extensive sclerotic areas throughout the pelvis and femur. Androgen deprivation is considered the primary approach to the treatment of metastatic prostate cancer. However, this approach has been found to be palliative, not curative. The goals of pharmacotherapy for prostate cancer are to induce remission, reduce morbidity, and prevent complications GnRH agonists provide medical castration in patients with prostate cancer. They are used early and late in the course of the disease. GnRH agonists bind to the GnRH receptors on pituitary gonadotropin-producing cells, causing an initial release of luteinizing hormone (LH) and follicle-stimulating hormone (FSH) and consequently a rise in testosterone levels for a few weeks. However, sustained use of these agents causes a decrease in the production of LH and FSH, which in turn leads to a decrease in testosterone production in the testes, reducing testosterone to castrate levels or to below the castrate threshold (50 ng/dL). Leuprolide, a GnRH agonist, is indicated as a palliative treatment for advanced prostate cancer when orchiectomy or estrogen administration is not indicated or is unacceptable to the patient. It is a potent inhibitor of gonadotropin secretion when given continuously in therapeutic doses. Antimicrotubule chemotherapy agents such as docetaxel and cabazitaxel have demonstrated improvements in overall survival in patients with metastatic, castrate-resistant prostate cancer. Docetaxel is indicated in combination with prednisone for the treatment of patients with androgen-independent (hormone-refractory), metastatic prostate cancer. Antifungal agents such as ketoconazole produce a response similar to that of antiandrogens. These agents provide an alternative option that may produce clinical benefit if initial androgen deprivation therapy fails. These agents inhibit various cytochrome P-450 enzymes, including 11-beta-hydroxylase and 17-alpha-hydroxylase, which in turn inhibit steroid synthesis. Proscar (finasteride) is a 5-alpha-reductase inhibitor. It inhibits this intracellular enzyme that converts the androgen testosterone into 5α-dihydrotestosterone (DHT). It is indicated for the treatment of symptomatic benign prostatic hyperplasia (BPH) in men. Rapaflo (silodosin) is an alpha-adrenergic antagonist used in the treatment of BPH.

A 12-year-old boy presents with a 3-hour history of extreme, severe pain in the right testis. It started suddenly, is 8/10 in intensity, and does not radiate. It is associated with nausea and scrotal swelling. He never had such pain in his lifetime, and he denies any problem in urination. He has never been operated on, and he denies any history of trauma. He is allergic to penicillin. On physical exam, the child is in visible distress. Temperature is 37°C, heart rate is 95, blood pressure is 120/70 mm Hg, and respiratory rate is 20 per minute. Genital examination reveals enlargement and edema of the entire scrotum. The right testicle is erythematous and tender to palpation; it appears to sit higher and lies horizontally in the scrotal sac relative to the left side. The cremasteric reflex is absent ipsilaterally, and there is no relief of pain upon elevation of the scrotum (Prehn's sign). Abdomen is non-tender and tympanic to percussion in all 4 quadrants. Bowel sounds are audible. Chest auscultation shows normal vesicular breathing with mild crepitations over the lower lung fields. Cardiac exam reveals normal S1 and S2, without rubs, murmurs, or gallop. His initial labs show a hemoglobin of 14.5 g/dL, WBC of 13,000/mm3, platelets of 210,000/mm3, sodium of 140 mmol/dL, potassium of 3.8 mmol/dL, chloride of 95 mmol/dL, urea of 25 mg/dL, and creatinine of 0.9 mg/dL. Question What clinical feature helps most to differentiate the patient's condition from other causes of scrotal pain?

Correct answer: Loss of cremasteric reflex Explanation Testicular torsion is a true urologic emergency and needs to be differentiated from other causes of testicular pain (e.g., trauma, epididymitis/orchitis, incarcerated hernia, varicocele, idiopathic scrotal edema, torsion of the appendix testis). The finding of an ipsilateral absent cremasteric reflex is the most accurate and sensitive sign of testicular torsion. This reflex is elicited by stroking or pinching the medial thigh, causing contraction of the cremaster muscle which elevates the testis. The cremasteric reflex is considered positive if the testicle moves at least 0.5 cm

Case A 13-year-old boy presents for an annual examination; he has no complaints about his health. Upon physical examination, his body temperature is 98.3°F, his blood pressure is 150/100 mm Hg, and he shows a slight periorbital puffiness. He explains that recently he has been staying up late to read and has noticed 'puffy eyes' in the morning. Urinalysis indicates light-brown urine, low-level proteinuria, and no bacteriuria. Question What is the most likely diagnosis?

Correct answer: Nephritic glomerular disease Explanation The correct response is nephritic glomerular disease. Nephritic syndrome is characterized by low-level proteinuria and edema of the face. The syndrome is caused by lesions within the glomeruli that allow the escape of red cells into urine; the glomerular filtration rate decreases and fluid retention might cause hypertension. IgA nephritis is incorrect because there is no indication of prolonged or repeated urinary bleeding or sudden swelling of the hands and feet, which are symptoms of this more serious condition. Nephrotic glomerular disease is incorrect; the symptoms for nephrotic disease are similar to the ones for the nephritic syndrome, but there is massive proteinuria which leads to hypoalbuminemia, a condition in which large amounts of albumin are released in the patient's urine. Orthostatic proteinuria is incorrect because this condition occurs when the total protein in the urine is higher when a patient stands than when laying down or sleeping. There are no other symptoms, and it is not associated with any risk for renal disease. Urinary tract infection is incorrect because there is no evidence of bacteriuria or blood in the urine and the patient does not have any fever.

A 54-year-old man notices that he has very large urine output and he is constantly thirsty. In addition to urinating large volumes during the day, he awakens at night to urinate. He has a 25-year history of a bipolar disorder, which is treated effectively with lithium. His lab results are as follows: TEST RESULTS REFERENCE RANGE BUN 17 mg/dL 10 - 20 mg/dL Calcium 9.9 mg/dL 8.5 - 10.5 mg/dL Potassium 4.2 mEq/L 3.5 - 5.0 mEq/L Sodium 149 mEq/L 135 - 145 mEq/L Glucose (fasting) 109 mg/dL 65 - 110 mg/dL The results of his urinalysis are as follows: TEST RESULTS REFERENCE RANGE Urine dipstick blood Negative Negative Urine dipstick glucose Negative Negative Urine dipstick ketones Negative Negative Urine dipstick protein Negative Negative 24 hour urine protein 124 mg/24 hour <150 mg/24 hour Urine osmolality 40mOsm/kg 50-1400mOsm/kg Urine specific gravity 1.001 1.001 - 1.035 Urine pH 6.2 4.5 - 8.5 Question A water deprivation test with exogenous vasopressin administration is done under meticulous supervision. His urine osmolality does not increase. What is most likely the case?

Correct answer: Nephrogenic diabetes insipidus Explanation A known side effect of lithium is nephrogenic diabetes insipidus. This side effect is very common. Nephrogenic diabetes insipidus results when the collecting duct of the kidney does not respond to the antidiuretic hormone (ADH). Antidiuretic hormone (ADH) is also known as arginine vasopressin. It is secreted from the pituitary. It works on the collecting tubules of the kidney to conserve water. Nephrogenic diabetes insipidus is characterized by renal resistance to ADH. This results in a large output of dilute urine. There will be polyuria and polydipsia. Hypernatremia would be present. In contrast to central diabetes insipidus, plasma ADH level would be elevated and there is no response to the administration of vasopressin. Nocturia can be present. In complete nephrogenic diabetes insipidus, the kidney is unresponsive to ADH. Therefore, exogenous vasopressin (ADH) will not increase the urine osmolality. The syndrome of inappropriate ADH secretion (SIADH) is due to excess ADH (antidiuretic hormone) or an ADH like substance. The syndrome of inappropriate ADH secretion is characterized by concentrated urine. There will be an accompanying serum hyponatremia. Hyponatremia can cause central nervous system symptoms (obtundation, seizure, and coma). Central diabetes insipidus is characterized by a lack of antidiuretic hormone (ADH) secretion from the pituitary. Because of the deficiency of ADH, there is an inability of the kidney to concentrate the urine. This results in a large urine output of dilute urine. There will be polydipsia and polyuria. Nocturia can be present. Hypernatremia would be present. In contrast to nephrogenic diabetes insipidus, there is an increase in urine osmolality in response to the administration of vasopressin. Diabetes mellitus is due to an absolute or relative insulin deficiency. Untreated, it is characterized by polyuria, polydipsia, and polyphagia, and there will be an elevated blood glucose and glucosuria. Glucosuria is the presence of glucose in the urine. Psychogenic polydipsia is a psychological condition. A patient with psychogenic polydipsia drinks an enormous amount of water. Because of this large water intake, there is polyuria. However, with psychogenic polydipsia nocturia is usually absent. There will also be hyponatremia.

A 43-year-old Caucasian man with a 20-year history of bipolar disorder presents for the first time with long-term polyuria and polydipsia. He previously took lithium for mood stabilization for 15 years before initiating divalproex sodium therapy. He stopped using lithium because of the polyuria, but he felt that the polyuria never fully subsided. His weight is stable, and he has no other urinary complaints. His blood pressure is 115/80 mmHg and his physical exam is normal. His urinalysis shows no blood, cells, protein, glucose, nitrate, casts, or crystals. Question What is the most likely cause of his polyuria?

Correct answer: Nephrogenic diabetes insipidus Explanation This patient's symptom of excessive production of urine (polyuria) is most likely caused by nephrogenic diabetes insipidus secondary to lithium use. Lithium impairs the distal water reabsorption in the collecting ducts, mediated by vasopressin (ADH), leading to the production of large quantities of dilute urine. Unfortunately, lithium use for as short a period as 1 year can lead to irreversible damage of the renal tubules (via down-regulation and production of receptors and channels responsible for water reabsorption). Treatments for lithium-induced nephrogenic diabetes insipidus include A) amiloride, a distal-tubule acting diuretic which competes with lithium for access to ion channels and thus prevents the lithium-induced polyuria, and B) hydrochlorothiazide with a low-salt diet, in order to effectively decrease the quantity of urine produced. Depakote can be substituted for lithium, but as mentioned above, lithium may cause irreversible tubular damage. In central diabetes insipidus, decreased levels of ADH are produced by the posterior pituitary. Nephrogenic, not central diabetes insipidus, is the expected complication of lithium use. ADH is usually released in response to increases in serum osmolality and/or decreases in arterial volume. Non-osmotic causes such as nausea in post-opertive setting can also elicit ADH release. Head injury, granulomas, and other central nervous system abnormalities can lead to impaired ADH production and release. Central and nephrogenic diabetes are distinguished by a water deprivation test as described in the following table:

A 40-year-old woman presents to the emergency department with unremitting left flank pain. She denies dysuria or fever. Upon interview of the patient, she also noted that her urine output has decreased over the last few days. As you observe the patient, she is writhing on the gurney and unable to find a comfortable position. On further inquiry, the patient states that she has been trying to lose weight by increasing protein in her diet, exercising, and decreasing her normal fluid intake. On urinalysis, red blood cells are noted. Question What is the most likely diagnosis?

Correct answer: Nephrolithiasis Explanation The clinical picture is suggestive of nephrolithiasis (renal stones). Most patients with renal stones present with flank pain and hematuria. There are numerous causes of renal stones, but dehydration favors stones formation and a high protein diet can predispose a patient to stone formation. Acute renal failure can be associated with renal stones. Renal stones are categorized as a post-renal cause of acute renal failure; however, with acute renal failure, a rapid rise in BUN and creatinine would be seen on serum chemistries, which is not seen in this patient. Cystitis, or bladder infection or inflammation, would present with suprapubic pain, dysuria, nocturia, odd or foul smell to urine, an increase in urinary frequency, and no fever. On urinalysis, a cloudy appearance and white blood cells would be seen. These symptoms are not present in this patient. Chronic renal failure is most commonly caused by untreated or poorly-controlled diabetes mellitus, and untreated or poorly-controlled hypertension. There is no indication of untreated or poorly-controlled diabetes mellitus and untreated or poorly-controlled hypertension in this patient. Nephritic syndrome is characterized by proteinuria, hematuria, azotemia, red blood cell casts, oliguria, and hypertension.

Case A 35-year-old woman is seen acutely in the office with severe right-sided flank pain that has persisted for the last 6 hours and is radiating to her groin. She states this feels like childbirth. Her LMP was 3 weeks ago. On examination blood pressure is 120/88 mm Hg, pulse is 82/min, temperature 99.0°F and respirations 16/minute. On physical assessment, the patient appears in acute distress, with diffuse tenderness over the entire right abdomen and flank. Hemoccult is negative. Pelvic exam is limited due to the patient's inability to lie still. Urinalysis in the office reveals the following: Urinalysis Result Specific gravity 1.00 pH 4.8 protein negative blood + 1 glucose negative ketones negative bilirubin negative urobilinogen negative nitrates negative leukocyte esterase trace Microscopic examination of the urine reveals: +2 RBCs Question You refer her to the nearest healthcare facility for further work up directing testing towards her most likely diagnosis which is

Correct answer: Nephrolithiasis Explanation The correct answer is nephrolithiasis based upon her classic 'loin to groin' pain, which keeps the patient writhing, and the presence of hematuria. The leukocyte esterase could be reflective of an inflammatory response or possibly an infected stone. The pain of pyelonephritis is located at the costovertebral angle without radiation and is usually accompanied by fever, chills, nausea, and vomiting. There is significant pyuria with strongly positive leukocyte esterase and possibly nitrites on dipstick of the urine. Hematuria may be present early in the course of the infection, but if it persists, a work-up for nephrolithiasis or tumor would be indicated. Appendicitis most commonly presents with right-sided abdominal pain with rebound tenderness. Patients with peritoneal irritation usually prefer to remain motionless. Ruptured ectopic pregnancy could present with right-sided abdominal pain though it is usually limited to the lower quadrant with radiation to the shoulder when supine. It is not associated with hematuria. Cholecystitis would present with right-sided abdominal pain that is steady rather than colicky and is usually limited to the upper quadrant with radiation to the right posterior thorax. It is not associated with hematuria. References:

A previously healthy 27-year-old man has had left flank pain radiating to his left groin for the past 2 hours. His urine output is also decreased. He denies hematuria, dysuria, penile discharge, and pyuria. He has had no trauma. He takes no medications. His temperature is 98.6 Fahrenheit, his pulse is 95 beats per minute, and his blood pressure is 150/95 mm Hg. He has left flank tenderness. The remainder of his exam is normal. His urinalysis shows: pH 6 No white cells 10 red blood cells/high powered field No nitrate, leukocyte esterase, or bacteria Multiple needle shaped crystals Question What is the most likely diagnosis?

Correct answer: Nephrolithiasis Explanation This patient most likely has flank pain due to nephrolithiasis. Of the listed diagnosis, the constellation of flank pain, hematuria, crystalluria, and decreased urine output is most suggestive of nephrolithiasis. In symptomatic nephrolithiasis, crystals form from urinary solutes in the lower and /or upper urinary tract, obstruct urinary flow, and cause a variety of symptoms. Symptoms of nephrolithiasis may include fever and chills, dysuria, urinary frequency, hematuria, and flank pain. If a ureter is blocked by a lodged stone, urine flow may be decreased downstream from the blockage, leading to decreased urine output. Because symptoms of nephrolithiasis may be similar to other disorders, it is important to keep a differential diagnosis in mind for nephrolithiasis. Fever and chills are often due to bacterial or viral infections, including pyelonephritis, prostatitis, and urethritis. Hematuria may be due to calculi, urinary tract infections, renal masses, or urothelial tumors. Dysuria, with or without urinary frequency, may be due to nephrolithiasis, urinary infection, or interstitial cystitis. The patient above lacked the fever and chills, pyuria, and urinary nitrate/leukocyte

A 44-year-old diabetic woman presents with a 2-week history of lower extremity edema. She has no other symptoms. Physical examination of her lower extremities reveals bilateral 2+ pitting edema. Urinalysis: Appearance Pale yellow and cloudy Glucose Negative Bilirubin Negative Ketones Negative Specific gravity 1.025 Blood Negative pH 7.0 Protein 4+ Urobilinogen Normal Nitrite Negative Leuk. esterase Negative Microscopic examination: RBCs 0 - 2 cells/hpf WBCs 0 - 2 cells/hpf Few squamous epithelial cells noted Moderate oval fat bodies noted Question What is the most likely diagnosis?

Correct answer: Nephrotic syndrome Explanation The clinical picture is suggestive of nephrotic syndrome. Peripheral edema is the hallmark of nephrotic syndrome; this occurs when serum albumin levels drop below 3 g/dL. Other findings include positive urinary protein (large amounts); oval fat bodies ('Maltese crosses') may be seen on microscopic urine exam. Clinical findings associated with cirrhosis include symptoms of weakness, easy fatigability, disturbed sleep, muscle cramps, and weight loss. In 70% of cases, the liver is palpable. Jaundice is not an early sign, but a later symptom occurring when the disease progresses. Urine findings include darkened urine color, positive bilirubin, variable urobilinogen, and negative protein. These findings are not present in this patient. Clinical findings in cystitis include irritative voiding symptoms and positive urine cultures. On urinalysis, leukocyte esterase is usually present with positive or negative nitrites and a cloudy appearance. Microscopic exam may reveal WBCs and bacteria. These symptoms are not present in this patient. Findings with pyelonephritis include fever, flank pain, and symptoms similar to cystitis. These findings are not present in this patient. Nephritic syndrome can present with proteinuria, hematuria, azotemia, RBC casts, oliguria, and hypertension. These findings are not present in this patient.

A 28-year-old man with no significant past medical history is rushed to the local trauma center following a stab wound to his chest. Paramedics report that there was significant blood loss. The patient has lost consciousness, is oliguric, and extremities are cool and moist to touch. His physical exam is also remarkable for tachycardia, tachypnea, a depressed systolic pressure, an immeasurable diastolic blood pressure. Question What is the preferred initial pharmacologic agent of choice for this patient?

Correct answer: Normal saline Explanation This patient is presenting with signs and symptoms consistent with hypovolemic shock due to acute traumatic blood loss. 3 goals exist in the emergency department treatment of the patient with hypovolemic shock as follows: (1) maximize oxygen delivery - completed by ensuring adequacy of ventilation, increasing oxygen saturation of the blood, and restoring blood flow, (2) control further blood loss, and (3) fluid resuscitation. Current recommendations are for aggressive fluid resuscitation with lactated Ringer solution or normal saline in all patients with signs and symptoms of shock, regardless of underlying cause. If a patient is moribund and markedly hypotensive (class IV shock), both crystalloid and type O blood should be started initially. These guidelines for crystalloid and blood infusion are not rules; therapy should be based on the condition of the patient. Epinephrine is indicated as initial resuscitation management in cardiac arrest, anaphylactic shock, symptomatic bradycardia, and hypotension refractory to volume replacement. Epinephrine is associated with a host of adverse effects such as induction of pulmonary hypertension, tachyarrhythmia, myocardial ischemia, lactic acidosis, hyperglycemia and compromise hepatosplanchnic perfusion, oxygen exchange, and lactate clearance. The combination of hypertonic saline and dextran also has been studied because of previous evidence that it may improve cardiac contractility and circulation. Studies in the US and Japan have failed to show any difference when this combination was compared with isotonic sodium chloride solution or lactated Ringer solution. Somatostatin and octreotide infusions have been shown to reduce gastrointestinal bleeding from varices and peptic ulcer disease. These agents possess the advantages of vasopressin without the significant side effects. In the patient with GI bleeding, intravenous vasopressin and H2 blockers have been used. Vasopressin commonly is associated with adverse reactions, such as hypertension, arrhythmias, gangrene, and myocardial or splanchnic ischemia. Therefore, it should be considered secondary to more definitive measures. H2 blockers are relatively safe but have no proven benefit.

A 35-year-old woman just found out she is pregnant. She is experiencing polyuria, but she denies dysuria and incontinence. Her urinalysis is unremarkable. Her fetal ultrasound is normal, and her renal ultrasound shows normal physiological hydronephrosis of pregnancy. Her pre-pregnancy weight was 155 lbs, and she is 5 feet tall. Her calculated body mass index (BMI) is 30.3 kg/m2. She takes no medications. She smokes ½ pack of cigarettes/day. Question Assuming that the patient has a normal vaginal delivery and no episiotomy, how can you best prevent urinary incontinence postpartum?

Correct answer: Normalize weight Explanation This patient will benefit from lifestyle modification to normalize her body weight post-pregnancy. Dietary modifications, exercise, breastfeeding, and nutritional counseling may be helpful. She was overweight pre-pregnancy, with a BMI > 30 kg/m2. Obesity is a known risk factor for incontinence. It is also a risk factor for insulin resistance and diabetes, which may also contribute to various forms of incontinence. None of the other options listed below will help prevent incontinence; in fact, they may be risks for incontinence. Incontinence is common in pregnancy. Fetal compression of the bladder plus large volumes of urine due to suggested volume intake and increased glomerular filtration rates may contribute to this. Postpartum, vaginal birth, and changes in the laxity/strength of the pelvic floor may contribute to stress incontinence. Stress incontinence is characterized by the involuntary leaking of urine during stress or increases in abdominal and bladder pressure, such as coughing and sneezing. Bladder pressure at these times exceeds urethral pressure, allowing urine to leak through the urethra. Treatments for stress incontinence include pelvic floor exercises. By repeatedly contracting and relaxing the vagina and pelvic floor, leaking may decrease. In this patient's case, treating her cough with cough suppressants may additionally help with the urine leaks. In obese patients, 5 - 10% weight loss may also improve symptoms. Pessaries may be inserted into the vagina to increase urethral support. Urethral support can also be increased surgically by inserting a fascial sling or vaginal tape to support the urethra. Smoking cessation is laudable on many accounts. It may contribute to low birth weight in the baby and is risk factor for a variety of cardiovascular diseases in the mother. It is not currently considered a risk factor for incontinence. Episiotomy may be a risk factor for fecal incontinence, but it is not a known risk for urinary incontinence. Oxybutynin is an anticholinergic amine used in the treatment of neurogenic bladder and overactive bladder/urge incontinence. Reports of its use during pregnancy and lactation are not available (Micromedex). Duloxetine is used in the management of stress incontinence. It is a reuptake inhibitor of serotonin and noradrenaline. Its use is not suggested in pregnancy because it is category C and may have teratogenic effects.

Case A 42-year-old morbidly obese woman wants to lose weight. Previous attempts with dietary modification and exercise have been unsuccessful; she has never taken any stimulants or weight-loss drugs. She has no history of excessive analgesic use, joint disorders, pain, paresthesias, or hematuria. She has seasonal allergic rhinitis and takes antihistamines in the springtime as needed. Her body mass index (BMI) is 40.5 kg/m2 and her blood pressure is 135/78 mm Hg. Her fundascopic eye examination is normal. She has mild lower extremity edema, good peripheral pulses, and no venous stasis. Her yearly laboratories are as follows: Renal ultrasound Normal size, no cortical thinning, no masses Urinalysis 1.020, pH 6, trace protein, no blood, no cells, no casts, no crystals, no glucose Urine albumin/creatinine ratio 40 mg/g Serum calcium, corrected 9.5 mg/dL Serum phosphorus 3.5 mg/dL Serum creatinine 1.2 mg/dL Serum bicarbonate 24 mEq/L Serum potassium 4 mEq/L Serum chloride 100 mEq/L Hepatitis B antibody and antigen, hepatitis C antibody All negative Antineutrophil antibody Negative Fasting serum glucose 100 mg/dL Complement 3,4 Normal Question What is the most likely cause of her proteinuria?

Correct answer: Obesity-related glomerular disease Explanation Given the lack of findings to support alternate diagnoses, obesity-related glomerular disease is the most likely cause of this patient's proteinuria. Obesity is an increasingly recognized modifiable risk factor for progressive kidney disease. The risk for developing progressive renal failure increases incrementally with increasing body mass index. Patients with a BMI >40 kg/m2 have an approximately 7-fold increased risk of developing progressive renal failure compared to the non-obese (<30 kg/m2) population. An increased BMI poses a risk for renal failure, even in patients without diabetes and hypertension. In obese patients, a fixed number of nephrons are challenged to process the fluids and nutrients of an increased body mass. They respond by hyperfiltering, increasing glomerular volume, decreasing podocyte number/density, and eventually leading to obesity-related glomerulopathy (proteinuria, elevated serum creatinine) and/or obesity-related glomerular scarring (glomerular sclerosis, proteinuria). Weight loss of even 5 - 10% can significantly improve a patient's risk factors for obesity-associated disease. Given her severely elevated BMI, a gastric surgical procedure is likely indicated. A BMI >40 kg/m2 is an indication for such procedures. Diabetic kidney disease is a well-known cause of renal failure; it is one of the leading causes of end-stage renal disease in the United States. This patient does not meet the criteria for diabetes (e.g., fasting glucose >126 mg/dL, random glucose >200 mg/dL); she also has no glycosuria. Although microvascular disease may precede the diagnosis of diabetes, no diabetic changes were noted on her retinal exam. Membranoproliferative renal disease typically presents with proteinuria and hematuria. Hypertensive nephrosclerosis is also a very common cause of progressive kidney failure in the United States, but is an unlikely cause of this patient's proteinuria. It occurs at 2 - 3 times the rate in patients with consistently elevated pressures (150 - 160/80 mm Hg) than in patients with pressures in the 130 - 140/80 mm Hg range. This patient's current pressure is 135/78 mm Hg. If her pressure were consistently at this level, she would be prehypertensive. Knowing that her creatinine is elevated and that she has proteinuria, a more desirable pressure would be in the 120 - 130/70 - 80 mm Hg range in order to best prevent progressive renal function loss. In some cases, hypocomplementemia, positive hepatitis titers, and/or positive antineutrophil antibodies are also noted. The lack of these findings makes this diagnosis unlikely. Tubular disease may lead to low levels of proteinuria; compared to glomerular disease, much lower levels of protein are found in the urine. Also expected in tubular disease would be some electrolyte or acid-base dysfunction since the tubules are key in regulating acid-base and mineral reabsorption and excretion. The normal urine pH, serum bicarbonate, serum chloride, serum potassium, serum calcium, and phosphorus make this a less likely diagnosis.

A 40-year-old otherwise healthy nulligravida woman presents with involuntary loss of urine that occurs after drinking a small amount of water, when washing the dishes, when hearing water running, and sometimes for no reason discernable to the patient. It is preceded by suddenly feeling the need to urinate, and it happens both during the day and at night. Urine analysis and culture, pelvic, gynecological, and neurological examinations are normal. Cystometric studies show residual volume of 45 mL (normal) with involuntary detrusor contractions, starting already with 200 mL. Question What will be your therapeutic approach?

Correct answer: Oxybutinin Explanation Your patient most probably suffers from idiopathic hypertonic incontinence. Her bladder is best described as instable, spastic, or overactive. The most common cause of urge incontinence is involuntary and inappropriate detrusor muscle contractions (normally there are no detrusor contractions during cystometry studies). Cholinergic receptors are present mostly in the detrusor muscle, and their stimulation enhances urination. Therefore, their inhibition with use of an anticholinergic such as oxybutynin will prevent involuntary urgency and incontinency for no apparent reason. Psychiatric consultation will probably not prevent involuntary activity of the detrusor. It can be performed after the organic cause is excluded. Patients with normal urine analysis do not need antibiotics. You will not use neostigmine because it is an inhibitor of acetylcholinesterase. Acetylcholinesterase inhibitors such as neostigmine indirectly stimulate receptors in the detrusor and enhance urination. They could worsen her symptoms. Kegel exercises with voluntary contractions of pubococcygeus muscle will help the patient with some forms of stress incontinence. Your patient is nulligravida and has no signs of problems with pelvic relaxation.

Case A 68-year-old man with a past medical history of congestive heart failure, hypertension, and hyperlipidemia has been admitted to the hospital for the evaluation of anemia due to a chronic gastrointestinal bleed. He takes oral enalapril and furosemide for CHF. While hospitalized, he developed polydipsia, dizziness, and decreased urine output; he notes that his urine is concentrated. His physical exam reveals orthostatic hypotension, poor skin turgor, dry mucous membranes, tachycardia, and peripheral edema. His bloodwork was remarkable for a hemoglobin of 7.0, hematocrit of 30, and serum sodium of 149. His BUN to creatinine ratio was 42 to 1, while the fractional excretion of sodium (FENa) was less than 1 % and fractional excretion of urea (FEUrea) less than 35%. Question What is the next most appropriate step in the management of this patient?

Correct answer: Perform a red blood cell transfusion Explanation This patient's most likely diagnosis is acute renal failure or acute kidney injury (AKI) due to hypovolemia and prerenal causes. The current treatment for AKI is mainly supportive in nature; no therapeutic modalities to date have shown efficacy in treating the condition. Maintenance of volume homeostasis and correction of biochemical abnormalities remain the primary goals of treatment. Supportive interventions include, as necessary, correction of fluid overload with furosemide; correction of severe acidosis with bicarbonate administration, which can be important as a bridge to dialysis; correction of hyperkalemia; correction of hematologic abnormalities (such as anemia, uremic platelet dysfunction) with measures such as transfusions; and administration of desmopressin or estrogens. Dopamine in small doses (e.g., 1-5 mcg/kg/min) causes selective dilatation of the renal vasculature, enhancing renal perfusion. Dopamine also reduces sodium absorption; this enhances urine flow, which helps to prevent tubular cast obstruction. However, most clinical studies have failed to establish this beneficial role of low-dose dopamine infusion, and one study demonstrated that low-dose dopamine may worsen renal perfusion in patients with AKI. Furosemide can be used to correct volume overload when patients are still responsive; this often requires high intravenous (IV) doses. Furosemide plays no role in converting an oliguric AKI to a nonoliguric AKI or in increasing urine output when a patient is not hypervolemic. Dietary changes are an important element of AKI treatment. Restriction of salt and fluid becomes crucial in the management of oliguric renal failure, in which the kidneys do not adequately excrete either toxins or fluids. Because potassium and phosphorus are not excreted optimally in patients with AKI, blood levels of these electrolytes tend to be high. Restriction of these elements in the diet may be necessary, with guidance from frequent measurements. Therapeutic agents (such as dopamine, nesiritide, fenoldopam, mannitol) are not indicated in the management of AKI and may be harmful for the patient. All nephrotoxic agents (e.g., radiocontrast agents, antibiotics with nephrotoxic potential, heavy metal preparations, cancer chemotherapeutic agents, nonsteroidal anti-inflammatory drugs [NSAIDs]) should be avoided or used with extreme caution. Similarly, all medications cleared by renal excretion should be avoided, or their doses should be adjusted appropriately.

Your patient is a 42-year-old woman with a 1-year history of amenorrhea and galactorrhea. She also has been experiencing fatigue, somnolence, and easy bruising. She takes Metformin for type II diabetes and an ACE inhibitor for hypertension. On examination, you find an obese woman with peripheral edema (ankles and hands), hirsutism, and diabetic retinopathy; the rest of the examination is normal. Her prolactin levels are 144.8 ng/mL (normal range is 4.8 to 23.3 ng/mL); repeated prolactin is 110 ng/mL. TSH and free T4 are normal. Question What should be your next step in regards to making a diagnosis?

Correct answer: Perform kidney function tests Explanation The correct response is that you should perform kidney function tests. Your patient has galactorrhea, amenorrhea, and signs and symptoms that suggest a renal insufficiency (fatigue, somnolence, easy bruising, peripheral edema); the insufficiency was probably caused by long-standing diabetes (diabetic retinopathy develops after a long history of diabetes). Chronic renal failure elevates prolactin by decreasing peripheral clearance of the hormone. Evaluation of hyperprolactinemia should include a review of medications, including estrogen therapy, and history of fertility or gonadal dysfunction. Elevated prolactin levels can result in secondary hypogonadism. In general, signs and symptoms of hyperprolactinemia are due to either the excess hormone secretion (i.e., galactorrhea and amenorrhea) or local compression (e.g., new-onset or persistent headache, dizziness, visual changes, and vision loss). Since your patient has no signs of local compression, biochemical tests should be ordered before the imaging studies. Laboratory evaluation should include a repeat serum prolactin test, measurements of TSH and free T4, and a pregnancy test. If the results come back normal and if other diagnoses are excluded, the most likely diagnosis is a prolactinoma. A pituitary MRI should only be obtained in such cases. Elevated prolactin levels can result in secondary hypogonadism. Serum testosterone levels should be checked in men with galactorrhea. Visual field testing can be performed in individuals with specific visual complaints, especially loss or impairment of peripheral vision.

A 55-year-old woman with diabetes presents with a sensation of incomplete voiding and occasional incontinence. She has been diabetic for 15 years and takes glyburide and metformin, but no other medications. Her symptoms have worsened over the past few months, and they do not include dysuria or fever. Her physical exam shows a heart rate of 60 beats per minute, a blood pressure of 128/78 mm Hg, and a mild decrease in monofilament sensation on her feet. Her diagnostic studies show: Urinalysis No cells, no blood, no glucose, trace protein Fingerstick glucose 90 mg/dL Bladder scan 400 ml pre-void, 150 mL post-void Spot urine microalbumin 45 microgram/mg creatinine Serum creatinine 0.9 mg/dL Question What initial management is suggested for her urinary symptoms?

Correct answer: Recommend scheduled voiding. Explanation This patient likely has a neurogenic bladder as a complication of her diabetes and should be initially advised to attempt scheduled voids. By voiding on a schedule, she'll empty her bladder and prevent the excessive buildup of urine that may lead to incontinence. If scheduled voids are unsuccessful, she may try intermittent self-catheterization using a sterile technique. Unfortunately, any catheterization has an infection risk. Foley catheters, suprapubic catheters, and nephrostomy tubes may all become infected. Furthermore, voiding may be achieved without them. This patient did not fully empty her bladder with voiding, but she did not have a post-void residual consistent with urinary retention; post-void residuals >200 mL are suggestive of retention. Her problem may be both impaired sensation of voiding in addition to impaired muscle contractility; cystometry may elucidate this further. Bethanechol and neostigmine are cholinergic agonists and may stimulate bladder emptying. They have a variety of side effects, including bradycardia. This patient's heart rate is 60 beats per minute and bethanechol may not be advised. Diabetic cystopathy may be complicated by urinary tract infections, urinary leaks and retention, bladder stones, and chronic kidney disease (Sasaki). It may also exacerbate kidney dysfunction secondary to diabetic glomerulosclerosis. This patient has microalbuminuria (>30 mg urine spot microalbumin), a sign of diabetic nephropathy and a calculated glomerular filtration rate of 59 mL/min/1.73 m2. Every attempt should be made to preserve renal function, including initiating renin-angiotensin system blockade and preventing loss of renal function secondary to urinary tract obstruction.

Case A 45-year-old morbidly obese woman develops new onset flank pain and hematuria 2 months after initiating Orlistat. A urinalysis shows gross blood, but it is otherwise normal. A CT scan shows a 2 mm stone in her right ureteropelvic junction. With hydration and analgesics, her symptoms improve. After 8 weeks, her symptoms resolve, and a 24-hour urine shows: Volume 1.1 liters pH 6.5 Calcium 180 mg (normal) Citrate 330 mg (normal>320 mg) Oxalate 75 mg (normal<40 mg) Creatinine clearance 90 ml/min/1.73 m2 Additional labs serum liver function tests Normal Question She wants to continue Orlistat therapy. How should you prevent stone recurrence in this case?

Correct answer: Prescribe calcium supplements with meals Explanation This patient had a single episode of flank pain and hematuria, presumably due to oxalate-based kidney stones. She should be prescribed calcium supplements to be taken with meals. Orlistat has been reported in association with calcium oxalate stones. Orlistat inhibits gastric and pancreatic lipase, leading to malabsorption of ingested fats and their elimination in the stool. These fats may bind calcium, leaving gastrointestinal oxalates unbound and available for reabsorption and deposition in the renal tubules. In her workup for stones, a 24-hour urine showed increased levels of oxalate. Hyperoxaluria can be treated by using calcium supplements with meals. Dietary calcium binds to oxalate and allows it to be eliminated in the stool. Primary hyperoxaluria is a genetic disorder whereby hepatic enzymes (alanine glyoxylate aminotransferase) are deficient and oxalate is produced in excess. Oxalate deposits in various organs, including the liver, kidneys, etc. Liver transplantation alone, or with renal transplantation, in patients with renal failure has successfully treated this disorder. Cholestyramine is also used to bind dietary oxalate. Hypocitraturia can be a risk for renal stones because citrate inhibits stone formation; however, no hypocitraturia was noted, making the need to prescribe sodium citrate unnecessary. Excess sodium intake can exacerbate calcium-based stones by allowing for increased renal calcium excretion; therefore, sodium bicarbonate use is not advised in this patient. Bicarbonate supplements may further alkalinize the urine. Since calcium stones precipitate at high pH, bicarbonate supplements may exacerbate their formation. Implementing a 1.1-liter daily fluid intake is likely insufficient for the prevention of stones. In most patients, urine is concentrated with this volume of intake; concentrated urine promotes stone formation.

A 56-year-old African American man with a history of hypercholesterolemia and obesity presents with a 6-week history of reduced urinary stream. Upon further questioning, he also admits to generalized weakness, loss of appetite, and a dull lower backache during this time, which he attributes to being "out of shape". He denies fever, chills, chest pain, shortness of breath, abdominal pain, hematuria, frequency, hesitancy, flank pain, dysuria, abnormal penile discharge, a history of trauma, alcohol use, or smoking. He states that he had an extramarital affair about 2 months ago and did not use any barrier methods during sexual intercourse. The physical exam is notable for a man who is in no acute distress, and he has normal vital signs. He is found to have a distended bladder on abdominal exam; there is vertebral tenderness of the lumbosacral spine as well as a firm, nontender nodularity of prostate upon digital rectal exam. Question What is the most likely diagnosis in this patient?

Correct answer: Prostate cancer Explanation This patient's presentation is most consistent with prostate cancer. Risks associated with prostate cancer include a high-fat diet, family history, and African American ethnicity. Upon digital rectal exam (DRE), it may manifest as focal nodules or areas of induration within the prostate. Obstructive voiding symptoms can occur with prostate cancer or benign prostatic hypertrophy; however, the prostate is more likely to demonstrate the absence of the median sulcus in BPH, not nodularity, as is observed in cancer. Manifestations of metastatic and advanced prostate cancer may also include weight loss and loss of appetite, anemia, bone pain (with or without pathologic fracture, most likely of the lumbar spine), neurologic deficits from spinal cord compression, and lower extremity lymphedema secondary to lymph node metastasis. Acute bacterial prostatitis typically presents with fever, chills, malaise, arthralgias, myalgias, perineal or prostatic pain, dysuria, and obstructive and irritative urinary tract symptoms, including frequency, urgency, dysuria, nocturia, hesitancy, weak stream, and incomplete voiding. There may also be lower abdominal or back pain and spontaneous urethral discharge. The prostate will be tender, nodular, hot, boggy, or normal-feeling on digital rectal examination in acute prostatitis. Suprapubic abdominal tenderness and an enlarged tender bladder due to urinary retention may also be present. Absence of systemic symptoms and persistence of pain for at least 3 months indicates chronic prostatitis. Risk factors that favor the development of bladder cancer include cigarette smoking and exposure to industrial dyes or solvents. Common presenting findings include gross or microscopic hematuria and irritative voiding symptoms such as frequency and urgency. Metastasis may cause hepatomegaly, lymphadenopathy, and lymphedema associated with involvement of pelvic lymph nodes. A form of nephrolithiasis, ureterolithiasis is caused by calculi in the ureters. It presents as abrupt, severe, colicky pain in the flank and ipsilateral lower abdomen. There is often radiation to testicles or vulvar area with intense nausea with or without vomiting. There is significant costovertebral angle tenderness; pain can move to the upper/lower abdominal quadrant coinciding with the migration of the ureteral stone. Patients typically are constantly changing body positions, such as writhing and pacing about. Tachycardia, hypertension, and microscopic hematuria are common.

A 67-year-old man presents with severe flank pain, fever, postural dizziness, dysuria, inability to pass urine for 1 day, and mild confusion. He has experienced 2 episodes of urine retention and incontinence over the previous 2 months, the most recent of which required urethral catheterization and outpatient antibiotic treatment for E. coli. His past medical history is significant for hypertension, prostatic hypertrophy, non-insulin dependent diabetes mellitus, and cognitive impairment that requires minimal assistance in activities of daily living. His temperature is 102.02 degrees Fahrenheit (38.9 degrees Celsius); blood pressure (BP) 80/50 mmHg; pulse rate 114/minute; respiratory rate 43 breaths/minute; and physical examination reveals a tender distended bladder that drained 2700 mL of turbid urine. Question What will be the next diagnostic step in this condition?

Correct answer: Quantitative cultures of urine Explanation Sepsis is a state caused by the infection that manifest as disruptions in heart rate, respiratory rate, temperature, and WBC. When it worsens to the point of end-organ dysfunction (renal, liver dysfunction, brain, or heart), then it is called severe sepsis. Once severe sepsis worsens to the point where blood pressure can no longer be maintained with intravenous fluids alone, then it is called septic shock. Therefore, your patient most probably has septic shock due to the urosepsis. The most common cause of urosepsis is obstruction. Patients at higher risk are elderly patients, diabetics, and immuno-suppressed patients. The diagnosis of urogenital tract infection can be established with absolute certainty only by quantitative cultures of urine. A positive urine culture confirms, but is not diagnostic of, symptomatic urinary infection. A negative urine culture, however, has a high negative predictive value and is useful for excluding urinary infection. Quantitative cultures of urine will also show what the causative agent is and determine the antibiotic therapy. Your patient has symptoms of septic shock due to urosepsis, and only some patients with severe urosepsis may develop bacteremia. Blood culture is useful, but urine culture is both more specific and more sensitive in urosepsis. Plain abdominal radiograph will show the presence and extent of calcification and calculi within the kidney or urinary tract. It is of help in monitoring change in position, increase in size, or number of renal stones, but it will not contribute to your management at this point. Ultrasound scan can define kidney size, identify renal scars, and help in the evaluation of prostate gland and various complications of acute pyelonephritis, but the first and best step in diagnosis of urosepsis is to find the causative agent in the urine.

A 55-year-old man presents with increased urinary frequency, nocturia, and the inability to achieve an erection for the past few months; he is also experiencing difficulty starting and maintaining a steady stream. Rectal examination reveals a nontender, nonenlarged prostate with an isolated left posterior lobe nodule. Question What option is the most appropriate?

Correct answer: Refer the patient for a transrectal ultrasound of the prostate and order a PSA level. Explanation The correct response is to refer the patient for a transrectal ultrasound of the prostate and order a PSA level. The clinical picture is suggestive of prostate cancer. Signs and symptoms include those similar to benign prostatic hyperplasia (BPH) (e.g., urinary frequency, urinary hesitancy, nocturia, hematuria, and difficulty achieving erections). Most patients are asymptomatic. Prostate cancer may manifest as focal nodules or areas of induration within the prostate. Modern transrectal ultrasound provides high-definition images of the prostate and guides biopsy. Measurement of the PSA is useful in detecting and staging prostate cancer. Serum acid phosphatase would not be useful since it is found in many organs. A prostatic acid phosphatase would be more specific in diagnosing prostate cancer. Since a nodule was found in this patient, it needs to be investigated further; reassuring the patient would be considered an insufficient medical practice. Prazosin is an alpha-blocker used in the treatment of BPH. BPH usually results in a smooth, firm, elastic enlargement of the prostate, which is not seen in this patient. Norfloxacin is an antibiotic used in the treatment of urinary tract infections. Signs and symptoms of a UTI include painful urination, increased frequency, and an odd smell to the urine. Urinalysis may show positive leukocyte esterase, nitrates, and blood, with WBCs and bacteria on microscopic examination.

A 3-month-old male infant presents with his mother, who brought him in because she is concerned that he has had a large scrotum since birth. She notices that when the patient cries the scrotum seems to get larger. Physical exam shows enlarged scrotum on the right side that does not transilluminate. A soft, reducible mass is felt. While palpating, you feel something like 2 layers of silk being rubbed together. An ultrasound confirms the diagnosis. Question What is the appropriate management of this patient?

Correct answer: Referral to surgeon for surgery within 14 days Explanation A scrotal mass that does not transilluminate and worsens with crying, coughing, or Valsalva maneuver in an infant is highly suggestive of inguinal hernia. "Silk sign," as described in the question, is felt when the hernia sac is palpated over cord structures, and it further supports the diagnosis. Because there is a high rate of incarceration in infants with inguinal hernia (14 to 31%), surgical repair is the recommended treatment. One study found that waiting beyond 14 days after diagnosis of inguinal hernia increased the rate of incarceration by 7%. It is not an emergency surgery unless complications are present, but it should be done as soon as possible after the diagnosis. Watchful waiting until 1 year of age would be an appropriate treatment for infants with hydrocele because there are rarely complications; the hydroceles usually resolve on their own before 1 year of age. Scrotal slings are not typically used in infants. They are sometimes used in adults with inguinal hernias. Inguinal hernias in infants need surgical repair. The risk of incarceration is too high to send parents home with reassurance only. Reassurance with follow-up would be appropriate treatment for hydrocele. Patient education on incarceration (hernia that is not easily reduced) and strangulation (incarcerated hernia with signs of vascular compromise) is extremely important in all cases of hernias; however, this is not a sufficient treatment plan and follow-up should be sooner than 2 months.

A 55-year-old woman presents with a 2-month history of gross hematuria. She states she has no pain with urination, but the hematuria is persistent. On questioning, she states that she does have some progressively worsening left flank pain and persistent back pain over the past month. The pains are not debilitating, but they are nagging. She has no chronic medical problems. She admits to a 50 pack-year smoking history, and she states she is currently retired from her job as a teacher. Vital signs are within normal limits and physical exam reveals a left side abdominal mass. Urine dipstick only shows too numerous to count RBCs; urine cultures are negative. Question What is the most likely diagnosis?

Correct answer: Renal cell carcinoma Explanation Hematuria is the most common presenting sign of urinary tract cancer. Silent or painless hematuria suggests tumor or renal parenchymal disease. Renal cell carcinoma can present with flank pain, hematuria, persistent back pain, and an abdominal mass. It can also be found incidentally on CT scan, so the clinical picture points to cancer with renal cell carcinoma as the most likely diagnosis. Smoking is a risk factor for renal cell cancer. RCC is more common in men than women (2:1), and it has a peak incidence in individuals in their 50s. Bladder cancer often presents with gross hematuria, but it is most typically painless. Bladder cancer can present with pain if the cancer is blocking the ureter. The left flank pain and back pain are more characteristic of RCC. Bladder cancer is the second most common urologic cancer, and the mean age at diagnosis is 65. It is more common in men than women (2.7:1) and 98% of primary bladder cancers are epithelial malignancies (majority urothelial cell carcinomas). Acute cystitis typically presents with irritative voiding symptoms (frequency, urgency, dysuria) and suprapubic discomfort in addition to possible hematuria. Urinalysis will show pyuria, bacteriuria, and varying degrees of hematuria. Urine cultures will show specific organisms. Urethritis is inflammation of the urethra that presents with urethral discharge, dysuria, and itching. Urethritis is most often caused by an STD. This patient does not have a history of unprotected sexual intercourse or any other symptoms that would indicate this diagnosis. Ureteral calculi can present with hematuria; it also typically presents with flank or abdominal pain, but not abdominal mass. If the stone is in the ureter, it often causes some hydroureter with or without hydronephrosis, both of which cause some pain or discomfort. The patient has no past history of forming stones.

A 22-year-old woman develops fever, rash, arthralgias, and decreasing urine output 2 weeks after completing a course of penicillin to treat streptococcal pharyngitis. Her physical exam is normal. She takes no other medications or supplements and has no other past history or symptoms. Her laboratory work results are as follows: Total white blood cell count 7.0x103/cmm Eosinophils elevated Serum creatinine 1.5 mg/dl Serum potassium 4.0 meq/l Serum bicarbonate 23 meq/l Fasting Blood glucose 80 mg/dl Urinalysis no casts, no bacterial growth, 2 red cells per high powered field, numerous white cells, trace protein Erythrocytes sedimentation rate elevated Question What diagnostic study will confirm her diagnosis?

Correct answer: Renal biopsy Explanation This patient most likely has acute interstitial nephritis (AIN) secondary to her penicillin exposure. Of the listed choices, only renal biopsies yield information specific to acute interstitial nephritis. Renal biopsy is the criterion standard for diagnosing AIN. Lymphocytic and plasma cell infiltrates in the peritubular areas of the interstitium are noted. However, being an invasive procedure, it is not used in all patients, especially if the condition is mild or if the patient improves rapidly after removing the offending cause. Renal ultrasound may show slight increases in renal size and cortical echogenicity in AIN, but this may also occur with other renal conditions. A clinician may order this test to evaluate for other forms of injury (such as acute renal obstruction) in the evaluation of acute kidney injury. 24-hour urine samples are used to assess urine output, daily protein, electrolyte excretion, and creatinine clearance, a measure of renal filtering ability. Currently, calculations of creatinine clearance are made using the modification of diet in renal disease (MDRD) equation. Here, a 24-hour urine collection will tell us how well her kidneys are working, but it will not show the cause of their decline in function. Elevated Urinary eosinophils may be found in a variety of other diseases, including pyelonephritis and prostatitis. The positive predictive value of urine eosinophils for diagnosing AIN is low (Kodner). Gallium scans have limited predictive value for diagnosing AIN (Markowitz). Cortical necrosis (i.e., secondary to ischemia) unrelated to AIN and other diseases may cause similar patterns of uptake as AIN. If the patient was still on the medication suspected to cause the problem, it should be discontinued and never again used. This alone may cause resolution of her illness in a couple of weeks. Her symptoms, urine output, volume status, serum creatinine, and electrolytes should be monitored to evaluate for the need for dialysis. She can be started on prednisone therapy for 2 weeks, to be tapered thereafter.

A 29-year-old man has been trying to have a child with his wife for the past 3 years; they have not met with any success. The wife was thoroughly evaluated for infertility; the workup revealed no abnormalities. Examination of his inguinal region reveals an ill-defined tortuous swelling that increases with standing and coughing. Ultrasonography is suggestive of being a varicocele. His initial semen analysis reveals mild oligospermia. Question What is the best next step in management?

Correct answer: Repeat semen analysis Explanation Repeat semen analysis is the correct answer. The results of a single semen analysis are inadequate in making a diagnosis of infertility. For a multitude of reasons, counts vary over time. 2 or 3 separate counts should be taken at least 2 - 4 weeks apart. Despite the presence of a varicocele, if a repeat semen analysis is normal and the varicocele remains asymptomatic, there is no reason for operative intervention. Embolization of dilated veins is incorrect. Embolization is an option in treatment of varicocele. The dilated veins of the pampiniform plexus are embolized using interventional radiology. It is used in the presence of a symptomatic varicocele or if infertility is confirmed. Surgical mesh repair is incorrect. Mesh repair is used in the management of a hernia, not in a varicocele. Low dose testosterone is incorrect. It may be an option if infertility is due to hypogonadism due to low testosterone. It is not an initial step in management. Surgical excision of dilated veins is incorrect. If the varicocele is symptomatic or if infertility is confirmed, surgical excision is an option.

A 17-year-old male arrives comatose in the emergency department where you are on duty. There is no history on your patient and no visible signs of trauma. You draw blood for gases. If the blood gas results are pH 7.6, H+ 38 nEq/L, PCO2 32 mmHg, and HCO3- 23 mEq/L, what acid-base disturbance does your patient have?

Correct answer: Respiratory alkalosis Explanation With respiratory acidosis, his blood pH should be below 7.4 and his pCO2 should be greater than 40 mmHg. Respiratory alkalosis is indicated by a decrease in the normal PCO2 levels of 35-45. The underlying mechanism is increased respiration decreases CO2 in the body. Some causes of respiratory alkalosis include CVA, sepsis, DKA, hyperthermia, aspirin overdose, pneumonia, pulmonary embolism, asthma, congenital heart disease, altitude acclimatization. With metabolic acidosis, his blood pH should be below 7.4. While the patient is alkalotic, you would expect his pCO2 to increase as his respiratory system compensates for metabolic alkalosis. Normal pH is roughly 7.4.

A 32-year-old man reports anorexia, constipation, fatigue, thirst, weakness, drowsiness, nausea, and muscle pain that has developed over the last few days. He has been bedridden for the last 3 months after a traffic accident wherein his 3 cervical vertebras were fractured. A few weeks ago, a diagnosis of kidney stones and chronic renal insufficiency was established. The rest of his personal and family history is noncontributing. His physical examination today demonstrates quadriplegia, and the rest of examination is within normal limits. Question While waiting for the laboratory results, you order an EKG, which you expect will probably reveal what?

Correct answer: Short Q-T interval Explanation Your patient has nonspecific symptoms, but the history of spinal cord injury, immobilization, kidney stones, and renal insufficiency suggests the presence of hypercalcemia. Prolonged immobilization may result in hypercalcemia, hypercalciuria, and osteoporosis because of the suppression of parathyroid-1,25-dihydroxyvitamin D axis. Hypercalciuria develops within the first week after injury and continues for 6 - 18 months. Hypercalcemia will occur when the rate of calcium resorption exceeds the capacity of urinary excretion. It usually happens 4 - 8 weeks after spinal cord injury. Children, adolescents, and persons with impaired renal function are particularly prone to hypercalciemia. Elevated calcium in an immobilized patient increases osteoclastic bone resorption because of the lack of signals from active muscles through the osteocytes. The release of calcium suppresses production of parathyroid hormone, resulting in increased serum phosphate and reduced synthesis of 1,25-dihydroxyvitamin D. The most common ECG abnormality associated with hypercalcemia is a shortened Q-T interval. A patient with ventricular fibrillation will not be able to give you his or her history. QT prolongation is seen in hypocalcaemia because of the prolongation of ST segment. Fusion of QRS-T is an ECG characteristic of hyperkalemia. Hyperkalemia also may demonstrate wide low P-waves, wide QRS, loss of the ST segment, and tall tented T waves. Torsades de pointes are usually caused by hypokalaemia, hypomagnesaemia, or, sometimes, with hypocalcaemia.

Case The patient is a 35-year-old woman who presents as a new patient with urinary frequency, urgency, dysuria, and suprapubic discomfort for several months. Repeated urinalysis and clean catch urine cultures ordered by her primary care physician have been unremarkable. The urologist does not find any significant physical exam findings and decides to perform a cystoscopy under IV sedation. Findings include velvety red patches known as Hunner's ulcers, and a bladder biopsy is negative for cancer. Passive hydrodistention of the bladder is performed at the time of the cystoscopy and is found to provide the patient with minimal relief from her symptoms following the procedure. Question What medication would be an appropriate next step in this patient's treatment?

Correct answer: Sodium Pentosanpolysulfate (Elmiron) 100mg TID Explanation The scenario is describing a patient with interstitial cystitis (IC). Patients with IC have a 10:1 female to male ratio and are typically in the third decade of life. Symptoms usually include urinary frequency, nocturia, urgency, and bladder or pelvic pain. Physical examination is usually unremarkable and helpful at ruling out other causes of the patient's symptoms. The urinalysis and urine culture are usually unremarkable, which also rules out other differential diagnoses. Cystoscopy with hydrodistention under sedation is often used to diagnose IC by both the appearance of the bladder and the bladder capacity (not usually over 350cc). Hunner's ulcers seen during cystoscopy with hydrodistention are pathognomonic for interstitial cystitis, although they do not have to be present for a patient to have this diagnosis (only present in 5-10% of cases). The hydrodistention can also help to relieve symptoms, and can be an effective treatment for many patients with IC. However, if symptoms persist, then other treatment options are warranted. Altering diet and avoiding foods and beverages that are bladder irritants can be helpful in improving symptoms in patients with IC. Beyond these measures, there are various medications that can offer relief. Elmiron stands alone in its class of medications, but is similar to a class of medications called low molecular weight heparins. It prevents the irritation of the bladder wall that is the cause behind the patient's symptoms. This medication is prescribed 100mg TID and is a first-line treatment. It is the best choice of those listed as potential answers. Ciprofloxacin (Cipro) is an antibiotic commonly used to treat urinary tract infections (UTI). While UTI would have been high on the list of differential diagnoses for this patient, it was ruled out by the negative urinalysis and urine culture. Bisacodyl (Dulcolax) is a medication commonly used to treat constipation and would therefore not be an appropriate treatment for this patient. Hydrocodone (Vicodin) and acetaminophen/aspirin/caffeine (Excedrin) are both commonly used to treat pain. Hydrocodone is often prescribed to patients with IC, as chronic opioid use is not uncommon due to the occasional extreme nature of the pelvic pain. However, it would not be the next best treatment and is essentially masking symptoms and not treating the IC. Excedrin is a pain reliever, but it contains caffeine. Caffeine is a bladder irritant and should be avoided by patients with IC, as it can potentiate the symptoms. References:

A 29-year-old Caucasian man with a prior history significant for right cryptorchidism presents for a routine physical examination. His prior cryptorchidism was corrected by orchiopexy at the age of 6 months. He has no complaints at present. His physical exam reveals no abnormalities other than bilateral gynecomastia and a right testicular mass that is painless and firm; it measures approximately 1.5 cm in diameter. The right testicular mass does not transilluminate, nor does it disappear when the patient lies supine. There is no femoral or inguinal lymphadenopathy, nor are there palpable hernias. Question What is the most likely diagnosis?

Correct answer: Testicular malignancy Explanation This patient's physical exam findings suggest testicular malignancy. In the United States, the incidence of testicular cancer in African Americans is approximately 1/4 of that in Caucasians. Within a given race, individuals in the higher socioeconomic classes have approximately twice the incidence of those in the lower classes. Testicular cancer is slightly more common on the right side than on the left, coinciding with higher rates of right cryptorchidism. Of all acquired and congenital risk factors, cryptorchidism is the strongest associated risk factor associated with testicular cancer. Placement of the cryptorchid testis into the scrotum (orchiopexy) does not alter the malignant potential of the cryptorchid testis, but it does facilitate examination and tumor detection. The most common symptom of testicular cancer is a painless enlargement of the testis. Enlargement is usually gradual, and a sensation of testicular heaviness is not unusual. Acute testicular pain is seen in approximately 10% of cases and may be the result of intratesticular hemorrhage or infarction. Approximately 10% of patients are asymptomatic at presentation, and the tumor may be detected incidentally following trauma or by the patient's sexual partner. On physical exam, a testicular mass or diffuse enlargement is found in most cases. The mass is typically firm and nontender, and the epididymis should be easily separable from it. A hydrocele may accompany the testicular tumor and help to camouflage it. Transillumination of the scrotum can help to distinguish between these entities. Palpation of the abdomen may reveal bulky retroperitoneal disease; assessment of supraclavicular, scalene, and inguinal nodes should be performed. Gynecomastia is present in 5% of all germ cell tumors but may be present in 30-50% of Sertoli and Leydig cell tumors. Its cause seems to be related to multiple complex hormonal interactions involving testosterone, estrone, estradiol, prolactin, and hCG. Hemoptysis may be seen in advanced pulmonary disease. Findings consistent with epididymitis are an enlarged tender epididymis associated with fever, urethral discharge, and irritative voiding symptoms. A translucent, fluid-filled hydrocele should be visualized by transillumination of the scrotum. Orchitis presents with inflamed, painful, tender, and swollen testes. A varicocele, which is an engorgement of the pampiniform plexus of veins in the spermatic cord, should disappear when the patient is in the supine position. It is frequently described as a "bag of worms" that is separate from the testes.

Question What condition of the male genitourinary tract is a surgical emergency?

Correct answer: Testicular torsion Explanation Testicular torsion is a surgical emergency due to the resulting circulatory obstruction. The patient usually presents with acute pain and swelling. A varicocele is a condition in which varicose veins develop along the spermatic cord. Although infertility is associated with this condition, it is not a surgical emergency. Hydroceles (a painless fluid-filled mass within the tunica vaginalis), scrotal hernias, and cryptorchidisms (undescended testicles) are also not reasons for immediate surgical intervention.

A 56-year-old African-American man presents with urinary hesitancy, frequency, and nocturia. He has to get up and urinate 3 - 4 times per night, and he is not sure if he empties his bladder completely. He states that his symptoms have been getting worse over the past 2 years. His urinary stream is weaker than it was a 1 year ago. He denies blood in his urine, and there is no history of urinary tract infections, dysuria, or pain. He is otherwise in a good state of health, and he has no significant past medical or surgical history. Currently, he takes no medications, and he has no known drug allergies. His father and brother died of prostate cancer in their 50s. The remainder of the history and ROS is non-contributory. Vital signs are stable and the patient is afebrile. General physical exam is unremarkable. Genital exam reveals a circumcised penis with no lesions or discharge. There is no inguinal adenopathy. Testicles are descended bilaterally with no lesions, masses, or hernias. Rectal exam reveals a smooth prostate, with no nodules or tenderness. Urinalysis is normal, and prostate-specific antigen (PSA) test is within normal range for age. For men aged 50 - 59 years, the normal reference range is 0 to 3.5 ng/mL. After emptying 250 mL of urine, the post-void residual urine volume is 50 mL. Question What is the most appropriate intervention?

Correct answer: Terazosin Explanation The correct response is terazosin . Given the history, physical exam, and negative PSA, there is enough information to make the diagnosis of symptomatic benign prostatic hyperplasia (BPH); no further diagnostic studies are necessary. Benign prostatic hyperplasia (BPH), also known as benign prostatic hypertrophy, is a histologic diagnosis characterized by proliferation of the cellular elements of the prostate. A classic history is usually sufficient to make the diagnosis. Clinical manifestations include urinary hesitancy, urinary frequency, urgency, nocturia (awakening at night to urinate), decreased or intermittent force of stream, and/or a sensation of incomplete bladder emptying. Depending on the patient's preferences, the next step is to begin treatment; in most cases, medical therapy is initiated first. If the symptoms do not significantly interfere with the patient's life, he may choose to wait and refuse treatment once he is reassured that he does not have a life-threatening illness (as in this case). If he selects treatment, management begins with a selective α1-receptor blocker (e.g., doxazosin or terazosin). A medication specific for α 1A-receptor subtype, such as tamsulosin (Flomax), may be used in patients who cannot tolerate traditional α1-receptor blockers. If medical therapy fails, or if a patient has severe BPH with ongoing obstruction, retention of large volumes of urine, or recurrent urinary tract infections, surgical therapy should then be considered. The most commonly performed surgery is transurethral resection of the prostate. Finasteride is a 5 α-reductase inhibitor. If the patient does not receive sufficient relief from maximum doses of a α1-receptor blocker, it may be added; however, it may take up to 6 months for a 5 α-reductase inhibitor to result in a noticeable difference in symptoms. Finasteride is not a first-line treatment; the full therapeutic benefit of a α1-receptor blocker will be apparent within 4 - 6 weeks. Because of the family history, this patient has an increased risk of prostate cancer; however, transrectal ultrasound with prostate biopsy is not indicated. This diagnostic procedure should be reserved for suspicion of prostate cancer. Based on this patient's family history and because he is African-American (African-Americans have a 50% higher incidence of, and mortality from, prostate cancer in comparison with Caucasians), a healthy index of suspicion is astute. Given this patient's classic BPH presentation and his normal PSA, prostate cancer is of low probability at this time. Caution must be exercised when using PSA as a diagnostic tool to rule in or rule out prostate cancer; the USPSTF recommends against PSA-based screening for prostate cancer. In addition, the negative prostate exam on rectal probing, while classically taught to be important, adds no additional information in most cases; currently, it is not recommended by the United States Preventive Services Task Force. No evidence in the case points towards the need for urine culture and sensitivity. Post-void residual is a diagnostic tool used to determine if a patient with BPH will benefit from scheduled bladder catheterizations. A post-void residual >200 mL is associated with an increased risk of urinary tract infections. Scheduled catheterizations are usually reserved for cases in which medical and surgical interventions do not correct the problem; they are also used when medical and surgical interventions are contraindicated.

A 25-year-old woman develops fever and malaise 3 weeks after starting trimethoprim/sulfamethoxazole for treatment of a urinary tract infection. Her plasma total white count is 7x10 3/cmm, with elevated eosinophils on the differential. Her urinalysis shows many white cells, 2 white cell casts/high powered field, no red cells or casts, trace protein, and no bacterial growth. Her serum creatinine is 1.3 mg/dl. Her erythrocyte sedimentation rate is elevated. The remainder of her laboratory work is normal. What findings are expected on renal biopsy?

Correct answer: Tubulitis, interstitial infiltrate Explanation This patient has findings suggestive of acute interstitial nephritis (AIN) and most likely will have tubulitis and interstitial infiltrates noted if she undergoes a renal biopsy. AIN is an immune-mediated form of acute kidney injury (acute renal failure). Patients develop varying degrees of renal failure, characterized by changes in urine output, electrolyte imbalances, acidemia, and azotemia (elevations in serum creatinine with or without nausea, sleep disturbances, shakiness, etc.) in response to viral, bacterial, immunological, or pharmaceutical insults. A variety of antibiotics, ACE inhibitors, proton pump inhibitors, seizure medications, etc. are known to cause AIN. The development of AIN is not dependent on the size of the dose of the medication given. Many infections, including HIV, EBV, and mumps, can also cause AIN. Additionally, immunological diseases, such as lupus and Wegener's granulomatosis, can cause AIN. Diagnosis is made by history of exposure, findings if noted of rash/fever/arthralgias, and the presence of pyuria, white cell urinary casts, and elevated erythrocyte sedimentation rate. Urinary eosinophils may be noted, but may also occur in the setting of other bladder or renal abnormalities, so they are not specific. Treatment involves removal of offending drugs, treatment of infections, monitoring urine volume, and laboratories for the need for dialysis support. Steroids have been used if patients do not respond to the above within a few days, but they may not hasten recovery (Clarkson). Immunomodulatory therapy (e.g., cyclophosphamide) has also been used. Red cell tubular casts are suggestive of glomerulonephritis. Glomerulonephritis occurs secondary to infectious, immunological, and neoplastic processes. Kimmelstiel-Wilson nodules are characteristic of diabetic nephropathy. Diabetic nephropathy is a complex disease found in many type I and II diabetics. It is the leading cause of end stage kidney disease in the United States. Necrosis of proximal tubules is characteristic of acute tubular necrosis (ATN). ATN is a form of acute tubular injury more common in the inpatient setting. Medications, infections, and renal hypoperfusion secondary to sepsis +- cardiac failure may lead to injury of the renal tubules.

A 29-year-old man is seen in clinic for groin pain. He works as a cashier at a local supermarket. The pain increases through the day. When asked to point out its location, he localizes it to his left groin fold. Examination of his inguinoscrotal region reveals an ill-defined tortuous swelling that increases with standing, with a palpable thrill on coughing. Question What is the best next step in diagnosis?

Correct answer: Ultrasound Explanation Ultrasonography is the correct answer. Ultrasound reliably diagnoses varicoceles. Being quick, reliable and non-invasive, it is the diagnostic test of choice. Sensitivity increases with the concomitant use of Doppler. Pampiniform plexus venous dilation of over 2 mm with standing or Valsalva maneuver is considered diagnostic. Diagnostic laparoscopy is incorrect. The pathology lies in the inguinal region extending to the scrotum, and it does not need laparoscopy. Venography is incorrect. While it is highly accurate, it remains an invasive technique; there is exposure to ionizing radiation. It is rarely (if ever) necessary in the age of ultrasonography. Computerized tomography (CT) is incorrect. If other tests indicated testicular cancer, a CT of the chest, abdomen and pelvis may be undertaken to inspect for metastasis. MRI is incorrect. It is unnecessary because ultrasonography is of such high sensitivity.

A 69-year-old woman presents with a 3-month history of intermittent urinary incontinence. After further questioning, she reveals that she experiences leakage after having an intense need to void. Question This is an example of which of the following types of incontinence?

Correct answer: Urge Explanation The correct answer is urge incontinence. It is defined as the inability to delay voiding when the urge to void is present. It is the most common type of incontinence and is due to detrusor muscle hyperactivity. Stress incontinence is leakage of urine with increased abdominal pressure that typically occurs with coughing, sneezing, etc. Overflow incontinence is due to detrusor muscle inactivity and it may be idiopathic or related to a nerve dysfunction causing neurogenic bladder. When the detrusor muscle does not contract effectively, the bladder becomes overfull and is at risk for leakage. The post-void residual urine is elevated in these cases as well. Functional incontinence is due to inability to use the toilet due to cognitive or physical impairment.

A 6-year-old boy presents with his mother, who is concerned because he wets his bed 2 or 3 times a week at night. He has no urinary symptoms during the daytime. He is the second child of 3. He is in the 1st grade and is struggling with his performance. He has had no medical problems; there is no history of developmental delay, and he was the product of a normal uncomplicated pregnancy and delivery. Since the birth of the 3rd child, his behavior has been poor. The vital signs are normal, and examination of other body systems is unremarkable. Question What investigation is most important in this child's workup?

Correct answer: Urinalysis Explanation Enuresis means any involuntary loss of urine. If it is used to denote incontinence during sleep, it should always be qualified with the adjective "nocturnal." Nocturnal enuresis is defined as involuntary urination in sleep without urological or neurological causes after the age of 5 years, at which time bladder control would normally be expected. The classification of enuresis is based on whether the child has ever achieved bladder control. Primary enuresis refers to a child who has never been dry; whereas, secondary enuresis means the child has been dry for a period but becomes enuretic later. Another useful classification is based on a period when the child does not have bladder control: • Nocturnal enuresis: Enuresis at night only. • Diurnal enuresis: Enuresis during the day only. • Nocturnal and diurnal enuresis: Enuresis during both day and night. The pathophysiology of enuresis is not completely clear; however, immaturity on the part of the autonomic nervous system that controls the bladder is present in the vast majority of cases. Only 20% of children with enuresis have a psychodevelopmental disorder (lower intelligence quotient or behavioral disorder). Secondary enuresis is often associated with a stressful environmental event. Urinalysis is the only mandatory investigation for nocturnal enuresis. Treatment: the therapeutic approach is still based on empirical data. Therapy is aimed at alleviating the symptoms of nocturnal enuresis rather than at curing the condition. A behavior-modification program is the treatment of choice, including buzzer/bell and pad, positive reinforcement, charting progress to increase confidence and self-esteem, urinating before bedtime, avoiding liquids after the evening meal, and avoiding psychological trauma through blame or belittling the child. Complete cystometric evaluation is incorrect. Cystometric analysis is used to evaluate the bladder's capacity to contract and expel urine. Intravenous pyelography (IVP) is incorrect. IVP refers to a series of X-rays taken of the kidneys, their collecting or drainage system (the ureters), and the bladder. It is done to locate a suspected obstruction to the flow of urine through the collecting system. Renal ultrasound is incorrect. If enuresis also occurs in the daytime or if urinary flow is small or interrupted, a renal ultrasound and a careful neurologic examination are indicated. Urine culture is incorrect. Urine culture may be appropriate, but is not most important. Urine culture is useful mainly when history, physical examination, or both suggests infection.

A 19-year-old woman presents with a 3-day history of urinary frequency, urgency, dysuria, and suprapubic pain that has progressively worsened since spending the weekend with her boyfriend, with whom she has been in a monogamus relationship for 3 years. Physical examination reveals only suprapubic tenderness. There is no discharge or erythema in the vaginal canal. Question What is the appropriate test to confirm your suspected diagnosis?

Correct answer: Urinalysis Explanation This patient most likely has an uncomplicated case of acute cystitis. The classic symptoms of cystitis include frequency, urgency, dysuria, and suprapubic pain. Women often develop these symptoms after intercourse and can sometimes have hematuria, but usually do not have fever. A urinalysis will show pyuria and bacteriuria and varying degrees of hematuria. E. coli is the most common causative organism. The degree of pyuria and bacteriuria does not always correlate with the severity of symptoms. Urine cultures should show the offending agent, but these symptoms should be treated as cystitis even if the colony count is less than 105/mL. A KUB, MRI, ultrasound, or CT scan will not show cystitis. These studies can reveal urolithiasis and hydronephrosis, which could lead to urinary infections, but they are not warranted for the first presentation with the above symptoms.

A 24-year-old man presents with a mass in his left testicle. He said he noticed it the other day when he was performing a self-exam on himself. You palpate his scrotum and note a "bag of worms" consistency in his left hemiscrotum. Question What condition is this consistent with?

Correct answer: Varicocele Explanation The clinical picture is suggestive of varicocele. A varicocele is a collection of dilated and tortuous veins surrounding the spermatic cord in the scrotum. It is sometimes referred to as a "bag of worms." A hydrocele is a fluid collection in the scrotal space. Clinical features are scrotal enlargement, scrotal heaviness, and back pain. Paraphimosis is a condition where the foreskin gets trapped behind the glans of the penis. Spermatocele is a cyst found on the rete testis or the head of the epididymis. Balanitis is an inflammation of the superficial tissues of the glans penis.

Question A 3-year-old girl presents with progressive abdominal enlargement associated with abdominal pain and occasional vomiting. Physical examination shows a palpable mass over the right upper quadrant extending to the right flank. She looks pale and the BP is slightly elevated. Urinalysis shows microscopic hematuria. What is the most likely diagnosis?

Correct answer: Wilm's Tumor Explanation The clinical picture is suggestive of Wilm's Tumor. It accounts for most renal tumors in childhood during the first 5 years of life. It affects both sexes equally. It is a solitary growth that affects either part of the kidneys. There are congenital anomalies associated with it, most commonly the GUT anomalies, hemihypertrophy, sporadic aniridia, and intellectual disability. It is commonly manifested by an abdominal mass that is described as generally smooth, firm and rarely crosses the midline, and it causes abdominal pain and vomiting. Hypertension is seen in 60% of the patients either due to elaboration of renin by the tumor cells or due to compression of the renal vasculature by the tumor. Hematuria is also uncommon and mostly microscopic. CT scan confirms the diagnosis which will show an intrarenal tumor, therefore ruling out Neuroblastoma. Treatment is by surgical removal. Chemotherapy is indicated post-operatively for the residual tumor. Neuroblastoma is a malignancy of the neural crest. It is the most common solid tumor in children outside the CNS. It is slightly more common in males and whites and median age of diagnosis is 2 years old. It arises mostly in the abdomen either in the adrenal gland or retroperitoneal sympathetic ganglia followed by the thoracic area mostly seen in the posterior mediastinum. Other sites are the head, neck, and epidural area. Tumors in the head and neck region are sometimes associated with Horner's Syndrome (Mioisis, Ptosis, Anhidrosis, and Enophthalmos). Diagnosis is by CT scan or MRI but pathologic diagnosis is made by biopsy. Tumor markers such as VMA and HVA (Homovanillic Acid) help confirm the diagnosis. Treatment is surgery, chemotherapy, and radiation depending on the stage of the tumor. Nephroblastomatosis are immature renal elements called Nephrogenic rest. It is a Wilm's tumor precursor lesion that is both unifocal and deep within the Renal parenchyma (intralobar rest) or multi-focal (perilobar rest). Subsequent development of Wilm's tumor in the other kidney is more likely in patients with this feature; therefore prompt inspection of the contralateral kidney is necessary during surgery of the neprhogenic rest. CT scan follow-up should also be done. Renal Cell Carcinoma is rare during the first decade of life but can occur occasionally in teenagers. Initial presentations are abdominal mass and hematuria. Surgical resection may offer cure, but prognosis is poor with post-operative residual disease. Mesoblastic Nephroma is a massive, firm, solitary renal mass and is generally thought to be benign. It resembles Leiomyoma or low-grade leiomyosarcoma grossly and microscopically. It also accounts for the majority of congenital renal tumors. It is more often seen in males and noted to produce renin. Treatment is surgical resection.

Upon inspection of an 18-year-old man's scrotum, you note that the left side is underdeveloped and a testis is not palpable. There is no scrotal tenderness, swelling, or nodularity. Considering this is not an acute finding, what is the most likely diagnosis?

Cryptorchidism

A 34-year-old female comes to the ED complaining of epigastric pain and intractable nausea and vomiting for the last 24 hours. Her vitals are as follows: Temperature 38.1 C, HR 97 beats/minute, BP 90/63 mm Hg, RR 12 breaths/minute. Arterial blood gas and labs are drawn. Which of the following sets of lab values is consistent with her presentation? A)pH 7.39, PaCO2 3, chloride 102 mEq/L, bicarbonate 27 mEq/L B)pH 7.36, PaCO2 75, chloride 119 mEq/L, bicarbonate 42 mEq/L C)pH 7.46, PaCO2 26, chloride 102 mEq/L, bicarbonate 16 mEq/L D)pH 7.51, PaCO2 50, chloride 81 mEq/L, bicarbonate 38 mEq/L E)pH 7.31, PaCO2 30, chloride 92 mEq/L, bicarbonate 15 mEq/L

D) Incorrect Answers: Answer A: These lab values are all within the normal range. pH 7.39, PaCO2 3, chloride 102 mEq/L, bicarbonate 27 mEq/L Answer B: These values are consistent with respiratory acidosis. pH 7.36, PaCO2 75, chloride 119 mEq/L, bicarbonate 42 mEq/L Answer C: These values are consistent with respiratory alkalosis. pH 7.46, PaCO2 26, chloride 102 mEq/L, bicarbonate 16 mEq/L Answer E: These values are consistent with compensated metabolic acidosis. pH 7.31, PaCO2 30, chloride 92 mEq/L, bicarbonate 15 mEq/L

A 41-year-old woman has been hospitalized for over a week. Her laboratory results reflect an electrolyte abnormality and her EKG demonstrates peaked T waves and a widening of the QRS complex. These EKG abnormalities are characteristic of which of the following conditions? A)Hypercalcemia B)Hypocalcemia C)Hyponatremia D)Hyperkalemia E)Hypokalemia

D)Hyperkalemia can result in peaked T waves on EKG. The T wave is a reflection of ventricular repolarization. There can also be widening of the QRS complex. Incorrect answers: Answer A: Hypercalcemia refers to an elevated calcium level. A shortened QT interval is seen with hypercalcemia. Answer B: Hypocalcemia refers to a depressed calcium level that results in QT prolongation. Answer C: Hyponatremia refers to a depressed sodium level. Hyponatremia is not associated with EKG changes. Answer E: Hypokalemia refers to a depressed potassium level which can cause prolongation of the PR segment as well as decreased amplitude of the T wave. U waves can be seen in leads V4- V6. Patient has serum sodium 231 mmol/L and serum osmolality 280 mOsm/kg. Blood pressure is low. There is poor skin turgor, JVP cannot be assessed by physical exam. Which of the following is the best next step? A)Urinary cortisol B)Thyroid function tests C)Kidney biopsy D)Urinary sodium E)IV insulin [D) Hypovolemic patient and hyponatremia. The best next step in the workup is to assess urinary osmolality/sodium to differentiate intrarenal (diuretics, mineralocorticoid deficiency, salt wasting syndrome), from extrarenal causes (diarrhea, vomiting, blood loss, sweating).

A 44-year-old female with diabetes presents with a 2-week history of lower extremity edema. She has no other symptoms. Physical examination of her lower extremities reveals bilateral 2+ pitting edema. Which is the most likely diagnosis? A)Acute glomerulonephritis B)Exogenous nephrotoxin C)Idiopathic membranous nephropathy D)Nephrotic syndrome E)Nephritic syndrome

D)The clinical picture is suggestive of nephrotic syndrome. Peripheral edema is the hallmark of nephrotic syndrome; this occurs when serum albumin levels drop below 3 g/dL. Other findings include positive urinary protein (large amounts); oval fat bodies (Maltese crosses) may be seen on microscopic urine exam. Acute glomerulonephritis occurs in the setting of hypertension, edema, azotemia and renal salt and water retention. It is the sudden onset of hematuria, proteinuria and red blood cell casts in the urine. This case does not present with such signs and symptoms. An exogenous nephrotoxin such as an aminoglycoside antibiotic can cause nephrotoxicity. This patient has not taken any medication, so this answer choice is incorrect. Idiopathic membranous nephropathy is a condition caused by autoantibodies to podocyte antigens or circulating factors like cytokines or microbial products that may induce podocyte CD80.Nephritic syndrome can present with proteinuria, hematuria, azotemia, RBC casts, oliguria, and hypertension. These findings are not present in this patient.

The effect of steroid therapy is evaluated in an 8-year-old Caucasian male being treated for fatigue and generalized edema following a Ηbad cold.Η His vitals are T ϯϳщьC, pulse ϵϬ/min, RR 20/min, and BP 110/70. Physical exam reveals the presence of mild periorbital edema and marked peripheral edema in hands and feet with the remainder of the exam within normal limits. What is the most likely diagnosis? A)Cirrhosis B)Congestive heart failure C)Nephritic syndrome D)Nephrotic syndrome E)Protein malnutrition

D)The pediatric patient described is suffering from nephrotic syndrome. Glomerular diseaseinduced proteinuria is the most common cause of nephrotic syndrome in children due to damage to the glomerular filtration barrier, resulting in leakage of plasma proteins into the glomerular ultrafiltrate. Signs and symptoms in children include edema, urine protein: creatinine ratio >0.2/mg creatinine, heavy proteinuria (urine protein >40 mg/m2/hr.), hypoalbuminemia, and hyperlipidemia. The nephrotic range of proteinuria in children is higher than in adults (>40 mg/m2/hr.).Nephritic syndrome can present similarly with edema and proteinuria, but it also presents with hematuria (cola/smoky urine) secondary to inflammation of the glomeruli. Laboratory findings include proteinuria but not as heavy as nephrotic syndrome (less than 35 g/24 hours) and RBC casts. Cirrhosis can result in significant edema secondary to hypoalbuminemia, but it would not result in proteinuria, and liver enzymes would be elevated. Congestive heart failure is unlikely, as this patient's cardiovascular exam is normal. This would not explain the laboratory findings. Protein malnutrition (kwashiorkor) is characterized by bilateral symmetric peripheral edema secondary to malnutrition. This is most common in underdeveloped countries. Other physical exam findings consistent with malnutrition include dry peeling skin, dry hair that may be falling out, bradycardia, hypotension, and hypothermia.

A 17-year-old female presents to the emergency department with dysuria. She denies any hematuria or dyspareunia. Her last menstrual period was 3 weeks ago, and she denies any recent sexual activity. Her temperature is 99.7°F (37.6°F), blood pressure is 127/67 mmHg, pulse is 90/min, and respirations are 17/min. An unusual odor is detected on inspection of the vagina and some gray discharge is noted. Speculum exam reveals a normal cervix and a bimanual exam is unremarkable for adnexal masses or tenderness. What is the next best step in management? A)Complete blood count (CBC) B)Urinalysis and Pap smear C)Urinalysis, urine culture, and potassium hydroxide prep (KOH) D)Urinalysis, urine culture, KOH prep, and urine pregnancy test E)Urinalysis, KOH prep, and nucleic acid amplification tests for N. gonorrhea and C. trachomatis

D)This patient of reproductive age and ureteropelvic symptoms should be screened for pregnancy in addition to other diagnostic studies (e.g., urinalysis, urine culture, and KOH prep.) . Urinary tract infections and sexually transmitted infections can be a confounding differential diagnosis in a female patient of reproductive age. This patient may have bacterial vaginosis, trichomoniasis, or a urinary tract infection. Although the patient denies any sexual history, the physical exam findings do not agree with this assertion. Pregnancy should be ruled out given her sexual history and likely need for antibiotics. Incorrect Answers: Answer A: CBC is insufficient to assess this patient's signs and symptoms. A leukocytosis may be present if systemic signs were evident. Answer B: Although this patient has a high suspicion of sexual activity, Pap smears are not indicated until she is 21 years of age. Answers C & E: While these (urinalysis, urine culture, KOH prep, and nucleic acid amplification tests for N. gonorrhea and C. trachomatis) are reasonable diagnostic investigations, both answers lack a screening assessment for pregnancy.

A 66-year-old female with history of multiple sclerosis (in a wheelchair) complains of dysuria, frequent urination, intermittent gross hematuria and chronic low back pain. The pain is constant, nonradiating and located over her flanks bilaterally. In the past 12 months she has had three urinary tract infections and one hospitalization for acute pyelonephritis of the left kidney. Vitals show BP of 130/70 mm Hg, pulse of 85 beats per minute, and respirations of 12 per minute. Heart and lung examinations are within normal limits. Her abdomen is soft and nondistended without guarding and there is mild bilateral costovertebral angle tenderness. Pelvic and rectal examinations are normal. Urinalysis revealed urine sedimentation, a pH of 8.0, leukocyte esterase positive, nitrite positive, WBC 75-90/HPF, RBC 30-40/HPF. A KUB radiograph was done, which showed large calcifications in both kidneys. What is the most likely composition of these calcifications? A)Calcium oxalate dehydrate. B)Calcium oxalate monohydrate. C)Cystine. D)Magnesium-ammonium-phosphate. E)Uric acid.

D)This patient provides a classic history for someone at risk for struvite kidney stones. Struvite stones are a result of urinary infection with bacteria, usually Proteus species, which possess urease, an enzyme that degrades urea to NH3 and CO2. The NH3 hydrolyzes to NH4+ and increases urine pH to 8 or 9. The CO2 hydrates to H2CO2 and then dissociates to CO3 2-, which precipitates with calcium as CaCO3. The NH4+ precipitates PO4 3- and Mg2+ to form MgNH4 PO4 (struvite). Remember that these stones are common in immobilized patients such as this woman with multiple sclerosis. In such cases, the stones can be bilateral. In this case the KUB shows bilateral stones. The result is a stone of calcium carbonate admixed with struvite. Struvite does not form in urine in the absence of infection, because NH4+ concentration is low in urine that is alkaline in response to physiologic stimuli. Chronic Proteus infection can occur because of impaired urinary drainage, urologic instrumentation or surgery, and especially with chronic antibiotic treatment, which can favor the dominance of Proteus in the urinary tract. The formation of these struvite stones in the renal collecting system leads to staghorn configuration typically seen with this type of stone. These stones are also called infective stones, because they contain numerous infective bacteria within their structure where antibiotics cannot penetrate. Because the antibiotics cannot penetrate the stone, the stones must be removed if the infection is to be cured. Prophylaxis against recurring struvite stones requires the maintenance of sterile urine, high urine volumes, and decreased urinary phosphate levels. Incorrect Answers: Answer A & B: Calcium oxalate, as either monohydrate or dihydrate (less dense), is a major component of most urinary stones. Although these metabolites comprise a majority of all stones formed, they are not normally seen in patients who have recurrent urinary tract infections and staghorn calculi. Answer C: Cystine stones form in patients who have this condition. Cystinuria is the result of an inherited autosomal recessive defect in the renal tubular reabsorption of four amino acids: cystine, ornithine, lysine, and arginine (mnemonic: COLA). Patients who are homozygous for the disease excrete increased levels of cystine. The stones form in acid urine. Answer E: Uric acid stones form in patients who have high urinary uric acid levels. This may occur in those who have gout, myeloproliferative disorders (leukemia and lymphoma), those on chemotherapy, or patients who have chronic diarrhea. These stones form in acidic urine (pH less than 5.5) and are radiolucent (i.e., they are not seen on regular plain radiographs). The patient in this scenario had stones visualized on radiograph. Tips to remember: Struvite stones are commonly seen in patients who have recurrent urinary tract infection with a urease-producing organism (Proteus, Pseudomonas, Providencia). Escherichia coli does not produce urease and therefore does not form struvite stones. Struvite tones are frequently large enough to fill the renal pelvis. They are composed of ammonium, magnesium, and phosphate (struvite) admixed with calcium carbonate apatite. They are visible on plain abdominal x-ray films.

A 3-year-old female is brought to the clinic by her mother, who tells you that the child has not been eating well over the past month and has developed swelling in the abdomen. On exam, the child has a smooth abdominal mass that is the size of a baseball on the left side. Vital signs reveal a blood pressure of 134/82 mm Hg, temperature of 99.8 F, and respirations of 16 breaths per minute. Urinalysis shows only 1+ red blood cells, and CBC and CMP are within normal limits. What is the most likely diagnosis? A)Multicystic dysplastic kidney disease B)Splenomegaly C)Lymphoma D)Wilms tumor E)Renal cell carcinoma

D)Wilms tumors account for 95% of all urinary tract malignancies in childhood. The median age of diagnosis is 3. These tumors are usually a solitary unilateral mass. Children typically present with an enlarging smooth abdominal mass confined to one side of the abdomen. Other associated symptoms include abdominal pain, fever, hypertension (60%), and hematuria (25%). Multicystic dysplastic kidney disease is more common on the left side and is more common in boys. 60% of unilateral multicystic dysplastic kidneys involute in the first 3 years of life; instead of getting bigger to reveal a palpable mass, these typically get smaller over time in children. Splenomegaly in children is usually indicative of hepatic or hematologic disease. This child's CBC and CMP are within normal limits, making this diagnosis unlikely. Lymphoma often presents with painless, firm lymphadenopathy typically confined to 1 or 2 lymph node areas (usually the supraclavicular and cervical nodes). Mediastinal lymphadenopathy is also a common presentation. It can manifest with coughing and shortness of breath. Abdominal masses are not a common presentation of lymphoma, and CBC is typically abnormal in these patients. Renal cell carcinoma accounts for only 3% of childhood urinary tract malignancies. Wilms tumor is much more likely to be the diagnosis in a child.

An 8-year-old boy is evaluated for persistent bedwetting. He has never been continent, averaging 2-3 episodes of bedwetting per week. His urological evaluation revealed a normal bladder and urethra and no neurological problems. Lately, his problem has been a source of much embarrassment; he is unable to attend camp or sleepovers due to fear of wetting his bed. He has tried multiple interventions, including lifestyle changes, alarm systems, and reward systems. His physical exam shows no abnormalities. His parents are keen on a rapid resolution to his problems, and they insist treatment be initiated. What is the best therapy?

Desmporessin

A 60-year-old man presents with difficulty initiating voiding, incomplete emptying, and increasing urinary frequency over the past few months. He has no history of stones, cancer, surgery, diabetes, or AIDS; he takes no medications. His physical exam shows a temperature of 98.6°F, a blood pressure of 128/78 mm Hg, suprapubic fullness, an enlarged prostate, and no peripheral edema. The remainder of his exam is normal. He has been referred to urology. What medical intervention will help prevent the loss of renal function that may accompany his primary disorder?

Doxazosin

A 26-year-old African American woman presents 2 weeks after an uncomplicated vaginal delivery complaining of signs and symptoms of a urinary tract infection (UTI). Which pathogen is the most likely cause of this patient's UTI?

E coli

A 26-year-old G1P0 pregnant woman with type 1 diabetes presents to her obstetrician for her 20-week appointment. Over the past day, she has had to urinate more frequently; she has also developed a burning sensation with urination. Urinalysis reveals the following: what organism is most likely to be grown

E coli

A 28-year-old female receives a lung transplant due to cystic fibrosis. 2 months later, she is readmitted with left-sided paralysis. CT scan is consistent with basal ganglia stroke. After 2 weeks in the hospital, she develops cardiac failure and is intubated. A posterior mitral valve leaflet vegetation with severe regurgitation is identified by transesophageal echocardiography (TEE). She is referred for mechanical mitral valve replacement. Colonies of Aspergillus fumigatus are detected in the excised mitral valve leaflets. Intravenous antifungal therapy with amphotericin B is started. A week later, she presents with fever, shortness of breath, and edema. Laboratory results include elevated creatinine and increased fractional excretion of sodium epithelial casts on urinalysis. What is the most likely cause of her impaired kidney function? A)Renal abscess B)Prerenal azotemia C)Diffuse cortical necrosis D)Polycystic kidney disease E)Acute tubular necrosis

E) Patients with acute tubular necrosis (ATN) have elevated serum creatinine and FeNa and epithelial casts in the urine. Causes include ischemia (hypovolemic shock, sepsis, cardiac heart failure), toxins (aminoglycosides, amphotericin B, contrast media, lead, cisplatin), myoglobinuria in rhabdomyolysis, and hyperuricemia in acute tumor lysis. Patients with renal abscess present with fever, weight loss, chills, and vague abdominal pain. They typically have significant elevations in white blood cell count. Patients with prerenal azotemia have hyaline casts and low FeNa values (secondary to decreased renal perfusion).Diffuse cortical necrosis (DCN) is an acute generalized cortical infarction of both kidneys leading to atrophy of the cortex with preservation of the medulla. DCN is the pathological progression of ATN. DCN is associated with late pregnancy complications (placental abruption) and septic shock. Polycystic kidney disease is a genetic condition that leads to the formation of multiple cysts and irreversible destruction of the kidney parenchyma. These lab values are not consistent with polycystic kidney disease.

A 65-year-old man develops oliguria and peripheral edema over a period of weeks. Urinalysis reveals hematuria and proteinuria; examination of the urinary sediment reveal red cell casts. Radiologic and US studies fail to demonstrate obstructive lesion. Renal biopsy shows many glomerular crescents. The presentation is most suggestive of which of the following conditions?

Good pasture syndrome

A 60-year-old man with a history of recurrent sinus infections presents with hemoptysis and hematuria. Physical examination shows a temperature of 101°F, a blood pressure of 145/85 mm Hg, decreased breath sounds on his right lower lobe, and palpable purpura on his bilateral lower legs. No warm or swollen joints were noted. ANCA: positive dx?

Granulomatosis with polyangitis (GPA, Wegener's granulomatosis)

The mother of a 2-year-old Caucasian boy explains that the boy has had a 4-day history of fever and cries during urination. She has been giving him acetaminophen, but the fever keeps coming back. She also says that his urine "smells funny." Family history is positive for a 4-year-old brother with Down syndrome. His temperature is 39°C. Urinalysis and culture reveal urinary tract infection. A VCUG and a renal bladder ultrasonography reveal vesicourethral reflux (VUR). What is a risk factor for VUR in this boy?

His ethnicity

A mother brought her 2-month-old infant son to a pediatric clinic because, during micturition, urine ran from the opening at the bottom of the midline groove of the scrotum instead of from the tip of the penis. What is the most likely diagnosis?

Hypospadias

An 83-year-old man presents to his urologist's office with "problems down there." He is a poor historian, but his wife accompanies him and helps with details of the history of present illness. She does not report that he has any voiding issues and reports that he has never had to see a urologist before now. Further genitourinary review of systems questions is suggestive of phimosis. What physical examination finding would support this diagnosis?

Inability to retract the foreskin over the glans penis in an uncircumcised male patient

A 12-year-old African American boy presents with poor appetite; he has felt unwell for over a month. He cannot delineate any specific symptoms, except feeling "puffy." He denies pain, eating disorder, rash, depression, drug use, and fevers. The family denies recent travel. PMH features no recent or chronic illnesses. He has had no surgeries and takes no medications. His family history includes grandparents on both sides with hypertension; one died from a kidney problem. ROS is negative except for some change in the urine, which he describes as frothy. He denies dysuria, gross hematuria, polyuria, and nocturia. Vitals: T 97.9°F; R 14, HR 90, BP 160/94. Weight is up 2 lb from his charted weight 3 months ago. HEENT, neck/thyroid, lungs, cardiac, abdominal, musculoskeletal, neurological, and derm exams are unremarkable. Examination of extremities reveals bilateral 1-2+ edema in the upper and lower extremities, with 2+ pulses. what is the cause of the elevated proteinuria

Increased glomerular permeability to protein

A 76-year-old African American man with a past medical history of diabetes, hypertension, and hyperlipidemia is referred to a urologist for the evaluation of supranormal PSA readings associated with a reduction in urinary stream, back and hip pain, hematuria, and weight loss. He has not yet been managed for these symptoms. A pelvic X-ray done in office reveals osteoblastic sclerotic areas of pelvis and femurs bilaterally. What is the most appropriate pharmacotherapeutic agent for this patient?

Leuprolide (metastatic prostate cancer)

A 55-year-old woman with a 15-year history of type 2 diabetes presents for follow-up. Her spot albumin/creatinine ratio was 100 mg/g 4 months ago and was confirmed at 100 mg/g yesterday. Her urinary analysis shows no cells, casts, or blood. Her creatinine is 0.7 mg/dL, and her estimated glomerular filtration rate is 95 mL/min/1.73 m2. What medication should you prescribe to help prevent her progression from micro- to macroalbuminuria and to help prevent progressive decline in glomerular filtration rate?

Lisinopril (ACE inhibitor)

A 51-year-old woman who has been in a road traffic accident presents with multiple injuries. She has a urine output of 350 mL over the previous 24 hours. Her serum creatinine level has increased from 4.2 mg/dL to 4.8 mg/dL over the same period. Question A drug from what class could be used to convert this patient's oliguric renal failure to the non-oliguric type, and to facilitate further management?

Loop diuretic Explanation Loop diuretic drugs produce diuresis in patients with an acute renal failure (ARF), congestive heart failure, and acute pulmonary edema. Loop agents can increase the rate of urine flow and enhance potassium excretion in cases of acute renal failure. They are sometimes used to convert oliguric renal failure to non-oliguric renal failure, and to facilitate fluid and electrolyte management. They do not, however, shorten the duration of renal failure or reduce mortality. Potassium sparing diuretic drugs are mild efficacy drugs with a relatively weak diuretic action; they are often used in combination with loop diuretics or thiazides in patients with congestive heart failure. Thiazide diuretics are used to treat hypertension, congestive cardiac failure, and edema. Carbonic anhydrase inhibitors produce a mild diuretic effect; they are used to treat glaucoma. Osmotic diuretics may be used to reduce intracranial pressure in cerebral edema.

A 3-year-old child develops severe generalized edema following a viral infection. On the basis of clinical chemistry tests, a renal biopsy is performed with normal light microscopic findings. Which of the following abnormal laboratory values might be expected in this individual?

Low serum albumin levels

A 41-year-old man presents to his physician complaining of swollen ankles. He states that he has never had this problem before. Physical examination demonstrates marked lower extremity edema and periorbital swelling. His pulse is strong and regular. Urine dipstick is 4+ positive for protein but negative for blood and glucose. 24-hour urine collection demonstrates proteinuria of 6 grams per day. What is the most likely cause of this patient's disorder?

Membranous glomerulonephritis

A 40-year-old man presents with extreme tiredness and progressive swelling of his limbs and face. Over the last couple of days, his urine output has reduced markedly. On asking about his personal habits, he turns out to be a heavy smoker, smoking 40 cigarettes per day. Clinical examination reveals a well-nourished male with pitting edema of limbs. His pulse is 62/min. and blood pressure 150/100 mm Hg. His heart sounds and breath sounds are normal. His abdomen is soft with no obvious organomegaly. Laboratory tests reveal an increase in his serum total and LDL cholesterol, Apo-B, CII, CIII, triglycerides, and VLDL. Serum C3 and C4 complement levels are normal. Urinalysis shows presence of protein, 4.5 g/day, microscopic hematuria, but no red cell casts or leucocytes. Light microscopy of renal biopsy reveals diffuse thickening of the glomerular basement membrane without increase in cellularity. Fuchsinophilic deposits are observed on the sub-epithelial aspect of the capillary basement membrane with Masson Trichrome stain. Methenamine silver stain reveals sub-epithelial projections or "spikes." What is the most likely diagnosis?

Membranous glomerulopathy

A 2-year-old boy with marked edema is seen by his pediatrician. Based on his prior medical history, his current history, and his current physical exam, a biopsy of his kidney is done. The pathologist discusses the case with the primary doctor. He reports that he saw no change in the glomeruli with the light microscope. Congo red stain was used on the biopsy specimen, but this did not demonstrate anything. Electron microscopy was also done, and the exam showed the loss of epithelial foot processes. Dx?

Minimal change disease

A lady brings her 4-year-old boy to your office. The child developed puffiness around the eyes about 2 weeks back and was prescribed an antihistamine by the local practitioner. The puffiness did not subside and gradually his limbs started to swell. The anxious mother tells you that since the previous day the boy is lethargic and refuses to eat. On inquiry you find that the child's urine output has decreased over the past week. Urinalysis shows proteinuria of 6g/L and maltese crosses (under polarized microscopy). The urine is negative for hemoglobin and there is no hematuria. Serum cholesterol and triglycerides are raised. Serum albumin is 20g/L. Serum C3 and C4 complement levels are normal. Throat swabs and urine cultures are sterile. What is the most likely diagnosis?

Minimal change disease

A 44-year-old woman presents with a 5-day history of weakness, polyuria, polydipsia, and frequent vomiting. She is insulin-dependent and has diabetes. A physical exam reveals a pale ill-appearing woman in moderate distress. Vitals: BP 85/55 mm Hg, P 130/min, RR 18/min, T 37.3°C. Head, eyes, ear, nose, and throat (HEENT) exam reveals dry mucous membranes. Neck is supple. Lungs are clear, and other than tachycardia, the cardiac exam is unremarkable. The abdomen is diffusely tender in the epigastric area. Rectal exam is heme-negative. A room air blood gas shows pH 7.01, PCO2 19 mm Hg, PO2 85 mm Hg. Glucose 590 mg/dL, Na 131 mEq/L, Cl 85 meq/L, HCO3 10 mEq/L, BUN 29 mg/dL, creatinine 1.7 mg/dL. Serum is positive for ketones. what best describes patient's acid base status?

Mixed anion gap metabolic acidosis and metabolic alkalosis

A 44-year-old woman with diabetes presents with a 2-week history of lower extremity edema. She has no other symptoms. Physical examination of her lower extremities reveals bilateral 2+ pitting edema. 4+ protein urine and oval body casts Diagnosis?

Nephrotic syndrome

A 54-year-old man presents with a lump in his scrotum recently. After answering many questions about possible symptoms and undergoing a thorough genitourinary examination, it is determined that the lump is actually a collection of fluid within the patient's tunica vaginalis. What finding most closely supports the most likely diagnosis?

Non-tender fluid-filled lesion that transilluminates (hydrocele)

A 50-year-old Caucasian man with diagnosed peptic ulcer disease (PUD) has nausea and severe vomiting for 2 days. Physical examination shows a patient with confusion, pale skin, tachycardia of 140 bpm, and blood arterial pressure of 90/50 mm Hg. Further evaluation reveals hypochloremic hypokalemic metabolic alkalosis. Urgent management should initially include what IV fluids?

Normal saline

A 20-year-old male with a history of nephrotic syndrome is seen in clinic after a renal biopsy. He presented a week ago with anasarca and loamy urine without hematuria. His blood pressure is 100/60 mmHg. Examination reveals periorbital edema, with clear lungs, a normal cardiac examination and 4 + edema and anasarca. His 24-hour urine contains 8 grams of protein and his serum creatinine, is 0.8 mg /dL. His urinalysis show 4 + protein, but no red cells or casts. His renal biopsy is read as minimal change disease. Which of the following is the most appropriate next step in management ?

Oral prednisone

A 42-year-old man presents with lower extremity swelling. His past medical history and review of symptoms are otherwise negative. The patient looks comfortable, with vitals showing the following: BP 142/91 mm Hg, HR 90 beats/min, RR 16 breaths/min, T 98°F, height 5'9"', and weight 158 lb. His examination is only remarkable for 2+ pitting edema in the lower extremities. The patient is counseled on a low-salt diet. The abnormal laboratory values are as follows: 3+ protein coarse granular casts 2-5 WBCs 0-2 RBCs Serum albumin 2.1 gm/dL Serum creatinine2.0 mg/dL Serum BUN18 mg/dL Hemoglobin12.1 gm/dL next best step in treatment?

Order a 24-hour urine to quantitate urine protein. (nephrotic syndrome)

A 40-year-old otherwise healthy nulligravida woman presents with involuntary loss of urine that occurs after drinking a small amount of water, when washing the dishes, when hearing water running, and sometimes for no reason discernable to the patient. It is preceded by suddenly feeling the need to urinate, and it happens both during the day and at night. Urine analysis and culture, pelvic, gynecological, and neurological examinations are normal. Cystometric studies show residual volume of 45 mL (normal) with involuntary detrusor contractions, starting already with 200 mL. What is the best treatment in this patient?

Oxybutinin

A 15-year-old boy presents for a high school basketball physical. After his mother leaves the room, he asks if you can "take a look at something." He then lifts his shirt and tells you that he is developing "man breasts." He is very embarrassed. There is firm mobile tender tissue palpable under both nipples. This tissue is approximately 75 mm in size on the right side and 1 cm on the left. There is no nipple discharge on either side. The patient's height is 66", weight is 125 lb (BMI 20.2). What is the most important next step in the diagnostic workup?

Physical examination including genitalia (gynecomastia)

A mother brings her 2-year-old son for the evaluation of frequent febrile urinary tract infections (UTIs); the infections have been present since birth. He has just finished a 10-day prescribed trimethoprim/sulfamethoxazole (Bactrim) course, and he now has no problems. The mother thinks that the main reason for the frequent UTIs is because a boy is neither interested nor willing to use the toilet; he is almost always wet. Physical examination today is unremarkable. He is not circumcised. You ordered dipstick urine analysis and perform ultrasound that came back normal. What will be your next step?

Plan voiding cystourethrogram. (recurrence of febrile UTIs)

A 65-year-old African American man presents in your office for a follow-up for hypertension. 3 months earlier, you prescribed furosemide. He checks his blood pressure daily and states that it is markedly lower since he has been on the medication, but it feels like his heart is skipping a beat once in a while. What deficiency is most likely?

Potassium (due to furosemide)

A daughter brings her otherwise healthy 50-year-old father to your office, located in a small town in the foothills of the Appalachian mountains. She complains that her father has been more lethargic than usual, and she states that she has seen him go into the backwoods and return obviously inebriated. You suspect that he is drinking moonshine from a homemade still and that it is leaching poisonous metals/substances into the distillate. The patient is morose and has difficulty answering questions. The rest of the physical exam uncovers no other abnormalities. The blood gas analysis shows a pH of 7.3, a pCO2 of 30 mmHg, and a bicarbonate of 15 mEq/L. Further laboratory testing revealed sodium of 140 mEq/L and a chloride of 100 mEq/L. Given the clinical history and laboratory findings, the clinical picture is most consistent with which one of the following?

Question Ico-delete Highlights A daughter brings her otherwise healthy 50-year-old father to your office, located in a small town in the foothills of the Appalachian mountains. She complains that her father has been more lethargic than usual, and she states that she has seen him go into the backwoods and return obviously inebriated. You suspect that he is drinking moonshine from a homemade still and that it is leaching poisonous metals/substances into the distillate. The patient is morose and has difficulty answering questions. The rest of the physical exam uncovers no other abnormalities. The blood gas analysis shows a pH of 7.3, a pCO2 of 30 mmHg, and a bicarbonate of 15 mEq/L. Further laboratory testing revealed sodium of 140 mEq/L and a chloride of 100 mEq/L. Given the clinical history and laboratory findings, the clinical picture is most consistent with which one of the following?

A 68-year-old man with a past medical history of congestive heart failure, hypertension, and hyperlipidemia has been admitted to the hospital for the evaluation of anemia due to a chronic gastrointestinal bleed. He takes oral enalapril and furosemide for CHF. While hospitalized, he developed polydipsia, dizziness, and decreased urine output; he notes that his urine is concentrated. His physical exam reveals orthostatic hypotension, poor skin turgor, dry mucous membranes, tachycardia, and peripheral edema. His bloodwork was remarkable for a hemoglobin of 7.0, hematocrit of 30, and serum sodium of 149. His BUN to creatinine ratio was 42 to 1, while the fractional excretion of sodium (FENa) was <1% and fractional excretion of urea (FEUrea) <35%. What is the next step in the management of this patient?

RBC transfusion (AKI due to hypovolemia and prerenal cause)

55-year-old man presents with increased urinary frequency, nocturia, and the inability to achieve an erection for the past few months; he is also experiencing difficulty starting and maintaining a steady stream. Rectal examination reveals a non-tender, non-enlarged prostate with an isolated left posterior lobe nodule. What option is the most appropriate?

Refer the patient for a transrectal ultrasound of the prostate, order a PSA level. (prostate cancer)

A 65-year-old man underwent an emergency laparoscopic cholecystectomy and arrives at the medical unit postoperatively with a Foley catheter. He is in stable condition, and his vital signs are stable. He is tolerating a regular diet, but his urine output is low, and he is admitted for monitoring. He has a foley catheter at this time to monitor urine output. What procedure is appropriate in the prevention of catheter-associated urinary tract infections in this situation?

Remove the catheter and monitor urine output by other means.

A 30-year-old man presents to the ER with fever, malaise, and decreased urine output while taking ibuprofen for back pain. His temperature is 101°F, his blood pressure is 135/85 mm Hg without orthostatic change, and the remainder of his examination is normal. A post-void residual is normal, as is his renal ultrasound. His urine output is estimated at 1.5 L/day. Best treatment course ?

Stop Ibuprofen (Acute Interstitial Nephritis)

A 71-year-old woman is brought in by her daughter due to dizziness. The daughter is worried about the potential for a fall, especially since her mother recently (10 days ago) had surgery for a right ankle fracture. The patient denies syncope, but she feels lightheaded when she tries to stand. She lives alone and is in a walking boot with crutches. She admits that she does not feel very stable walking, but she had not been dizzy and lightheaded until 2 days ago. She has not had to use her narcotic pain medicine for the last 5 days, stating her ankle pain is controlled with ibuprofen alone. She had been limiting fluid intake so that she does not require frequent trips to the bathroom, preferring to remain in a chair all day. Her past medical history is remarkable for stable overactive bladder and hypothyroidism. Medications include oxybutynin, ibuprofen, and levothyroxine. Her only surgery is the recent ankle fracture repair. She denies allergies and use of tobacco, alcohol, and recreational drugs. On physical exam, the patient is wearing the boot and reports dizziness as she moves to the exam table. Exam is remarkable for mild tachycardia and decreased skin turgor, with the remainder of exam being normal. She is hypovolemic and hypernatremic. Appropriate intravenous fluid therapy is initiated. Assuming none of this patient's medications are outside normal dosing parameters, what is the most appropriate pharmaceutical intervention for her fluid and sodium status?

Stop Oxybutinin

A 7-year-old boy present to the pediatrician because his mother noticed a smoky color to his urine. Upon questioning the mother, it is revealed that eh child suffered a sore throat several weeks ago that was left untreated. Physical examination reveals hypertension and mild general edema. Urinalysis is significant for red blood cell casts. Which of the following is the microorganisms responsible for this child's illness?

Streptococcus pyogenes

A 10-year-old boy is brought to the clinic by his mother. She noted that his face is swollen, and he told her that his urine was cloudy and reddish. He has a history of falling and abrading the skin of his right thigh 2 weeks ago. The next day, the skin became red, hot, and tender; the infection was treated with a topical antibiotic ointment. The cellulitis gradually healed. What organism is the most likely cause of this patient's disease?

Streptococcus pyogenes (group A beta-hemolytic)

A 56-year-old African American man presents with urinary hesitancy, frequency, and nocturia. He gets up to urinate 3-4 times per night, unsure if he empties his bladder completely. This has been worsening for 2 years. His urinary stream is weaker than it was 1 year ago. He denies hematuria, dysuria, or history of UTIs. He has no significant past medical or surgical history. The remainder of the history and ROS is non-contributory. Vital signs are stable, and the patient is afebrile. General physical exam is unremarkable. Genital exam reveals a circumcised penis with no lesions or discharge. There is no inguinal adenopathy. Testicles are descended bilaterally with no lesions, masses, or hernias. Rectal exam reveals a smooth prostate with no nodules or tenderness. Urinalysis is normal, and prostate-specific antigen (PSA) test is within normal range for age. After emptying 250 mL of urine, the post-void residual urine volume is 50 mL. What is the most appropriate intervention?

Terazosin (BPH)

A 21-year-old man presents with cough, fever, and hemoptysis. Blood tests show significantly elevated BUN and creatinine. Immunofluorescent microscopy reveals a diffuse linear pattern of Jesus Take The Wheel Take the Whole Car fluorescence along the basement membranes of alveolar septa and glomerular capillaries. Which type of Hypersensitivity is associated with this disease ?

Type II

A 29-year-old man is seen in the clinic for groin pain. He works as a cashier at a local supermarket. The pain increases through the day. When asked to point out its location, he localizes it to his left groin fold. Examination of his inguinoscrotal region reveals an ill-defined tortuous swelling that increases with standing and a palpable thrill on coughing. What is the next step in diagnosis?

US (varicocele)

A 62-year-old man presents with persistent hematuria. He denies any dysuria, urgency, or frequency. The first episode of hematuria was discovered 4 months ago on a routine urinalysis for a workplace physical. A repeat urinalysis 6 weeks later again shows hematuria, confirmed by microscopic evaluation. The remainder of his urinalysis is within normal limits. His past medical history is remarkable for COPD and obesity. He has smoked 1.5 ppd x 45 years. He uses inhaled medications for his COPD and has NKDA. His physical exam is significant for a temperature of 100.5°F. Abdominal exam elicits mild tenderness in the left upper quadrant, but no masses are palpable (although his exam is limited by his obesity). Chest and abdominal CTs with contrast show a normal chest and a left renal mass enhanced by radiocontrast, suggestive of renal cell carcinoma (RCC). No metastatic disease was noted on imaging. A renal biopsy is pending. Assuming the biopsy supports the diagnosis of renal cell carcinoma, what referral would be most appropriate for this patient?

Urologist for radical nephrectomy (diagnose, stage, treat RCC)

A 3-year-old girl is brought to the clinic by her mother, who tells you that the child has not been eating well over the past month and has developed swelling in the abdomen. On exam, the child has a smooth abdominal mass that is the size of a baseball on the left side. Vital signs reveal a blood pressure of 134/82 mm Hg, temperature of 99.8°F, and respirations of 16 breaths per minute. Urinalysis shows only 1+ red blood cells, and CBC and CMP are within normal limits. What is the most likely diagnosis?

Wilms tumor

A 3-year-old girl presents with progressive abdominal enlargement associated with abdominal pain and occasional vomiting. Mother denies any past medical history. Physical examination shows a palpable mass over the right upper quadrant extending to the right flank. She looks pale and the BP is slightly elevated. Urinalysis shows microscopic hematuria. What is the most likely diagnosis?

Wilms tumor

A 35-year-old woman presents to the emergency department with pain with urination and urinary frequency. Vital signs are T 101.1°F, HR 90, BP 120/76 mm Hg, RR 16, and oxygen saturation is 99% on room air. She has costovertebral angle tenderness on examination. Which of the following is the most likely diagnosis?

acute pyelonephritis

A 23-year-old woman presents with dysuria and left flank pain. She is 27 weeks pregnant. Physical examination reveals a temperature of 38°C (100.4°F); there is tenderness with percussion over the left costovertebral angle. Urinalysis shows 15-20 WBC/hpf and 15-20 bacteria/hpf. What is the most appropriate management plan?

admit for IV antibiotics (pregnant)

A 43-year-old Caucasian woman, previously in good health, presents to the emergency department with headache, blurred vision, and dizziness. Symptoms started 3 days ago and progressively worsened. Past medical history: hypertension, hypothyroidism, prior cholecystectomy. No known drug allergies. Medications: HCTZ 25 mg daily, Diltiazem CD 120 mg daily, and Levothyroxine 88 mcg daily. She ran out of all medications 2 weeks ago. Vital signs were normal, except for blood pressure 210/114 in the right arm, 215/115 left arm, 220/100 right leg, and 215/112 left leg. Physical exam: Heart - no visible or palpable PMI; normal S1 and S2 without murmur, rub, or gallop. Pulmonary - few faint RLL crackles, which cleared upon coughing. Remainder of the physical exam, including neurologic exam, was unremarkable. CBC and BMP were unremarkable except for K+ 2.3 mEq/L. EKG - NSR with one PVC. Chest X-ray - clear lung fields; normal pulmonary vasculature. CT head - no evidence of intracranial pathology. Renal artery sonogram - Unremarkable. Patient was treated with IV Nitroprusside and IV KCl 40 mEq x 2 doses and was admitted for further treatment. Over the next 2 days, patient's blood pressures gradually normalized with medical therapy; potassium levels remained low despite treatment. best screening test for suspected diagnosis?

aldosterone-renin ratio (hyperaldosteroneism...hypotension and hyperkalemia)

A 55-year-old man presents with a strong sudden need to urinate. He feels his bladder spasms and sometimes has involuntary loss of urine. The problem started several months ago and seems to be worsening. He is very upset because it sometimes happens during business meetings. His urinalyses were always normal. He takes no medications. Careful examination shows that he has urge incontinence. Bladder training and behavioral techniques were unsuccessful and you decided to introduce medication. Before suggesting a medication for this condition, you will tell him that this drug may have some side effects: dry mouth, difficulty in urination, constipation, blurred vision, tachycardia, drowsiness, and dizziness. This may happen because you will be prescribing what type of drug?

anticholinergic

An 8-year-old boy presents to your office with his parents for an evaluation of his bedwetting. He is being teased about his bedwetting on overnight outings and his parents are concerned about his self-esteem. He has never had a prolonged period of dryness at night. He has no wet accidents during the day and has not had difficulties with stool incontinence. His parents report that his mother wet the bed until age 12. They state that he appears to be a very sound sleeper. He is on no medications and his past medical history is unremarkable. The physical examination is within normal limits. What is considered first-line intervention due to proven evidenced success rates?

bed wetting alarm system

A 17-year-old sexually active boy is seen for the onset of excruciating right testicular pain 3 hours ago. There has been no fever, testicular trauma, or prior episodes of testicular pain. Placing cold packs on the scrotum provided no relief. Examination revealed a Tanner V male with normal vital signs. The right hemiscrotum is red, painful, warm, and swollen. The testicle and surrounding structures could not be palpated due to edema and pain. Elevation of the scrotum did not diminish the pain. What is the priority action?

bedside US to determine blood flow

A 35-year-old man presents by EMS after MVA that occurred 24 hours ago. He was found lying by the road by a local citizen after the MVA. Emergency department evaluation shows a fractured pelvis, acute kidney injury, and bilateral hydronephrosis near the upper ureters. His kidneys are of normal size with normal cortical thickness. He was previously well, and his only medications were multivitamins. He has not yet voided, and he notes decreased urine output after his accident. What will most likely assist in urine flow and resolution of this acute kidney injury?

bilateral ureteral stents and nephrostomy tubes

A 62-year-old woman presents with bright red gross hematuria for the past 2 months. She states it is painless but persistent. She denies other symptoms. She has no chronic medical problems. Upon further questioning, she admits to a 50 pack-year smoking history, and she states she is currently retired from her job in a rubber factory. Vital signs are within normal limits, and physical exam is normal. Urine dipstick only shows red blood cells that are too numerous to count and urine cultures are negative. What is the most likely diagnosis?

bladder cancer

A 28-year-old man with no significant past medical history is rushed to the local trauma center following a stab wound to his chest. Paramedics report that there was significant blood loss. The patient has lost consciousness. He is oliguric and his extremities are cool and moist to touch. His physical exam is also remarkable for tachycardia, tachypnea, a depressed systolic pressure, an immeasurable diastolic blood pressure. What is the preferred initial pharmacologic agent of choice for this patient?

blood products (hypovolemic shock secondary to traumatic blood volume loss)

A 28-year-old woman with a 4-year history of end-stage renal disease secondary to diabetes has had intact parathyroid hormone (PTH) levels of 600-800 pg/mL (ref 150-300 pg/mL) for the past 15 months. She does not take prescribed phosphorus binders with meals and she routinely misses her dialysis treatments, during which she receives intravenous paricalcitol. What additional finding is expected in this patient?

bone softening

A 13-year-old girl is drowsy and unable to answer questions. Her mother says that she has been extremely thirsty lately and urinates frequently. Her father notes that the patient has also been fatigued. There is a fruity odor to the patient's breath. Blood gases are drawn and reveal the following: pH 7.3 CO 232 HCO3 17 What is the most likely cause of the patient's condition?

diabetes (diabetic ketoacidosis currently)

A 65-year-old Caucasian man presents for a routine physical. He states that he is concerned about the development of prostate cancer. His history is significant for benign prostatic hyperplasia (BPH), for which he underwent a transurethral resection of the prostate (TURP) 3 years ago. His social history is significant for a 50 pack-year smoking history, and he worked for 40 years as a coal miner. His father died of prostate cancer at age 76. What factor would most likely contribute to his risk of developing prostate cancer?

family hx

A 55-year-old Latina woman presents to establish care. She recently went to a health fair, where she had some basic serum chemistries drawn. Her serum creatinine was 1.5 mg/dL. On a questionnaire she completed in your waiting room, she noted that she has no known past medical history except for occasional muscular aches, for which she takes indomethacin (2-3 times in the past 8 months). There is no family history of renal disease. Her BP is 142/82 mm Hg, and her body mass index is 31 kg/m2. What additional study would be most helpful in determining the cause of her elevated creatinine?

fasting serum glucose (to r/o DM as cause)

A 77-year-old right-handed woman presents with a history of right-side hemiparesis and global aphasia. She has long-standing diabetes, for which she takes metformin; she also has a history of well-controlled hypertension. CT shows large left hemispheric infarction. On examination, she is alert, and you notice that she appears queasy. She has been incontinent of urine since admission. Serum analysis shows elevated glucose. Glomerular filtration rate shows mild renal insufficiency; dipstick urinalysis is glucose positive, and post-void residual volume is 80 mL. What is the most likely diagnosis?

functional incontinence (secondary to CVA)

An 18-year-old male high school baseball player has been hospitalized with a severe throat infection, fever, and possible pneumonia. He had been taking a number of antibiotics, and his physician noted lower extremity edema and an elevated blood pressure. 10 days after being discharged, he began to note blood in his urine. You order a urinalysis, and the dipstick results are positive for blood and protein. Microscopic results are positive for RBCs and RBC casts. What is the most likely diagnosis?

glomerulonephritis

Question A 2-year-old boy presents with a firm painless mass in his right testicle. It is determined that he has an endodermal sinus tumor. What tumor marker is most likely to be elevated?

hide Correct answer: Alpha-fetoprotein Explanation An endodermal sinus tumor is also called a yolk sac tumor, infantile embryonal carcinoma, embryonal adenocarcinoma of the prepubertal testis, or orchioblastoma. The presence of alpha-fetoprotein is very typical. Alpha-fetoprotein is seen with a variety of tumors, such as hepatocellular carcinoma, pancreatic carcinoma, testicular tumors, and others. 5-hydroxytryptamine is serotonin. Serotonin is an indolamine. The precursor for serotonin is tryptophan. Serotonin is a neurotransmitter. Serotonin is also the precursor to melatonin. As a tumor marker, 5-hydroxytryptamine (serotonin) is the major product seen with carcinoid tumors. Gastrin is a hormone that is ordinarily secreted by cells within the stomach. Specifically, gastrin is secreted by G cells, which are in the stomach antrum. Gastrin stimulates acid secretion of the stomach. Pathologically, gastrin can be produced by pancreatic islet cells tumors. Vasoactive intestinal peptide is sometimes abbreviated as VIP. Vasoactive intestinal peptide can be seen with islet cell tumors and pheochromocytoma. The prostate has the enzyme acid phosphatase. Acid phosphatase is actually a group of enzymes that can be found in a few other tissues as well. With the development of prostate cancer, elevated serum acid phosphatase can be seen.

A 2-week-old male infant presents with his father for evaluation of enlarged scrotum. The father states that the scrotum was a little larger in the first few days after birth than it is now, but it has not reduced in size enough to make him feel comfortable that it is normal. Physical examination reveals normally developed penis with abnormally large scrotum that transilluminates on the right side when light is shined on it. What is the most likely diagnosis?

hydrocele

A 68-year-old man presents with scrotal swelling he has had for the past few months. It is not bothering him, but his wife wants him examined. His history is not significant for any other GU history or symptoms. A genitourinary examination reveals a non-tender non-erythematous area of swelling located superior and anterior to the right testicle. The swelling becomes smaller and softer after laying down. A flashlight is held to the scrotum and it transilluminates well. What is the most likely diagnosis?

hydrocele

A 41-year-old woman has been hospitalized for over a week. Her laboratory results reflect an electrolyte abnormality and her EKG demonstrates peaked T waves and a widening of the QRS complex. These EKG abnormalities are characteristic of what condition?

hyperkalemia

A 65-year-old man presents with a 3-week history of progressive leg swelling, dyspnea on exertion, and increased thirst. A couple days ago, he started having nausea, headache, vomiting, and his gait became unsteady. He does not have chest pain or shortness of breath at rest. His medical history is significant for hypertension, type 2 diabetes, and chronic kidney failure. His medications include a beta blocker/alpha-1 blocker (carvedilol), aspirin, angiotensin-converting enzyme (ACE) inhibitor (lisinopril), biguanide (metformin), and a loop diuretic (furosemide). Physical examination reveals bilateral pitting edema up to his knees, swelling in the face, peripheral pulses 2+ in all extremities, and an unsteadiness during Romberg testing with both eyes open and closed. Auscultation reveals bilateral crackles at the lung bases, and the rest of his physical examination is non-contributory. DX?

hyponatremia

A 14-year-old boy presents due to embarrassment after an incident in school: while undressing before the class in physical education, the other boys laughed at him because of his "underdevelopment." His personal and family history is non-contributory. What is a definitive sign of the onset of puberty in this patient?

increase in size of testis

An 18-year-old pregnant woman presents with nocturia and increased urinary frequency. She is at 18 weeks gestation and she has no past medical history. She has no suprapubic fullness; her fundal height is appropriate for her weeks of gestation. Diagnostic findings are as below: UrineNo cells, no blood, no protein; trace glucose specific gravity 1.020 Fingerstick glucose85 mg/dL Renal ultrasound Bilateral mild hydronephrosis, no noted ureteral abnormalities; bladder not distended post-void Serum creatinine0.5 mg/dL What is the most likely explanation for her urinary frequency and nocturia?

increased GFR

A 24-year-old woman presents with severe diarrhea that she has been experiencing for 3 days, with no medical issues before then. She now feels dizzy upon standing, her tongue is dry, and her eyes appear glazed. Her serum sodium concentration is 130 mEq/L. What finding is most likely?

increased serum ADH

A previously healthy 35-year-old man has had right flank pain radiating to his right groin for the past 3 hours. CT shows a 11 mm uric acid stone in his right proximal ureter. best treatment option?

lithotripsy (stone is > 10 cm so likely won't pass on its own)

A 51-year-old woman who has been in a road traffic accident presents with multiple injuries. She has a urine output of 350 mL over the previous 24 hours. Her serum creatinine level has increased from 4.2 mg/dL to 4.8 mg/dL over the same period. What class of drug could be used to convert this patient's oliguric renal failure to the non-oliguric type and to facilitate further management?

loop diuretic

A 12-year-old boy presents with a 3-hour history of extreme severe pain in the right testis. It started suddenly, is 8/10 in intensity, and does not radiate. It is associated with nausea and scrotal swelling. He never had such pain in his lifetime, and he denies any problem in urination. He has never been operated on, and he denies any history of trauma. He is allergic to penicillin. On physical exam, the child is in visible distress. Temperature is 37°C, heart rate is 95, blood pressure is 120/70 mm Hg, and respiratory rate is 20 per minute. Genital examination reveals enlargement and edema of the entire scrotum. The right testicle is erythematous and tender to palpation; it appears to sit higher and lies horizontally in the scrotal sac relative to the left side. The cremasteric reflex is absent ipsilaterally, and there is no relief of pain upon elevation of the scrotum (Prehn's sign). Abdomen is non-tender and tympanic to percussion in all four quadrants. Bowel sounds are audible. Chest auscultation shows normal vesicular breathing with mild crepitations over the lower lung fields. Cardiac exam reveals normal S1 and S2, without rubs, murmurs, or gallop. His initial labs show a hemoglobin of 14.5 g/dL, WBC of 13,000/mm3, platelets of 210,000/mm3, sodium of 140 mmol/dL, potassium of 3.8 mmol/dL, chloride of 95 mmol/dL, urea of 25 mg/dL, and creatinine of 0.9 mg/dL. What clinical feature helps most to differentiate the patient's condition from other causes of scrotal pain?

loss of cremesteric reflex

An otherwise healthy 50-year-old man is brought to your office, located in a small town in the foothills of the Appalachian mountains, by his daughter. The daughter reports that her father has been more lethargic than usual, and she states that she has seen him go into the backwoods and return obviously inebriated. You suspect that he is drinking moonshine from a homemade still and that it is leaching poisonous metals/substances into the distillate. The patient is morose and has difficulty answering questions. The rest of the physical exam uncovers no other abnormalities. The blood gas analysis shows a pH of 7.3, a pCO2 of 30 mm Hg, and a bicarbonate of 15 mEq/L. Further laboratory testing revealed sodium of 140 mEq/L and a chloride of 100 mEq/L. What is the most likely diagnosis?

metabolic acidosis

A 58-year-old woman presents with pelvic pressure and a bulging sensation in her vaginal region. During her pelvic examination, the Valsalva maneuver results in prolapse of the uterus. What risk factor is most closely related to the patient's diagnosis?

multiparous

A 55-year-old woman with no significant medical problems presents with a CC of pink urine, stating, "I think I have blood in my urine." She states she has no pain with urination, but the hematuria is persistent. On questioning, she states that she has had a 1-month history of some progressively worsening left flank pain. The pains are not debilitating, but they are nagging. She admits to a 50 pack-year smoking history, and she states she is currently retired from her job as a teacher. Vital signs: T 98.6°F, BP 118/76 mm Hg, P 78/min, R 14/min. Abdominal exam reveals a left side abdominal mass. Urine dipstick only shows too numerous to count RBCs, and urine cultures are negative. CT scan of the abdomen and pelvis with and without contrast reveals a 4.2 cm solid enhancing lesion in the left renal parenchyma. What is the most effective treatment for this patient?

nephrectomy (renal cell carcinoma)

A 13-year-old boy presents for an annual examination; he reports no problems with his health. Upon physical examination, his body temperature is 98.3°F, his blood pressure is 150/100 mm Hg, and he shows a slight periorbital puffiness. He explains that recently he has been staying up late to read and has noticed "puffy eyes" in the morning. Urinalysis indicates light brown urine, low-level proteinuria, and no bacteriuria. What is the most likely diagnosis?

nephritic syndrome

A 12-year-old African American boy presents with a 1-month history of poor appetite and has complained of overall not feeling well. When questioned, the boy cannot delineate any specific symptoms except that he feels "puffy." He denies pain, eating disorder, rash, depression, drug use, and fevers. The family denies recent travel. His PMH is unremarkable with no recent or chronic illnesses. He has had no surgeries, and he takes no medications. His family history includes grandparents on both sides with hypertension, with one of these grandparents also having died from some type of kidney problem. His ROS is entirely negative except for the above symptoms and some noted change in urine, which he describes as frothy. He denies dysuria, gross hematuria, polyuria, and nocturia in the ROS.On physical exam, his vitals are: Tem: 97.9°F; Resp 14; HR 90; BP 120/74 mm Hg right arm sitting. It is noted weight is up 2 pounds from his charted weight 3 months ago. HEENT, neck/thyroid, lungs, cardiac, abdominal, musculoskeletal, neurological and derm exams are unremarkable. Examination of extremities reveals bilateral 1-2+ edema in the upper and lower extremities, with 2+ pulses. Given the proteinuria, a urine protein-to-creatinine ratio is obtained, showing 3.6 mg/mg. What is the most likely diagnosis?

nephrotic syndrome

The effect of steroid therapy is evaluated in an 8-year-old Caucasian boy being treated for fatigue and generalized edema following a "bad cold." His vitals are T 37°C, pulse 90/min, RR 20/min, and BP 110/70. Physical exam reveals the presence of mild periorbital edema and marked peripheral edema in hands and feet with the remainder of the exam within normal limits. Lab values include Dipstick urine protein 3+; urine protein 50 mg/m2/hr (<40 mg/m2/hr) Specific gravity 1030 (1008-1020) Urine protein/creatinine ratio 2.0/mg creatinine (<0.2/mg creatinine) Serum albumin 3.9 (5.9-8.0 mg/dL) Cholesterol 250 (112-247 mg/dL) Remainder of laboratory values including BUN and plasma creatinine are within normal limits. What is the most likely diagnosis?

nephrotic syndrome

A 40-year-old woman presents to the emergency department with unremitting left flank pain. She denies dysuria or fever. She notes that her urine output has decreased over the last few days. As you observe the patient, she is writhing on the gurney and unable to find a comfortable position. On further inquiry, the patient states that she has been trying to lose weight by increasing protein in her diet, exercising, and decreasing her normal fluid intake. On urinalysis, red blood cells are noted. All other labs are within normal range. diagnosis?

neprholithiasis (flank pain and hematuria)

A 36-year-old man presents with severe left flank pain that started at 4 AM. After pacing around his house for several hours and taking ibuprofen and acetaminophen (Tylenol), he comes to the office at 7:30 AM asking to be seen. He describes the flank pain as burning and steady, with some radiation to the left lower abdomen. He has had some nausea, and he unsuccessfully tried to vomit. He has not had any diarrhea or urinary symptoms. He describes his past medical history as negative for significant illnesses or injuries. Physical exam reveals normal vital signs except for tachycardia, a negative heart and lung exam, and a negative abdominal exam. There is no tenderness noted in the CVA regions. Which test is recommended to most accurately confirm the diagnosis?

non-contrast CT

A 50-year-old Caucasian man presents to establish care. He has no history of diabetes, hypertension, cardiovascular disease, cerebrovascular disease, peripheral vascular disease, nephrolithiasis, or obstructive uropathy. His only problem on review of systems is musculoskeletal aches. His parents died in their 80s of unknown causes. He has never smoked, taken recreational drugs, had a transfusion, or had unprotected sex. He worked for years as a handyman. His only medication is ibuprofen, which he has taken at several times per week for the past 10 years. His exam is normal. You order some basic diagnostic tests, including a basic serum chemistry panel and a urinalysis. The urinalysis shows microscopic hematuria and a few white cells but no leukocyte esterase or nitrites. His serum creatinine is 1.5 mg/dL. His other tests are normal. What additional test should you order and why?

non-contrast CT abdomen or renal US (analgesic nephropathy)

A 35-year-old woman just found out she is pregnant. She is experiencing polyuria, but she denies dysuria and incontinence. Her urinalysis is unremarkable. Her fetal ultrasound is normal, and her renal ultrasound shows normal physiological hydronephrosis of pregnancy. Her pre-pregnancy weight was 155 lbs, and she is 5 feet tall. Her calculated body mass index (BMI) is 30.3 kg/m2. She takes no medications. She smokes half a pack of cigarettes per day. In this patient how can you best prevent urinary stress incontinence postpartum?

normalize weight

A 55-year-old man presents with painless gross hematuria since last evening. Urine cytology and cystoscopy reveal transitional cell carcinoma of the bladder. He has had diabetes and hypertension treated with metformin and atenolol, respectively, for the past 10 years. He has never smoked but consumes alcohol: about 2 pints of beer every day. He worked as a car mechanic at a garage for 30 years. He recently visited several African countries; upon returning, he had diarrhea and was diagnosed and treated for amebiasis. What in his history predisposed him to bladder cancer?

occupation (exposure to diesel and petroleum)

Upon inspection of an 18-year-old man's scrotum, you note that the left side is underdeveloped, and a testis is not palpable. There is no scrotal tenderness, swelling, or nodularity. What is the patient's condition most likely due to?

orrect answer: Cryptorchidism Explanation An underdeveloped scrotum, either unilateral or bilateral, is likely due to undescended testicles, which is also known cryptorchidism. Acute epididymitis and strangulated inguinal hernia are usually tender to the touch and cause swelling. Hydroceles are commonly accompanied by scrotal swelling. If painless nodules are palpated in the testes, testicular cancer should be ruled out, especially in men between 15-35.

A 68-year-old man presents with scrotal swelling; he has had the swelling for the past few months. It is not bothering him, but his wife wants him examined. His history is not significant for any other GU history or symptoms. A genitourinary examination reveals a right hydrocele. Question What would the examination findings describe?

orrect answer: Fluid collection in the scrotum that transilluminates Explanation A hydrocele is a collection of fluid between layers of the tunica vaginalis that surrounds the testicle. The typical presentation of a hydrocele is a painless scrotal swelling that can worsen throughout the day. When a flashlight or penlight is held behind the scrotum in a dark room, the light will transmit through the fluid; this is known as transillumination. A hydrocele is not usually bothersome; patients will usually only take issue if it is large enough to get in the way. Hydroceles can be treated surgically, but this is usually reserved for the most bothersome cases. A solid lesion in the scrotum that does not transilluminate describes how a testicular mass may present on physical examination. A hydrocele is not solid and does transilluminate. While a hydrocele is a fluid collection in the scrotum, it usually does transilluminate. There are no genitourinary findings that would involve a fluid collection in the scrotum that would not transilluminate. Dilated veins in the scrotum that transilluminate typically represent a varicocele rather than a hydrocele. The dilated veins are often described as feeling like a "sack of worms" on physical examination.

A 36-year-old man presents with severe left flank pain that started at 4 AM. After pacing around his house for several hours and taking ibuprofen and acetaminophen (Tylenol), he comes to the office at 7:30 AM asking to be seen. He describes the flank pain as burning and steady, with some radiation to the left lower abdomen. He has had some nausea, and he unsuccessfully tried to vomit. He has not had any diarrhea or urinary symptoms. He describes his past medical history as negative for significant illnesses or injuries. Physical exam reveals normal vital signs except for tachycardia, a negative heart and lung exam, and a negative abdominal exam. There is no tenderness noted in the CVA regions. Question What test should you suggest in order to most accurately confirm the diagnosis?

orrect answer: Non-contrast helical CT Explanation The correct response is a non-contrast helical CT. This patient presents with a classic presentation of renal colic. His presentation is suggestive of a stone lodged in the proximal ureter. Pain from a stone lodged in the kidney gives rise to vague flank pain that, if lodged in the middle section of the ureter, gives rise to more anterior abdominal pain in addition to the flank pain. A stone lodged in the distal ureter gives rise to symptoms of urinary frequency, as well as a possible radiation of the pain to the penis in men (labia and vulva in women). Similar presentations of pain may be due to diverticulitis, colitis, hernias, and arterial aneurysms. In this patient, the negative physical exam, his young age, and the fact that the pain is on the left (effectively ruling out gallbladder-related diseases) all point out to a ureteral stone as the cause of his severe flank pain. The prevalence of urolithiasis in the general population is 2-3%. 50% of patients who have had previous urinary calculi will have a recurrence within 10 years. Imaging modalities have different sensitivities for the diagnosis of a ureteral stone:

A 66-year-old man presents to the office with polyuria and erectile dysfunction. He denies any other symptoms or significant past medical history. Physical examination reveals Tanner stage 5 of the external genitalia, balanitis of an uncircumcised penis, and slightly enlarged, symmetrical and smooth prostate. His condition is most likely the result of:

orrect answer: Diabetes mellitus Explanation The correct answer is diabetes mellitus since the presence of polyuria would indicate hyperglycemia and the associated erectile dysfunction and/or balanitis may be the only other presenting symptom or sign of diabetes mellitus in a male patient. Erectile dysfunction is a common vascular and neurological complication of diabetes and occurs in up to 75% of male diabetics. Elevated blood sugars result in autonomic neuropathy of the cavernous nerve of the penis so that erectile dysfunction serves as one of the earliest indications of neuropathy. Likewise, hyperglycemia results in microvascular damage to the dorsal and cavernous arteries, in the same way retinopathy, nephropathy, and neuropathy develop, further contributing to poor perfusion and erectile dysfunction. Hyperglycemia also results in the colonization of skin organisms, commonly Candida, resulting in typical superficial yeast infections seen in diabetics such as balanitis in men and vulvovaginitis in women. Benign prostatic hypertrophy (BPH) typically occurs in the periurethral zone of the prostate and usually presents with lower urinary symptoms (LUTS) that suggest obstruction (i.e. hesitancy, weak stream, straining, post-void leaking) or irritation (i.e. nocturia, frequency, urgency). Digital rectal examination of prostatic hyperplasia typically reveals a smooth, firm enlargement of the gland which may be asymmetrical or indurated. Early BPH is not typically associated with erectile dysfunction or Candidaskin infections. Prostate cancer most often develops in the peripheral zone of the prostate and is usually asymptomatic. Locally advanced prostate cancer may encroach on the central transition zone of the prostate and present with irritative urinary symptoms. Prostate cancer that extends outside the prostate capsule may result in erectile dysfunction. Carcinomas in the peripheral zone are often palpable and typically a hard, irregular nodule or induration. Prostate cancer is not typically associated with Candidaskin infections. Hypogonadism may present with fatigue, decreased libido, diminished erections, gynecomastia, or decreased testicular size, muscle mass, or hair growth associated with secondary sexual characteristics. It is typically not associated with an enlargement of the prostate, urinary complaints, or Candidaskin infections. The characteristic presentation of diabetes insipidus (DI) is abnormally large amounts of dilute urine - insipidus means tasteless. Polyuria is massive, often associated with nocturia and enuresis, and results in dehydration, which is often not evident due to a compensatory increase in thirst and polydipsia. DI is the result of the posterior pituitary's failure to secrete antidiuretic hormone (ADH) resulting in central diabetes insipidus (DI) or the kidney's resistance to ADH resulting in nephrogenic DI. DI is not typically associated with Candidaskin infections. References:

A 39-year-old woman presents due to severe injuries to her brain, spine, and pelvis sustained 6 months ago in a rollover car accident. Since then she has been incontinent. She says that she dribbles urine without being aware of it throughout the day and night. She feels no sense of urgency or sudden desire to avoid. She also complains of decreased sensation around her anus. Urinary stream is normal. Evaluation of her bladder shows the following findings: normal anatomy, large flaccid bladder with a normal wall, low pressures, and high residual volume. Question What is the most likely location of her injury?

pelvic nerves

Case A 54-year-old man presents with a lump in his scrotum recently. After answering many questions about possible symptoms and undergoing a thorough genitourinary examination, the patient is told that he most likely has a hydrocele. Question What findings most closely support this diagnosis?

orrect answer: Non-tender, fluid-filled lesion that transilluminates Explanation The correct answer is a non-tender, fluid-filled lesion that transilluminates. A hydrocele is a collection of fluid within the tunica vaginalis. It is non-tender, usually develops slowly over time, and will transilluminate when a light is held up to the scrotal wall. Patients can experience fluctuating size of the hydrocele, swelling of the scrotum or inguinal canal, heavy sensation within the scrotum, and do not typically experience any pain. Solid mass within the testicle that does not transilluminate is not the correct answer. Solid masses that are actually in the testicle itself are malignancies until proven otherwise. Patients may have a reactive hydrocele in addition to the malignancy, but hydroceles do not actually present as solid masses. In addition, hydroceles will transilluminate, whereas testicular malignancies will not transilluminate on examination. Painful swollen retracted testis that does not transilluminate is not the correct answer. This more closely describes testicular torsion as opposed to a hydrocele. Testicular torsion is painful, whereas hydroceles are not painful. Testicular torsion can also cause one testicle to retract and will not transilluminate upon examination. Testicular torsion is a urologic emergency, whereas hydroceles are often not even treated. Non-tender mass with the consistency of a "bag of worms" without transillumination is not the correct answer, as this description is more closely associated with a varicocele. Both hydroceles and varicoceles are non-tender, but hydroceles will transilluminate and varicoceles will not. Since a varicocele is a venous varicosity without the spermatic vein, it is often described as having the consistency of a "bag of worms" on examination. It will also decrease in size if the scrotum is elevated or the patient lies supine. Painless cystic mass containing sperm that transilluminates is not the correct answer, as this more closely describes a spermatocele. A spermatocele is usually a palpable cystic mass that is free-floating above the testicle and will transilluminate upon examination. Spermatoceles and hydroceles are similar in that they are painless, harmless, and usually do not require treatment.

A 42-year-old woman presents for an evaluation of back pain. She states that she has a history of recurrent UTIs; she notes that on occasion her urine appears red. She states her mother and brother both have "kidney problems." Upon physical examination, she demonstrates CVA tenderness. Her vital signs include blood pressure of 145/90 mm Hg, respiratory rate of 16 breaths/min, and heart rate of 72 bpm. Urinalysis shows leukocyte esterase 1+, protein trace, and blood 3+. The microscopic examination reveals 5-10 WBCs/HPF and 20-30 RBCs/HPF. An abdominal ultrasound demonstrates multiple bilateral fluid collections within the kidneys. What is the underlying renal pathology?

polycystic kidney disease

A 6-year-old boy presents with a fever, malaise, and dark urine. His mother states that he missed school earlier this month with a sore throat. On examination, his blood pressure is 120/88 mm Hg, pulse is 82/min, temperature is 100.6°F, and respirations are 16/minute. On physical assessment, the patient appears ill with only mild costovertebral angle tenderness noted. Urinalysis reveals the following: dx?

post strep glomerulonephritis

A 7-year-old girl presents with a 1-day history of bloody urine. The grossly bloody urine scared both the girl and her parents, but she denies dysuria and frequency. No trauma or sexual abuse has occurred. The parents deny recent fever in the patient, but they note that she had a fever for a few days accompanying a sore throat. She was given acetaminophen at an appropriate dosage for her weight, and about 3 days of some leftover amoxicillin; both the fever and pharyngitis then resolved. Her past medical history is unremarkable for any chronic illnesses. Her only medication is a multivitamin, and she has NKDA. She has had no surgeries, and family history is unremarkable for urinary tract disorders or any bleeding disorders.On physical exam, she appears interactive and in no apparent distress. She is well-nourished, non-obese, and perhaps mildly edematous. Vitals are a temperature of 99.0°F and a BP of 138/85 mm Hg; pulse is 98, and respiratory rate is 20. No rashes are found. Cardiac exam reveals normal rate and rhythm; there are no murmurs or rubs. On abdominal exam, her abdomen is non-distended, non-tender, and without masses or hepatosplenomegaly. She has no CVA tenderness. A urinalysis is performed; the significant findings are as follows: Protein 2+ Glucose Negative KetonesNegative Blood 4+ Nitrites Negative Leukocyte esterase1+ Microscopic analysis is performed, confirming the presence of red blood cells (RBCs) that are dysmorphic and indicate RBC casts.Blood tests are ordered, revealing a complete blood count within normal limits and a complete metabolic panel with elevated creatinine. The patient's antistreptolysin O level is elevated, and her serum complement level is low. Dx?

post-infectious glomerulonephritis

A 71-year-old male farmer presents because he is experiencing a gradual increase of frequency of urination over the previous 4-5 months. He has difficulties starting to urinate, the stream is slow, and he frequently has a sensation of incomplete emptying. He has not had a physical examination in several years. He has no history of sexually transmitted diseases (STDs) or urinary tract infections. Upon examination, his abdomen and his prostate are non-tender. His prostate seems moderately enlarged (estimated at 35 g) but smooth and symmetrical. What diagnostic test do you recommend as the next step in this patient's management?

post-void residual urine ( enlarged prostate)

A 56-year-old African American man with a history of hypercholesterolemia and obesity presents with a 6-week history of reduced urinary stream. Upon further questioning, he also admits to generalized weakness, loss of appetite, and a dull lower backache during this time, which he attributes to being "out of shape." He denies fever, chills, chest pain, shortness of breath, abdominal pain, hematuria, frequency, hesitancy, flank pain, dysuria, abnormal penile discharge, a history of trauma, alcohol use, or smoking. He states that he had an extramarital affair about 2 months ago and did not use any barrier methods during sexual intercourse. The physical exam is notable for a man who is in no acute distress, and he has normal vital signs. He is found to have a distended bladder on abdominal exam; there is vertebral tenderness of the lumbosacral spine as well as a firm non-tender nodularity of prostate upon digital rectal exam. What is the most likely diagnosis?

prostate cancer

A 54-year-old man is admitted to the intensive care unit and intubated on a ventilator after a serious motorcycle accident where he sustained multiple fractures, spinal cord trauma, and a liver laceration. On his fifth week of hospital admission, blood cultures from a central line reveal staphylococcal septicemia. After another month in the intensive care unit, the patient is found to have ascites and pitting edema. Vitals are within normal limits. Ultrasound demonstrates a fatty liver and unremarkable gallbladder. Echocardiogram shows no structural defects and a normal ejection fraction. What is the most likely diagnosis?

protein deficiency

Your patient is a 55-year-old African American man with bilateral foot edema that started 1 week ago and has progressively worsened. He has a 5-year history of type 2 diabetes, but his last physical exam was 2 years ago. He does not take any medications because he prefers to "control the health with a healthy lifestyle." On examination, you find pale male patient with bilateral ankle edema. His temperature is 37°C, blood pressure 155/100 mm Hg, heart rate 80, and respirations 15/min. He is otherwise well-developed and well-nourished; BMI is 25 mg/m2. In addition to starting glycemic and blood pressure control, what dietary advice would you give this patient to help slow the progression of his new diagnosis?

protein restriction (CKD)

A 19-year-old woman presents with a 4-day history of fever accompanied by chills, nausea, vomiting, and back pain. She denies any chest pain, cough, or urinary symptoms. On further questioning, she said she had 2 episodes of diarrhea yesterday with vague abdominal discomfort. She has no other significant past medical history, is on no medications, and has no allergies. Family history is significant for hypertension in father and arthritis in mother. She is single, has no children, does not smoke or drink, and works part time as a waitress. On exam, she has a temperature of 102.4°F, pulse of 110/min, BP 110/60 mm Hg, and SPO2 of 92%. Mucous membranes are dry, and sclera is clear. Lungs are clear, and heart sounds are normal. There is mild left flank tenderness and tenderness in the left costovertebral angle. Labs: Hb 12g/dL, WBC 17,000/uL, bands were 18% and platelets 350,000/uL. Chest X-ray and EKG are normal. Urinalysis shows 35 WBC, 6 RBC, and no casts. What is the most likely diagnosis?

pyelonephritis

A 67-year-old man presents with severe flank pain, fever, postural dizziness, dysuria, inability to pass urine for 1 day, and mild confusion. He has experienced two episodes of urine retention and incontinence over the previous 2 months, the most recent of which required urethral catheterization and outpatient antibiotic treatment for E. coli. His past medical history is significant for hypertension, prostatic hypertrophy, non-insulin dependent diabetes mellitus, and cognitive impairment that requires minimal assistance in activities of daily living. His temperature is 102.02°F (38.9°C); BP 80/50 mm Hg; pulse rate 114/minute; respiratory rate 43 breaths/minute. Physical examination reveals a tender distended bladder that drained 2700 mL of turbid urine. What will be the next diagnostic step in this condition?

quantitate urine culture (Sepsis)

A 74-year-old multiparous woman with a history of breast cancer has been referred to a gynecologist due to a sensation of vaginal fullness and pressure, sacral back pain with standing, coital difficulty, lower abdominal discomfort, and urinary frequency and incontinence. She states that she feels "a bulge" in the lower frontal vaginal area. She denies any fever, chills, flank pain, history of infectious diseases, changes in weight, dysuria, or hematuria. The physical exam is remarkable for an obese body habitus and non-tender bulge located anterior within the vaginal introitus that becomes more pronounced upon Valsalva and standing. What is the most appropriate next step in the management of this patient?

recommend pelvic muscle exercises

A 62-year-old man is hospitalized because of a 1-week history of extreme malaise and painful skin changes. It started as "just a rash" but developed into clusters of clear vesicles; new vesicles appear while old ones dry and crust over. His past medical history is significant for type 2 diabetes mellitus, for which he is on a controlled diet. You find the rash distributed over his body and you make a diagnosis of disseminated herpes zoster. Acyclovir IV is introduced together with the supportive therapy, but after 2 days, the patient starts experiencing nausea and swelling; his urine output dramatically decreased and urinalysis revealed gross hematuria, with other laboratory studies showing elevated BUN and creatinine. What is the most probable mechanism of renal failure in this patient?

renal tubule obstruction (Acyclovir)

A 32-year-old man reports anorexia, constipation, fatigue, thirst, weakness, drowsiness, nausea, and muscle pain that has developed over the last few days. He has been bedridden for the last 3 months after a traffic accident wherein his 3 cervical vertebrae were fractured. A few weeks ago, a diagnosis of kidney stones and chronic renal insufficiency was established. The rest of his personal and family history is non-contributory. His physical examination today demonstrates quadriplegia, and the rest of examination is within normal limits. While waiting for the laboratory results, you order an EKG. What do you expect it to reveal?

shorted QT interval (hypercalcemia)

A 65-year-old man presents with flank pain, blood in his urine, and an unexplained weight loss. His past medical history is significant for numerous infections, kidney stones, cigarette use, and alcohol use. On physical exam, there is a palpable abdominal mass, as well as a low-grade fever. What puts this patient at the greatest risk for the development of the most likely diagnosis?

smoking (renal cell carcinoma)

A 44-year-old woman G5P5 presents for her annual pelvic examination. Her menses are regular, and she is currently mid-cycle. She notes "leaking urine" when she coughs, sneezes, or strains. There is a bulge into the anterior vaginal wall; it is exacerbated when the patient is asked to "bear down." A urinalysis is unremarkable. There is no cervical motion tenderness or discharge noted on pelvic examination. The patient is afebrile and in no distress. No lesions are noted on the external genitalia and the pelvic examination is unremarkable except for the noted bulge. Vaginal cultures for gonococcus (GC) and chlamydia are pending. What is the most likely diagnosis?

stress incontinence

A 29-year-old Caucasian man with a prior history significant for right cryptorchidism presents for a routine physical examination. His prior cryptorchidism was corrected by orchiopexy at the age of 6 months. He has no complaints at present. His physical exam reveals no abnormalities other than bilateral gynecomastia and a right testicular mass that is painless and firm; it measures approximately 1.5 cm in diameter. The right testicular mass does not transilluminate, nor does it disappear when the patient lies supine. There is no femoral or inguinal lymphadenopathy, nor are there palpable hernias. What is the most likely diagnosis?

testicular malignancy

A 15-year-old boy is seen in your office at 11:30 AM due to left scrotal pain and swelling; it started at 7 AM when he woke up. He recalls no trauma. When questioned, he says that he has never had intercourse. He has been feeling nauseated, and he vomited once. Physical examination demonstrates a well nourished well developed boy, appearing moderately uncomfortable. Vital signs are normal except for an oral temperature of 37.9°C. Pain assessment score (Wong-Baker scale) is 6/10. He is Tanner Stage III puberty. The left testicle is approximately 1.5 times the size of the right testicle and high-riding compared to the right. The skin is diffusely erythematous. It is difficult to palpate the scrotum due to tenderness. Cremasteric reflex is absent. Penis is circumcised and appears normal. Scrotal ultrasonography with Doppler ultrasound demonstrates decreased blood flow to the testis. The remaining physical examination is normal. What is the most likely diagnosis?

testicular torsion

A 25-year-old Caucasian man presents to the local emergency department due to severe testicular pain. The pain began abruptly about 2 hours ago and has gotten progressively worse. He is numerically currently rating it as a 9/10. This is only affecting the left testicle. He has never had an episode like this before. He also states he has become nauseated during the time you are in with him. Which of the following is the most likely diagnosis, categorizing this patient as a surgical emergency?

testicular torsion

A 69-year-old woman presents with a 3-month history of intermittent urinary incontinence. After further questioning, she reveals that she experiences leakage after having an intense need to void. This is an example of what type of incontinence?

urge

A 6-year-old boy presents with his mother, who is concerned because he wets his bed 2-3 times a week at night. He has no urinary symptoms during the day. He is the second child of three. He is in the first grade and is struggling with performance. He has had no medical problems; there is no history of developmental delay, and he was the product of a normal uncomplicated pregnancy and delivery. Since the birth of the third child, his behavior has been poor. The vital signs are normal, and examination of other body systems is unremarkable. What investigation is most important in this child's workup?

urinalysis (to r/o infection renal disease )

A male newborn is noted to have hypospadias. A complete evaluation determines that the child has no other genitourinary anomalies. For the prevention of what possible sequelae will hypospadias repair be performed?

urinary tract infection

A previously healthy 8-year-old African American boy presented with a 3-day history of worsening fatigue and generalized edema. You saw him about a week ago when he had symptoms of a cold, for which you advised only supportive therapy. He appears alert and cooperative. His vitals are normal (temperature 37°C, pulse 90/min, respiratory rate 20/min, and blood pressure 100/70 mm Hg). Physical examination reveals the presence of generalized edema, and the rest of the examination is within normal limits. What parameter would be most important to follow and evaluate the effect of therapy in order to prevent chronic kidney disease?

urine protein

A 62-year-old woman with diabetes and hypertension is evaluated for incontinence. She is found to have a cystocele and grade 3 uterine prolapse. Her diabetes and hypertension are both poorly controlled. On examination, the uterus can be repositioned with firm digital pressure. In addition to being declared unfit for surgery, she is not keen on undergoing any procedure. She has no history of any urinary infections or pelvic inflammatory disease. What is the next step in management?

vaginal pessary

A 64-year-old woman presents with urine leakage that has been progressively worsening for the past 6-12 months. She also admits to having vaginal dryness, poor lubrication with intercourse, and subsequent dyspareunia. Urinary leakage primarily seems to occur with coughing, sneezing, or high-impact exercise. She denies dysuria, hematuria, urgency, and vasomotor symptoms. She tries to empty her bladder every 2-3 hours and has cut out all alcoholic and caffeinated beverages, but symptoms persist. The patient is a G6P6. She has no chronic medical conditions and no past surgeries. She wants to do everything she can to avoid surgery. She is in a monogamous relationship. She was last seen 1 year ago with a normal well-woman exam and normal Pap smear. On physical exam, vitals are normal. The pelvic exam reveals dry pale vaginal mucosa with a bulge on the anterior vaginal wall that increases in size with a Valsalva maneuver. Urinalysis is normal. Given this patient's strong preference to avoid surgery, what else could be done to improve her urinary symptoms?

vaginal pessary

A 4-year-old uncircumcised boy presents with a 2-day history of penile pain. The patient is afebrile and vital signs are stable. On genital examination, his foreskin is retracted proximally and the glans is edematous and cold. You are unable to reduce the proximal foreskin distally over the glans penis; it is strongly suspected that arterial flow is compromised. The only urologist available will arrive in 1.5 hours. What is the most appropriate treatment for this patient?

vertical incision (parpaphimosis)

A 70-year-old woman presents to the office due to frequent urination for the past year. The patient states that she must use the bathroom 6 times a day. She explains that even after she voids, she does not feel as though her bladder is empty. She denies any incontinence or leakage. She reports drinking plenty of fluid so she does not get dehydrated. The patient has a history of dementia and applies an estrogen cream for vaginal atrophy. A urinalysis is negative for a urinary tract infection. What lifestyle modification can improve this patient's symptoms?

void at scheduled intervals (overactive bladder)

A 33-year-old woman presents with a 1-day history of burning sensation during urination and foul-smelling urine. She is sexually active with one partner and uses a diaphragm for contraception. Fluid intake and blood glucose are normal. Leukocyte esterase dipstick test is positive, and urinalysis reveals 10 WBC/mL and bacteria. Further investigations confirm urinary tract infection (UTI), and you treat it appropriately. During a follow-up visit, she tells you that this is the third episode of UTI in the past few months and asks you what she can do to reduce the risk in future. What step can she take to reduce the risk of infection?

void frequently

A 71-year-old male farmer presents because he is experiencing a gradual increase of frequency of urination over the previous 4-5 months. He has difficulties starting to urinate, the stream is slow, and he frequently has a sensation of incomplete emptying. He has not had a physical examination in several years. He has no history of sexually transmitted diseases (STDs) or urinary tract infections. Upon examination, his abdomen and his prostate are non-tender. His prostate seems moderately enlarged (estimated at 35g), but smooth and symmetrical. Question What diagnostic test do you recommend as the next step in this patient's management?

Correct answer: A prostate-specific antigen test Explanation A prostate-specific antigen (PSA) test is correct. The patient presents symptoms of enlarged prostate, a common occurrence in a man of his age. His symptoms started several months before the visit, and they clearly bothered and irritated him. He does not present with symptoms of infection. The level of PSA, a marker of prostate cancer, is known to correlate well with the prostate's volume. Although most localized prostate cancers are discovered through PSA screening, active surveillance rather than immediate surgery, radiotherapy, or hormone treatment is now recommended. The answer a serum creatinine test is incorrect.A creatinine test determines the level of creatinine, a substance that is normally eliminated by the kidneys. The amount of creatinine in the blood is an indicator of how well the kidneys work. This patient presents clear symptoms of an enlarged prostate, so this test, even though one of the basic check-up tests, is not a priority for good diagnosis in the case of this patient. The answer a post-void residual urine test is not correct. This test measures the amount of urine left in the bladder after urination and can help evaluate an enlarged prostate; however, the PSA test is preferred, as it can give additional information on the health status of the patient. The patient is an elderly man, and the PSA test also provides a way to detect the presence of prostate cancer, a disease more frequently found in the elderly. The answer a urine culture test is incorrect. The patient does not have symptoms of a urinary tract infection. He has no fever and does not experience sensations of burning or pain while urinating. The answer a blood urea nitrogen test (BUN test)is incorrect. This test measures how much urea nitrogen is in the blood, and it is frequently performed with other tests to monitor or diagnose kidney dysfunction. This patient does not show any symptoms associated with kidney disease, but shows signs of an enlarged prostate problem.

Case The effect of steroid therapy is evaluated in an 8-year-old Caucasian boy being treated for fatigue and generalized edema following a "bad cold". His vitals are T 37°C, pulse 90/min, RR 20/min, and BP 110/70. Physical exam reveals the presence of mild periorbital edema and marked peripheral edema in hands and feet with the remainder of the exam within normal limits. Lab values include dipstick urine protein 3+; urine protein 50 mg/m2/hr (<40 mg/m2/hr) Specific gravity 1030 (1008-1020) Urine protein/creatinine ratio 2.0/mg creatinine (< 0.2/mg creatinine) Serum albumin 3.9 (5.9-8.0 mg/dL) Cholesterol 250 (112-247 mg/dL) Remainder of laboratory values including BUN and plasma creatinine are within normal limits. Question What additional treatment should be initiated in this patient to decrease the risk of chronic kidney disease?

Correct answer: ACE inhibitor Explanation The pediatric patient described is apparently suffering from nephrotic syndrome. Glomerular disease induced proteinuria is the most common cause of nephrotic syndrome in children due to damage to the glomerular filtration barrier resulting in leakage of plasma proteins into the glomerular ultrafiltrate. Signs and symptoms in children include edema, urine protein: creatinine ratio > 0.2/mg creatinine; heavy proteinuria (urine protein >40 mg/m2/hr), hypoalbuminemia, and hyperlipidemia. The nephrotic range of proteinuria in children is higher than in adults (> 40 mg/m2/hr). Angiotensin-converting enzyme inhibitors and angiotensin receptor blockers not only lower blood pressure but have that additional benefit of slowing the progression of kidney disease even in patients with normal blood pressure. Although this child is normotensive and is already receiving steroid treatment, the next best additional treatment, therefore, is an ACE inhibitor or an angiotensin receptor blocker (ARB) to decrease the proteinuria and GFR decline in order to reduce the risk of chronic kidney disease. Diuretic is incorrect. Although a diuretic would provide symptomatic relief for the edema, it would not prevent the development of chronic kidney disease in this patient. Spironolactone is incorrect. Spironolactone is an inhibitor of renal aldosterone effects such as sodium and water retention and would help combat the edema in this patient but would not decrease the risk of chronic kidney disease. Beta blocker is incorrect. Beta blockers can be useful in the treatment of hypertension but this patient has not yet developed hypertension. Mixed alpha and beta antagonist is incorrect. The use of a mixed alpha and beta antagonist drug is not indicated in this normotensive patient and would likely not reduce the risk of chronic kidney disease.

Case A 25-year-old man has malaise and decreasing urine output 2 weeks after completing a course of tetracycline for acne. He takes no other medications and has no other medical problems. He is afebrile and has a normal physical exam. Urinalysis: no casts, no red cells, many white cells, and no bacterial growth. Peripheral blood white cell count: 7x103/cmm. Blood eosinophil count: elevated. Serum creatinine: 1.5 mg/dl. Serum potassium: 4.5 meq/l. Serum bicarbonate: 22 meq/l. Renal ultrasound: normal. Question What is the most likely diagnosis?

Correct answer: Acute interstitial nephritis Explanation This patient most likely has acute interstitial nephritis(AIN) secondary to his tetracycline exposure. AIN is an immune-mediated form of acute kidney injury (acute renal failure). Patients develop varying degrees of renal failure, characterized by changes in urine output, electrolyte imbalances, acidemia, and azotemia (elevations in serum creatinine with or without nausea, sleep disturbances, shakiness, etc.) in response to viral, bacterial, immunological, or pharmaceutical insults. Hypersensitivity reactions to the above exposures lead to tubulointerstitial Inflammation in AIN. Elevations In blood eosinophils and pyuria follow. White blood cell casts may be seen in the urine, as well as a small amount of hematuria and proteinuria (<1.5 g/d-2 g/day). Some 5 - 15% of cases of acute kidney injury occur secondary to acute interstitial nephritis. AIN is often diagnosed based on history of renal failure and urine findings 2 weeks after exposure to a new medication or a viral or bacterial infection. It may occur earlier in patients previously sensitized to the offending medication. A variety of antibiotics, ACE inhibitors, proton pump inhibitors, seizure medications, etc., are known to cause AIN. The development of AIN is not dependent on the size of the dose of the medication given. Many infections, including HIV, EBV and mumps, can also cause AIN; additionally, immunological diseases, such as lupus and Wegener's granulomatosis, can cause AIN. AIN is treated by removing the offending medication and avoiding its future use. If AIN occurred in the setting of an infection, or immunologic or neoplastic process, that disease should be treated. Acute glomerulonephritis is a form of acute kidney injury in which glomeruli are inflamed and irritated, usually due to infectious or immunologic processes. Azotemia, (see above) hematuria, and red blood cell casts are noted, which were not seen in this patient. Acute tubular necrosis (ATN) is a form of acute kidney injury in which the renal tubules are directly affected. Contrast and cisplatin are 2 toxins known to cause ATN. Damaged tubules slough cells into the filtrate, giving urine a dark appearance; 'muddy brown casts' are seen in the urine. No such casts were noted here. Acute urinary obstruction is a form of acute injury caused by the inability to clear urine (and hence all wastes cleared by the kidneys) from the body. Acute obstruction, with associated hydronephrosis, should be noted on renal ultrasound. It was not noted in this case. Pyelonephritis is an infection of the kidney. Patients may present with fever, flank tenderness, nausea and decreased urinary output. White cell urine casts and bacteria may be noted in the urine. Patients often appear more ill than this patient did. Additionally, there was no bacterial growth noted in his urine.

A 39-year-old male was out on a ranch with his friends for the weekend. He indulged in horseback riding daily, stretched out over several hours in the afternoon. On his return home, he experienced high fever with chills and malaise, myalgias, dysuria, perineal pain, and cloudy urine. Examination in the ER revealed a temperature of 101.5°F, pulse 110/min, BP 120/80 mmHg, and respiratory rate of 16/min. There was no pallor, jaundice, or lymphadenopathy. Lungs were clear and no murmurs appreciated. Abdominal exam showed no tenderness, masses, ascites, or hepatosplenomegaly. Bowel sounds were active, and rectal exam showed exquisite tenderness. Significant labs included WBC 13,400/uL and urinalysis with 15 WBC and 4 RBC. Question What would be a provisional diagnosis with blood and urine cultures pending?

Correct answer: Acute prostatitis Explanation Acute prostatitis is defined as an inflammation of the prostate gland that develops suddenly and is common in men, likely due to reflux of infected urine into intraprostatic ducts. This can happen after instrumentation, catheterization, or trauma, like horseback riding, biking, etc., and worsened by dehydration, as in this patient. The National Institutes of Health classification of inflammatory conditions of the prostate is as follows: I Acute prostatitis II Chronic bacterial prostatitis III A Chronic prostatitis/pelvic pain syndrome, inflammatory III B Chronic prostatitis/pelvic pain syndrome, noninflammatory IV Asymptomatic inflammatory prostatitis Gram negative organisms are the main culprit, including E.coli, proteus, klebsiella, enterobacter, and pseudomonas. Symptoms of dysuria, fever, perineal pain, and tender prostate are typical. Treatment is with trimethoprim-sulfamethoxazole or quinolones for 4 weeks. In sicker patients, hospitalization may be needed, in which case IV antibiotics with aminoglycoside and ampicillin should be given until the patient is afebrile for 24-48 hours, then oral antibiotics continued for total of 4-6 weeks to avoid complications such as abscess formation or chronic prostatitis. Acute pyelonephritis presents with fever, flank pain, tender renal angle, and normal rectal exam. Treatment includes oral fluoroquinolone or trimethoprim-sulfamethoxazole for mild to moderate disease and IV ceftriaxone or a fluoroquinolone for hospitalized patients, to be substituted with oral antibiotics after improvement in symptoms. Total duration of antibiotics should be 10-14 days. Acute urethritis is associated with dysuria and urethral discharge with pruritus at urethral meatus. Fever, chills, frequency, urgency, and hematuria are uncommon. It may be gonococcal, which is the most common cause of urethritis in men or nongonococcal urethritis (NGU). Although most cases of NGU are due to chlamydia trachomatis, other etiologies include T. vaginalis, Mycoplasma genitalium, and Ureaplasma urealyticum. Gram stain and culture or the urethral discharge should be done. Treatment is with ceftriaxone 125mg IM, cefixime 400mg PO, ciprofloxacin 500mg PO, or ofloxacin 400mg PO, all in a single dose in gonococcal urethritis and azithromycin 1gm PO or doxycycline 100mg BID for 7 days or ofloxacin 400mg PO BID for 7 days for NGU. Rectal abscess is a distant possibility in this patient. It presents with constant pain in the rectal area and perhaps fever and malaise but no dysuria or cloudy urine. Rectal exam will be tender and reveal a fluctuant mass. UA, however, will not be abnormal. Treatment is with incision, drainage, and perhaps antibiotics for anaerobic coverage. Anal fissure presents with excruciating pain with the passage of bowel movements and is associated with constipation. The passage of stool may be accompanied by bright rectal bleeding usually limited to a small amount on the toilet paper but sometimes more profuse bleeding. Treatment aims at relaxing the sphincter, keeping bowel movements soft and smooth, and pain control.

A 19-year-old woman presents with a 4-day history of fever accompanied by chills, nausea, vomiting, and back pain. She denies any chest pain, cough, or urinary symptoms. On further questioning, she said she had 2 episodes of diarrhea yesterday with vague abdominal discomfort. She has no other significant past medical history, is on no medications, and has no allergies. Family history is significant for hypertension in father and arthritis in mother. She is single, has no children, does not smoke or drink, and works part time as a waitress. On exam she has a temperature of 102.4°F, pulse rate of 110/min, BP 110/60 mmHg, and SP02 of 92%. Mucous membranes are dry, and sclera is clear. Lungs are clear, and heart sounds are normal. There is mild left flank tenderness and tenderness in the left costovertebral angle. Labs: Hb 12g/dl, WBC 17,000/uL, bands were 18% and platelets 350,000/uL. Chest X ray and EKG are normal. Urinalysis shows 35 WBC, 6 RBC, and no casts. Question What is the most likely diagnosis?

Correct answer: Acute pyelonephritis Explanation The symptoms of high fever with chills, nausea, vomiting, and back pain with tenderness in the renal angle are classic for pyelonephritis. It is a common condition in young women. The common organisms are gram negative, for example E.coli, klebsiella, proteus, enterobacter, and pseudomonas. Gram positive bacteria, like staphylococcus aureus and enterococcus fecalis, may also be seen. The usual mode of infection is ascent from the lower urinary tract, except for staphylococcus aureus, which is hematogenously spread. Leukocytosis with a left shift and abnormal urine with pyuria and bacteriuria confirm the condition. Absence of pyuria should be an indication to look for an alternative diagnosis. Hematuria may also be present. Blood and urine cultures should be done. Imaging may be needed in complicated cases, in which scenario an ultrasound may reveal hydronephrosis due to obstruction from a calculus or other causes. It is generally recommended that all males with acute pyelonephritis undergo imaging with ultrasound or CT scan, since such an infection is usually associated with an anatomical abnormality like enlarged prostate, etc. A long urethra and absence of organisms residing in vagina makes it unusual for men to have a urinary infection with a normal anatomy. Treatment should be started empirically without waiting for culture results, since they are usually not available immediately and, as they become available, antibiotics may be changed accordingly. Urine gram stain, which is available right away, may be a useful tool to direct antibiotic treatment. Indications for hospitalization include vomiting, pregnancy, HIV disease, diabetes, impending septic shock with unstable vitals, and other comorbidities like renal failure, post transplant, etc. It should be treated with oral fluoroquinolone or trimethoprim-sulfamethoxazole for mild to moderate disease and IV ceftriaxone or a fluoroquinolone for hospitalized patients, to be substituted with oral antibiotics after improvement in symptoms. Total duration of antibiotics should be 10-14 days. Prognosis is usually good if diagnosis is prompt, treatment appropriate, and complications absent. Acute cystitis is a milder disease, which is more common in women than men due to a short urethra and proximity to vagina with its abundance of micro-organisms. About 50-60% adult women have had a urinary tract infection in their lives at some point. 10% postmenopausal women also have been found to get these infections. Coitus seems to be a predisposing factor, and symptoms quite often arise after sexual intercourse (honeymoon cystitis). The offending organisms include gram negative bacteria, such as E.coli, in 80-85% cases in women; it is also common in men. Most other cases in women are due to staphylococcus saprophyticus, though this is uncommon in case of males. This is a coagulase negative staphylococcus, which is normally considered benign but is actually a true urinary pathogen and should not be ignored. Rarely, klebsiella, proteus, enterococci, etc. may be isolated. Symptoms include low grade fever, dysuria, urgency, increased frequency of urination, and suprapubic abdominal pain. Occasionally women may have gross hematuria. There is suprapubic tenderness on examination without costovertebral angle tenderness. Urinalysis shows pyuria, bacteriuria, and hematuria. Hematuria is absent in female patients with urethritis and vaginitis, which can cause similar symptoms and can be used to differentiate the conditions. Urine culture is usually positive for the causative organism. Treatment is based on culture reports. Uncomplicated cystitis in women can be treated with a 3-day course of trimethoprim-sulfamethoxazole, trimethoprim alone, fluoroquinolone, or cephalexin. A 7-day course of nitrofurantoin is also adequate. Men should be evaluated for underlying conditions since uncomplicated cystitis is uncommon in males. A 7-day course is recommended even for uncomplicated cases in men. Acute gastroenteritis, or food poisoning, has a similar picture but without costovertebral angle tenderness. Diffuse abdominal pain and watery diarrhea are the predominant symptoms. Fever may be low grade or high grade. UA is usually normal. Acute salpingitis, or pelvic inflammatory disease, is characterized by lower abdominal pain and tenderness, abnormal vaginal discharge and/or bleeding, dyspareunia with adnexal tenderness, and cervical motion tenderness on a pelvic examination. An acute episode may present with high fever and chills, profuse vaginal discharge, and severe lower abdominal pain. Leukocytosis is found in less than 50% patients; UA is mostly normal and culture of the vaginal fluid should be done. Treatment is with broad spectrum antibiotics. Acute diverticulitis is usually left sided and manifested by left lower quadrant abdominal pain and tenderness with diarrhea and occasionally low grade fever without chills. Leukocytosis may sometimes be present with sterile pyuria. The patient does not have back pain or costovertebral angle tenderness and seems well hydrated. Treatment is with ciprofloxacin and metronidazole for 7 - 10 days.

A 28-year-old woman receives a lung transplant due to cystic fibrosis. 2 months after the transplant day, she is readmitted with paralysis of the left body. A basal ganglia stroke is diagnosed by computed tomography (CT). After 2 weeks of hospital stay, she shows signs of cardiac failure and is intubated. A posterior mitral valve leaflet vegetation with severe regurgitation is identified by transesophageal echocardiography (TEE). She is submitted to mechanical mitral valve replacement. Colonies of Aspergillus fumigatus are detected in the excised mitral valve leaflets. Intravenous anti-fungal therapy with amphotericin B is started. 8 days later, she presents with fever, shortness of breath, and edema. A rapid increase in creatinine levels is observed. Fraction of excretion of sodium (FENa) is increased (> 3%), and urine analysis reveals epithelial casts. Question In what example would regeneration with complete re-establishment of normal morphology and function of kidney tissue most likely happen?

Correct answer: Acute tubular necrosis Explanation The clinical and laboratory findings of the vignette point to the diagnosis of acute tubular necrosis (ATN). In cases of ATN, cells heal completely by regeneration. Epithelial cells that line the kidney tubules are destroyed by ischemia (e.g. hypovolemic shock, sepsis, cardiac heart failure), toxins (e.g., aminoglycosides, amphotericin B as in this case, contrast media, lead, cisplatin), myoglobinuria in rhabdomyolysis, or hyperurecemia in acute tumor lysis. Re-absorptive mechanisms of sodium and water are lost with the tubular cells and acute renal failure occur. 2 features corroborate for the regeneration in ATN to be complete: 1) necrosis occurs in a patchy pattern; 2) the collagen framework of the tubules (epithelial basal membrane and the interstitium) remains intact. The existing epithelial cells replicate using the basement membrane as a guide, and they bring the kidney back to normal. After regeneration is complete, the damage is undetectable, even microscopically. In contrast, healing will take place by repair when the causative process affects the kidney collagen framework. While enzymes released by inflammatory cells in chronic pyelonephritis damage this framework, an infarction leads to its total collapse. Because of the history of previous cerebral infarction and fungus endocarditis, the possibility of another embolization with kidney infarction in this case is high. However, the clinical and laboratory findings do not support this diagnosis. Diffuse cortical necrosis (DCN) is an acute generalized cortical infarction of both kidneys, leading to atrophy of the cortex with preservation of the medulla. DCN is the pathological progression of ATN: once rapidly corrected, acute renal ischemia leads to ATN. A more prolonged ischemia may lead to DCN. DCN is associated with late pregnancy complications (e.g., placental abruption) and septic shock. Polycystic kidney disease is a genetic condition that leads to the formation of multiple cysts and irreversible destruction of the kidney parenchyma.

Case A 23-year-old woman presents with dysuria and left flank pain. She is 27 weeks pregnant. Physical examination reveals a temperature of 38°C (100.4°F); there is tenderness with percussion over the left costovertebral angle. Urinalysis shows 15-20 WBC/hpf and 15-20 bacteria/hpf. Question What is the most appropriate management plan?

Correct answer: Admit for IV antibiotics. Explanation The patient should be admitted for IV antibiotics. Urinary tract infections in pregnant patients are associated with significant risks greater than in non-pregnant patients. Mechanical pressure on the ureters and bladder by the enlarging uterus, progesterone-mediated relaxation of smooth muscle, incomplete bladder emptying, and increased urinary tract volume contribute to the increased incidence of urinary tract infections during pregnancy as well as the increased severity of associated complications. Approximately 5-7% of pregnant women have asymptomatic bacteriuria. Untreated bacteriuria is associated with a high incidence of prematurity and fetal wastage as well as a 20-40% incidence of acute maternal pyelonephritis. Patients with uncomplicated bacteriuria should be treated with ampicillin, amoxicillin, nitrofurantoin, or a cephalosporin. Pregnant patients with pyelonephritis (i.e., fever, chills, and flank tenderness) are at increased risk for sepsis and preterm labor, however, and they require hospitalization for aggressive therapy with parenteral antibiotics. Sulfonamide antimicrobial agents increase serum bilirubin levels, thereby increasing the risk of neonatal kernicterus; they should be avoided during the third trimester.

A 62-year-old woman is being treated for chronic congestive heart failure. She has been put on hydrochlorothiazide therapy. Her serum electrolyte levels are being monitored and show a persistent hypokalemia. Question The addition of what to her therapeutic regimen would be most appropriate? - Mannitol - Furosemide - Indapamide - Amiloride - Acetazolamide

Correct answer: Amiloride Explanation Amiloride is a potassium-sparing diuretic. Its diuretic effect is not very potent; therefore, it is good to use in combination with other diuretics. Acetazolamide is a carbonic anhydrase inhibitor. It causes a mild diuresis, a marked elevation of urinary pH, and a significant loss of potassium. Furosemide is a loop diuretic. It has a rapid onset of action and is a potent diuretic. However, it also causes potassium depletion and would only worsen the hypokalemia. Indapamide is a thiazide analog with a long duration of action. If anything, it would exacerbate the hypokalemia. Mannitol is an osmotic diuretic and would not be recommended in this patient. Furthermore, it would not have a potassium-sparing effect.

Case A 6-year-old boy presents with a fever, malaise, and dark urine. His mother states that he missed school earlier this month with a sore throat. On examination, his blood pressure is 120/88 mm Hg, pulse is 82/min, temperature is 100.6°F, and respirations are 16/minute. On physical assessment, the patient appears ill with only mild costovertebral angle tenderness noted. Urinalysis reveals the following: Urinalysis Result Specific gravity 1.00 pH 5.2 protein + 1 blood + 2 glucose negative ketones negative bilirubin negative urobilinogen negative nitrates negative leukocyte esterase + 1 Microscopic examination reveals RBCs, renal tubular epithelial cells, RBC casts, and granular casts. Question Considering the most likely diagnosis, what laboratory test will help determine the most likely etiology?

Correct answer: Anti-DNase B serology Explanation The correct answer is anti-DNase B serology to identify post-streptococcal glomerulonephritis. Group A β-hemolytic streptococci pharyngitis may result in the delayed complication of post-streptococcal glomerulonephritis 10-14 days after the infection. Patient presentations may range from subclinical symptoms to acute nephritic syndrome as streptococci may produce streptolysin, DNase, and hyaluronidase that lead to tissue destruction and disseminate infection. Serology testing to identify antibodies to these exoenzymes can aid in the diagnosis by demonstrating indirect evidence of infection. Confirmation may require serial antibody draws that reveal a rise in titer levels above the baseline. Urine culture and sensitivity would be appropriate if the clinical picture only entailed fever and flank tenderness in the presence of pyuria and hematuria, suggesting a urinary tract infection. That diagnosis does not explain the presence of proteinuria with renal tubular epithelial cells and casts. These indicate intrinsic kidney damage, which is not seen in urinary tract infections. Further serology testing is indicated in the post pharyngitis period. Urine cytology is ordered in the presence of gross or microscopic hematuria, which is often painless, to identify malignant cells in the urinary tract. This patient does not fit the epidemiologic profile or clinical presentation of malignancy to warrant cytology testing. Erythrocyte sedimentation rate can aid in detecting an inflammatory response, such as nephritis, but it lacks specificity to identify the infectious process. Urine protein electrophoresis is indicated to identify abnormal levels of free monoclonal light chains (Bence Jones protein) from immunoglobulins in cases of myeloma. References

Your patient is a 55-year-old man presenting with a strong, sudden need to urinate. He feels his bladder spasms and sometimes has involuntary loss of urine. The problem started several months ago and seems to be worsening. He is very upset because it sometimes happens during business meetings. His urinalyses were always normal. He takes no medications. Careful examination shows that he has urge incontinence. Bladder training and behavioral techniques were unsuccessful and you decided to introduce medication. Before suggesting a medication for this condition, you will tell him that this drug may have some side effects: dry mouth, difficulty in urination, constipation, blurred vision, tachycardia, drowsiness, and dizziness. Question This may happen because you will be prescribing what type of drug?

Correct answer: Anticholinergic Explanation Urge incontinence is defined as involuntary loss of urine occurring for no apparent reason together with a feeling of urinary urgency (a sudden need or urge to urinate) that represents a hygienic or social problem to the individual. The most common cause of urge incontinence is involuntary and inappropriate detrusor muscle contractions. The drug you will suggest is anticholinergic (like Oxybutynin). It will relieve urinary and bladder difficulties, including frequent urination and urge incontinence by decreasing muscle spasms of the bladder, increasing the capacity of the bladder, and delaying the initial urge to void. It is a competitive antagonist of M1, M2, and M3 muscarinic acetylcholine receptors, and it can act as spasmolytic on bladder smooth muscle at higher doses. Anticholinergic side effects are dry mouth, difficulty in urination, constipation, blurred vision, tachycardia, drowsiness, and dizziness. Cholinergic drugs will cause slowing of the heartbeat and an increase in normal secretions. For this reason, patients who already have a problem with incontinence should not be advised to use these drugs. Epinephrine is not indicated in this patient. Epinephrine stimulates the ends of the sympathetic or inhibitory nerves of the bladder, with the effect of relaxation of the bladder muscles and the increase in tone and rate of contraction of the ureter. The secretion of urine is increased synchronously with the rise in arterial pressure. It will also cause overacting heart, palpitation, and vomiting. There is no need for antibiotics in a patient with normal urine analysis for a problem that lasts several months. Botulinum toxin is given as intradetrusor injection in patients who have failed pharmacological therapy. It has been shown to decrease episodes of urinary leakage by preventing the release of acetylcholine from presynaptic membrane. It is also indicated for urinary incontinence in patients with neurologic conditions (e.g., spinal cord injury, multiple sclerosis). It sometimes can cause urinary retention given as intradetrusor injections and occasionally headache, light-headedness, fever, abdominal pain, and diarrhea (not necessarily a direct result of Botox).

A 78-year-old Caucasian woman is admitted with a provisional diagnosis of diverticulitis, with complaints of acute abdominal pain and lack of eating or drinking well for several days. Her past medical history is significant for diabetes mellitus, diagnosed 12 years ago, and hypertension (though she was hypotensive at admission). Both conditions were reported to have been under good control. Her medications include regular and long-acting insulin and hydrochlorothiazide/lisinopril 25/20 mg QD. Her son had recently been giving her 800mg ibuprofen BID-TID for her abdominal pain during the last week. She has no known allergies. While in the hospital, the patient's laboratory results are followed daily. Two days after the CT with contrast, it is noted that her serum creatinine has risen to a level of 3.5 mg/dL. Records from 1 month ago at her family physician showed her labs to include a hemoglobin A1C of 6.8%, creatinine of 1.8 mg/dL, and her blood pressure was 107/68 mm Hg. Question Ico-delete Highlights What measure would have been the best prevention for this patient's sudden decline in renal function?

Correct answer: Begin volume expansion prior to the contrast study Explanation Mild volume expansion prior to the CT with IV contrast is recommended to prevent contrast nephropathy in higher-risk patients.1 Hydration should begin an hour or more before the procedure and continue several hours after the CT. There is evidence showing better results with NaHCO3 over regular saline (NaCl).2 This patient had several indicators of volume depletion - she was taking a diuretic and had decreased oral intake, along with potentially some fluid/blood loss with the diverticulitis, and her BP was hypotensive. A change to metformin is not recommended for this patient. Metformin, while not nephrotoxic directly, can increase the likelihood of lactic acidosis.3 She likely has some pre-existing renal disease (diabetic nephropathy) and metformin is relatively contraindicated in this patient. Some experts recommend cessation of metformin when patients undergo contrast studies in order to decrease the likelihood of contrast nephropathy. Loop diuretics are also not recommended. These drugs can increase risk of contrast nephropathy and should be discontinued, preferably a few days prior to any contrast studies.4 Though mannitol has a role in treatment and prevention of oliguria, it is an osmotic diuretic and can increase risk of renal damage after exposure to contrast.3,5

A 65-year-old man presents with gradual increase of urinary frequency over the previous few months. Most recently he has difficulties starting to urinate and the stream seemed slow. Despite of waking up several times per night to release his bladder, he continues to have a sensation of incomplete emptying. He has no history of sexually transmitted diseases (STDs), surgeries, or urinary tract infections. Upon examination, he is slightly overweight and his vitals are normal. His abdomen feels soft, the genitals are benign, and his prostate is non-tender and moderately enlarged (35-40g). Lab tests are negative for blood presence in urine and indicate a protein-specific antigen (PSA) of 1.3 ng/ml. Question What is the most likely diagnosis?

Correct answer: Benign prostatic hyperplasia Explanation Benign prostatic hyperplasia (BPH) is the most likely diagnosis. The symptoms are most likely caused by the enlarged prostate. They are common symptoms in men in the age-range of the patient. The patient had symptoms established gradually during several months, and is clearly bothered and irritated by them. Urinalysis does not indicate any sign of infection, the PSA level is normal and the prostate is smooth and does not show any asymmetries. The answer overactive bladder is incorrect.The patient has difficulties starting the stream. A patient suffering from an overactive bladder usually has difficulties controlling the urge to urinate and most likely experiences involuntary loss of urine. The answer prostate cancer is incorrect. The patient has a slightly enlarged prostate, which is non-tender and symmetrical, and the PSA level is normal, not indicative of a malignant growth. The answer prostatitis is not correct. Prostatitis is defined as an inflammation and/or infection of the prostate. It usually leads to a tender prostate, local pain, and high fever. In such cases the symptoms are of much shorter duration than in the symptoms presented in this patient. The answer urinary tract infection is not correct. The patient does not show any symptoms associated with infections: fever, headaches, burning sensation or pain while urinating etc., and the urinalysis is negative for blood presence.

Case A 78-year-old woman presents after falling down the staircase and remaining there for 24 hours. Emergency department evaluation shows a fractured pelvis, acute kidney injury, and bilateral hydronephrosis near the upper ureters. Her kidneys are of normal size with normal cortical thickness. She was previously well, and her only medications were multivitamins and calcium. She has not yet voided, and she notes decreased urine output after her fall. Question What will most likely assist in urine flow and resolution of her acute kidney injury?

Correct answer: Bilateral ureteral stents and nephrostomy tubes Explanation This patient has bilateral hydronephrosis and an acute kidney injury resulting from a pelvic fracture, which is compressing and obstructing the most proximal part of both her ureters. Until the fracture is repaired and the compression is relieved, only bilateral ureteral stents and nephrostomy tubes will assist in urine flow. Because of the complexity of the urinary system, more than one type of treatment may be required. A ureter stent, involves inserting a hollow tube inside the ureter to keep it open. Nephrostomy tubes are placed through your back to drain the kidney directly. Relief of urinary obstruction represents the most common indication for percutaneous nephrostomy placement, representing 85-90% of patients in several large series. Her compression is proximal to the bladder, making all choices that bypass the bladder (suprapubic catheter, Foley catheter) ineffective. Once the compression is relieved, hydronephrosis resolves and associated edema is diminished, the patient may void on her own. Diapers may not absorb the high urine outputs seen after obstruction, so they may not facilitate monitoring intake and output. Intravenous fluids may be necessary for several reasons. The patient likely had little fluid input following her fall. Decreased effective circulating volumes may contribute to impaired glomerular filtration. After her obstruction is relieved, she will likely have large volume urine output and have high urine sodium losses as her tubules recover. How completely her renal function will recover remains to be seen. Animals may recover <25-50% of glomerular function after such an obstruction. Close monitoring of serum creatinine, electrolytes, and urine output is critical in the days following obstruction. In the weeks and months afterwards, further recovery may be seen.

A 35-year-old man presents with right flank pain that has progressively worsened over the past 12 hours and is now radiating into his right groin and testicle. He currently rates the pain as an 8/10 and complains of some nausea but no vomiting. He denies ever having this type of pain previously. He states that he thinks he had some blood in his urine at his last void, but he did not notice any prior to that. The patient is unable to sit still during the interview and refuses the portion of the physical exam where CVA tenderness is assessed; he also refuses any palpation of the abdomen or testicle on the right side. Vital signs include a temperature of 99.2°F, BP is 156/84 mm Hg, RR is 12, and oxygen sat 99% on room air. Question What is the best study to assess this patient's condition?

Correct answer: CT scan of abdomen and pelvis without contrast Explanation This patient most likely suffers from urolithiasis. Such patients typically present with unilateral renal colic that often radiates to the ipsilateral groin or testis and hematuria. These patients generally cannot sit still secondary to the pain and discomfort. They can also exhibit guarding, nausea, and vomiting in some cases. A stone protocol (non-contrast) CT scan has become the standard initial workup of patients with suspected stone. This study is especially useful in the emergency department since it can quickly and effectively diagnose urolithiasis. A CT scan gives the most information (location and size of the stone, hydronephrosis, any anatomical variations, etc.) to the urologist who will likely see the patient. Stones are more common in men than women (3:1), and initial presentation is typically in the third or fourth decade of life. MRIs are used more often in assessing soft tissues; they are not typically useful in urolithiasis cases. Ultrasound of the kidneys can reveal hydronephrosis and cystic or solid lesions. Stones can sometimes be assessed as well, but small stones are occasionally not visible. A kidney, ureter, bladder X-ray will show approximately 90% of stones. Uric acid stones are not visible on plain film, and some stones are difficult to see because of size or location. A KUB is a viable option but not the best study to order. Intravenous urogram is most useful after a thorough bowel preparation, so a non-emergent study will typically yield more information than an emergent study. However, if a high-grade obstruction is suspected, emergent intervention with intravenous urogram may be warranted. These patients can have an elevated temperature and a CT scan can show high-grade obstruction, so that is still the best initial study.

A 55-year-old woman presents with a 2-month history of gross hematuria. She states she has no pain with urination, but the hematuria is persistent. Upon questioning, she states that she does have some progressively worsening left flank pain. The pains are not debilitating, but they are nagging. She has no chronic medical problems. She admits to a 50 pack-year smoking history, and she states she is currently retired from her job as a teacher. Vital signs are within normal limits, and physical exam reveals a left side abdominal mass. Urine dipstick only shows too numerous to count RBCs, and urine cultures are negative. Question What test would best confirm your diagnosis?

Correct answer: CT scan with and without contrast kidney protocol Explanation Hematuria is the most common presenting sign of urinary tract cancer. Silent or painless hematuria suggests tumor or renal parenchymal disease. Renal cell carcinoma can present with flank pain, hematuria, persistent back pain, and an abdominal mass; also, it can be found incidentally on CT scan. So the clinical picture points to cancer, with renal cell carcinoma as the most likely diagnosis. The study of choice to evaluate the kidneys for masses is a CT scan with and without contrast kidney protocol. Any mass that enhances with IV contrast should be considered RCC until proven otherwise. Bladder cancer often presents with gross hematuria, but it is most typically painless. The left flank pain and back pain are more characteristic of RCC. Bladder cancer is the second most common urologic cancer, and the mean age at diagnosis is 65. It is more common in men than women (2.7:1), and 98% of primary bladder cancers are epithelial malignancies (majority urothelial cell carcinomas). A cystoscopy is the diagnostic study of choice for suspected bladder cancer. An MRI can help differentiate between a cyst and a solid lesion, but it is not the best study for suspected RCC. A KUB can often miss some smaller solid lesions in the kidney, as can an IVP, especially if the lesion is not located in the renal collecting system. These are both useful in imaging stones. The correct workup for hematuria includes both upper and lower urinary tract studies. Ideally, a patient would get both a CT scan with and without contrast kidney protocol and a cystoscopy, but you are looking for different things with each of these studies; in this patient, a CT scan is more likely to confirm your diagnosis.

A 27-year-old man has a 2-hour history of left-sided flank pain. He was previously healthy and takes no medications or herbal supplements. A CT scan showed a 4 mm stone in his left ureter. He is diagnosed to have the most common stone. What type of renal stone is most likely in this patient?

Correct answer: Calcium oxalate Explanation Calcium oxalate stones are the most common type of renal stones or nephrolithiasis. Nephrolithiasis is a common problem, affecting some 2 - 9% of the population. Patients with nephrolithiasis are likely to have recurrent episodes. 40 - 50% of patients are likely to have recurrent stones after an episode of nephrolithiasis. According to Pietrow, each of the following stone types is seen with the following frequency: Calcium oxalate (70%) Calcium phosphate (5 - 10%) Uric acid (10%) Struvite (magnesium ammonium phosphate) (15 - 20%) Cystine (1%) Crystals form in urine when the urine is supersaturated with crystal-forming solutes such as calcium, phosphate, and uric acid. Some patients overexcrete solutes; others drink inadequate amounts of fluids to keep solutes dissolved. Stones also occur when the urine is infected with urea-splitting bacterium. Here, urea is broken down into ammonia and bicarbonate, which then forms ammonium hydroxide and bicarbonate, which are the components of struvite stones. Struvite stones consist of a triple phosphate of calcium, magnesium, and ammonium. Certain stone inhibitors, such as pyrophosphate, citrate, and magnesium, prevent crystal growth. In patients who have low levels of these inhibitors, stones are more likely to form. Stone prevention focuses on adequate hydration and dietary moderation of foods likely to cause stones. Specifically, decreased sodium and dairy products are recommended for patients with calcium stones; decreased liver and purine rich foods are recommended for patients with uric acid stones; and decreased of nuts, chocolate, some vegetables (beets and spinach) is recommended for patients with oxalate stones. Citrate supplementation is used in patients with low levels of urinary citrate, a stone inhibitor. Thiazides may be used to treat hypocalciuria in patients who overexcrete calcium.

Case A 23-year-old man presents with burning on urination and a light greenish-yellow penile discharge. He is afebrile, but otherwise well. He admits to having unprotected anal sex with a man. Question According to the Centers for Disease Control, what treatment is most appropriate for the most likely diagnosis?

Correct answer: Ceftriaxone 250 mg IM once and azithromycin 1 g now Explanation The correct answer is ceftriaxone 250mg IM once and azithromycin 1g now. According to the Centers for Disease Control, the correct first-line treatment for uncomplicated gonococcal urethritis is ceftriaxone; there has been evidence of increasing resistance of gonococci strains against fluoroquinolones in the United States. In addition, when chlamydia status is unknown in a patient with gonorrhea, it is appropriate to treat for suspected chlamydia infection with either azithromycin 1g once or doxycycline 100 mg 2 times daily for 7 days. The other answers are incorrect; fluoroquinolones, such as ciprofloxacin, are no longer indicated for the treatment of gonorrhea infections. Penicillin is not indicated for the treatment of uncomplicated gonococcal urethritis.

A 24-year-old man presents with gradual onset of scrotal pain. The pain is constant, and it lessens with elevation of the testicles. The patient developed a fever this morning. He has been voiding small amounts frequently, and he is also experiencing dysuria. The patient has a negative past medical history and past surgical history. He does not smoke, drink alcohol, or abuse illicit drugs. He is currently sexually active with a new partner. Examination reveals edema of the left testicle; there is tenderness to palpation. Cremasteric reflex is positive. Question What is the most appropriate intervention at this time?

Correct answer: Ceftriaxone and doxycycline Explanation The patient is most likely suffering from acute epididymitis, so he should be treated with a single dose of IM ceftriaxone along with a 10-day course of oral doxycycline. Epididymitis is inflammation of the epididymis, the tube that connects the testicle to the vas deferens. Epididymitis is typically the result of infection. In younger, sexually active man, it is usually caused by Chlamydia trachomatis or Neisseria gonorrhoeae. In older man, epididymitis is associated with Gram-negative rods. Given the patient's age and sexual history, he should be treated with ceftriaxone, which will cover gonorrhea, as well as doxycycline, which is active against chlamydia. Biopsy of the testicle is an incorrect response. The patient has no identifiable mass on examination, only testicular edema. Biopsy of the prostate is incorrect. The patient is likely suffering from epididymitis and a prostate biopsy would be inappropriate in this patient Insertion of a Foley catheter is an incorrect response. "Ciprofloxacin and doxycycline" is an incorrect response. If the patient had been older and did not have sexual risk factors for sexually transmitted epididymitis, then ciprofloxacin might have been required to cover for Gram-negative rods. However, in this case, ceftriaxone and doxycycline will better cover the likely causative agents.

Case A 25-year-old sexually active man notices that he has burning and pain while urinating. He also notices some urethral discharge. He sees you in your office for a consultation, and you order several laboratory tests. One of the tests that you order is a Gram stain and culture on a sample of the discharge. The results are negative, and gonorrhea is ruled out. After reviewing all the lab results, you tell your patient that he has nongonococcal urethritis (NGU). Question What is the most common cause of nongonococcal urethritis?

Correct answer: Chlamydia trachomatis Explanation Chlamydia are bacteria. Characteristic of chlamydia infections is the development of inclusion bodies. Chlamydia trachomatis cause 30-40% of all cases of nongonococcal urethritis in heterosexual males. Chlamydia trachomatis causes more nongonococcal urethritis in boys/men than Trichomonas vaginalis does. Urethritis is an inflammation of the urethra. It is classified as either gonococcal urethritis (caused by Neisseria gonorrhoeae) or nongonococcal urethritis (caused by something other than Neisseria gonorrhoeae). Common causes of nongonococcal urethritis are Chlamydia trachomatis, Trichomonas vaginalis, and Ureaplasma urealyticum. Chlamydophila psittaci does not cause nongonococcal urethritis. Chlamydophila psittaci causes a systemic illness called psittacosis, parrot fever, or ornithosis. Psittacosis is acquired from birds. Trichomonas vaginalis is a protozoan. Trichomonas vaginalis can cause nongonococcal urethritis in males. It also causes trichomoniasis vaginitis, sometimes called trichomoniasis. Ureaplasma urealyticum is considered a mycoplasma. Ureaplasma urealyticum is in the family Mycoplasmataceae and the genus Ureaplasma. Bacteria in the genus Ureaplasma require urea; therefore, Ureaplasma urealyticum is found primarily in the genitourinary tract. Ureaplasma urealyticum is a common cause of nongonococcal urethritis. Pneumocystis jirovecii (formerly called Pneumocystis carinii) is a fungus, not a protozoan. In an immunosuppressed host, Pneumocystis jirovecii can cause pneumonia. Pneumocystis jirovecii does not cause nongonococcal urethritis.

A 67-year-old man presents with a subacute onset of lower urinary tract symptoms. He is unable to discuss his past medical history or current medications. An initial genitourinary workup is started, and a microscopic urinalysis reveals granular and waxy casts. Question With what disease process are his results closely associated?

Correct answer: Chronic renal disease Explanation The correct response is chronic renal disease. Casts are cylindrical structures, consisting of clumps or clusters of cells or material that can form in the renal distal and collecting tubules of the kidney. Casts form when the pH of the urine is acidic and when the urine is very concentrated. Casts dislodge from the kidney and can be seen in the urine. In order to see casts, urine must be visualized under low power on a microscope. There are various types of casts that can be characterized into acellular versus cellular casts; each category can be further characterized, and the various casts can be associated with various disease processes. Granular casts are the 2nd most common type of cast and result from the breakdown of cellular material. They are most often indicative of chronic renal disease, but can also be seen if a patient has just vigorously exercised. Waxy casts are also indicative of advanced renal disease, specifically indicating a more chronic issue. Fatty casts are the result of the breakdown of lipid-rich epithelial cells; they are pathognomonic for high urinary protein nephrotic syndrome. High urinary protein nephrotic syndrome does not lead to the formation of granular casts. Nephritic syndromes, urinary tract injury, glomerulonephritis, and vasculitis can all result in red blood cell casts. Whenever there are red blood cells within a cast, there is a strong indication for glomerular damage from a number of different disease processes. Glomerulonephritis does not lead to the formation of granular casts.

A 35-year-old woman presents with a 24-hour history of fever, right flank pain, vomiting, dysuria, and hematuria. A urinalysis reveals large numbers of red and white blood cells, as well as leukocyte esterase, and a subsequent urine culture was positive for >100,000 Escherichia coli. The patient denies having a history of renal calculi, and a bedside renal ultrasound does not show any stones or hydronephrosis bilaterally. She is diagnosed with a right-sided pyelonephritis and admitted for pain control, hydration, and IV antibiotic therapy. Her symptoms, including her fever, abate quickly, and she is discharged after 48 hours of being afebrile. Question Which prescription would she most likely receive upon discharge?

Correct answer: Ciprofloxacin 500 mg PO BID x 14 days Explanation Ciprofloxacin 500 mg PO BID x 14 days is the correct answer. Patients with pyelonephritis who are sick enough to be treated as an inpatient receive IV antibiotics until they have been afebrile for 24-48 hours. They also must be able to tolerate oral hydration and oral medications before being discharged. Upon discharge, they will be given a prescription for antibiotics that will complete at least 2 weeks of antibiotic treatment. Ciprofloxacin has good coverage for E. coli urinary tract infections, and given at 500 mg PO BID x 14 days would be an appropriate choice as long as sensitivities from her culture showed ciprofloxacin to have sensitivity. Motrin 800 mg PO q 8 hours prn pain is not the correct answer. While Motrin is a good choice for patients who may still have some discomfort related to the pyelonephritis, it is not the most likely prescription for this patient to receive. She had become asymptomatic prior to discharge, so there would not be any reason to prescribe anything to help with pain or discomfort at that time. In addition, she is more likely to receive treatment for her infection than for pain. Nitrofurantoin 100 mg PO BID x 7 days is not the correct answer for several reasons. While nitrofurantoin is often an appropriate antibiotic for E. coli urinary tract infections, it does not achieve tissue levels reliable enough for pyelonephritis treatment. In addition, only 7 days of treatment does not add up to at least 2 weeks total of antibiotics. Fluconazole 150mg PO daily x 7days is not the correct choice. This is an antifungal medication is would not have an indication to use as treatment is this patient case. Amoxicillin 500 mg PO BID x 14 days is not the correct answer. While 14 days of antibiotics is a good length of time, amoxicillin does not have good coverage for E. coli, so it would not be a good choice to treat an E. coli-related pyelonephritis.

An 8-year-old boy is evaluated for persistent bed wetting. He has never been continent, averaging 2 - 3 episodes of bedwetting per week. His urological evaluation revealed a normal bladder and urethra, with no neurological problems. Lately, his problem has been a source of much embarrassment; he is unable to attend camp or sleepovers due to fear of wetting his bed. He has tried multiple interventions, including lifestyle changes, alarm systems, and reward systems. His physical exam shows no abnormalities. His parents are keen on a rapid resolution to his problems, and they insist treatment be initiated. Question What is the best therapy?

Correct answer: Desmopressin Explanation Desmopressin is the correct answer. Desmopressin is an analogue of anti-diuretic hormone, which reduces the production of urine at night. It is effective and has a rapid onset of action. The intra nasal form has been removed from the market in favor of the oral tablet. Desmopressin is indicated in enuresis not responding to lifestyle changes or alarms, when patients/parents want rapid improvement, or in children in whom alarm systems are ineffective. Continued lifestyle changes is incorrect. Lifestyle changes include fluid restriction after 5 pm, restriction of caffeinated drinks, and making the child urinate before bed. In this child, enuresis is pathological; it is persistent after the age of 5, and medical intervention is indicated due to the failure of lifestyle measures. Imipramine is incorrect. Imipramine is an effective drug, but is avoided in children due to its numerous side effects, such as dry mouth, constipation, and drowsiness. Alarm system is incorrect. Alarm systems are effective; however, they have already been tried in this child. Therefore, a different form of therapy is indicated. Coricosteroids is incorrect. They are not used in the management of enuresis.

Case A 13-year-old girl is drowsy and unable to answer questions. Her mother says that she has been extremely thirsty lately and urinates frequently. Her father notes that the patient has also been fatigued. There is a fruity odor to the patient's breath. Blood gases are drawn and reveal the following: pH 7.3 CO2 32 HCO3 17 Question What is the most likely cause of the patient's condition?

Correct answer: Diabetes mellitus Explanation The patient's blood gas results indicate metabolic acidosis. Given her presentation of polydipsia, polyuria, fatigue, and fruity odor to her breath, she is likely suffering from diabetic ketoacidosis from undiagnosed diabetes mellitus. Frequent vomiting would lead to metabolic alkalosis from hydrogen loss. Myasthenia gravis would be associated with respiratory acidosis due to the associated decrease in pulmonary function and therefore decreased clearance of CO2. Ethylene glycol ingestion is associated with metabolic acidosis, but the patient's presentation is classic for diabetic ketoacidosis. Salicylate ingestion is associated with respiratory alkalosis.

A 30-year-old man presents to the ER with fever, malaise, and decreased urine output while taking ibuprofen for back pain. His temperature is 101°F, his blood pressure is 135/85 mm Hg without orthostatic change, and the remainder of his examination is normal. A post-void residual is normal, as is his renal ultrasound. His urine output is estimated at 1.5 L/day. His laboratory work shows: Serum creatinine 2.0 mg/dL Serum potassium 4.2 meq/L Serum bicarbonate 23 meq/L Urinalysis pH 6, trace protein, no blood, no casts, many white cells, no leukocyte esterase or bacteria Complete blood count 5.0x103/mL with elevated eosinophils Fractional excretion urine sodium 1.5% These values are unchanged after administration of 1.5 liters of normal saline. Question What is the initial treatment for your suspected diagnosis?

Correct answer: Discontinue ibuprofen. Explanation This patient has a history and findings consistent with acute (tubulo) interstitial nephritis (AIN) and should discontinue ibuprofen. AIN is an immune-mediated form of acute kidney injury (acute renal failure). Patients develop varying degrees of renal failure, which are characterized by changes in urine output, electrolyte imbalances, acidemia, and azotemia (elevations in serum creatinine with or without nausea, sleep disturbances, shakiness, etc.) in response to viral, bacterial, immunological, or pharmaceutical insults. A variety of antibiotics, ACE inhibitors, proton pump inhibitors, seizure medications, etc. are known to cause AIN. The development of AIN is not dependent on the dose of the medication given. Many infections, including HIV, EBV, and mumps, along with immunological diseases such as lupus and Wegener's granulomatosis can also cause AIN. In the case of this patient, no other triggers for AIN were noted, and ibuprofen is the likely cause. AIN is treated by removing the offending medication and avoiding its future use. If AIN occurs in the setting of an infection or immunologic or neoplastic process, that disease should be treated. Discontinuing the offending medication (if still in use) may cause resolution of illness in a couple of weeks. Patients with incomplete resolution may improve slowly over months. Symptoms, urine output, volume status, serum creatinine, and electrolytes should be monitored to evaluate the need for dialysis. Although he has an elevated serum creatinine, this patient lacks the traditional indications for dialysis, such as uncontrollable hyperkalemia, acidemia, volume overload, and uremia (azotemia + pericardial rub, encephalopathy, and/or asterixis). If renal failure does not improve within a few days of discontinuing the offending medication, patients with AIN can be started on prednisone therapy for a couple of weeks, to be tapered thereafter. For patients with AIN unresponsive to steroid therapy, immunomodulatory therapy such as cyclophosphamide (Cytoxan) can be considered to target the tubular and interstitial irritation that is occurring. Immunomodulatory therapy is premature in this case because other therapeutic maneuvers have not been attempted. Routine steroid therapy is not advised in the treatment of AIN. Cyclophosphamide therapy has many side effects, including neutropenia, and patients on this therapy require close monitoring for infections and decreasing blood counts. References Waikar SS, Bonventre JV. Acute Kidney Injury. In: Kasper D, Fauci A

A 26-year-old G1P0 pregnant woman with type I diabetes presents to her obstetrician for her 20 week appointment. Over the past day, she has had to urinate more frequently; she has also developed a burning sensation with urination. Urinalysis reveals the following: Component Value Color - Yellow Clarity - Hazy Glucose - Negative Ketones - Negative Hemoglobin 3+ Protein - Negative Nitrites - Positive Leukocyte esterase - 3+ WBC > 25 RBC > 25 Urine is sent to the lab for culture. Question What organism is most likely to be grown from this patient?

Correct answer: Escherichia coli Explanation Escherichia coli is the most common cause of urinary tract infections. Neisseria gonorrhoeae may be associated with irritation when voiding, but it is not the most common cause of urinary tract infections. Since the patient is 20 weeks pregnant and has received proper prenatal care, she was likely screened for gonorrhea during her 1st trimester. While it is important to screen for group B streptococcus in the 3rd trimester of pregnancy to avoid colonization of the infant during labor, it is not the most common cause of urinary tract infections. While diabetes may make the appearance of Pseudomonas aeruginosa or Candida species more likely, Escherichia coli is still the organism most commonly associated with urinary tract infections.

Case A 65-year-old Caucasian man presents for a routine physical. He states that he is concerned about the development of prostate cancer. His history is significant for benign prostatic hyperplasia (BPH), for which he underwent a transurethral resection of the prostate (TURP) 3 years ago. His social history is significant for a 50 pack-year history of smoking, and he worked for 40 years as a coal miner. His father died of prostate cancer at age 76. Question What factor would most likely contribute to his risk of developing prostate cancer?

Correct answer: Family history Explanation The correct response is family history. Several risk factors have been identified for prostate cancer, including race and nationality. African-Americans are twice as likely to develop prostate cancer as are Caucasian Americans, and the incidence is higher in North America and western Europe than in other areas of the world. Age is the single largest risk factor, with rates of prostate cancer increasing rapidly after the age of 50. High fat diets and a sedentary lifestyle have also been linked to prostate cancer. Men with first-degree relatives with prostate cancer are twice as likely to develop prostate cancer as are other men, and the risk is even higher if multiple relatives are affected. Other factors, such as a history of smoking and occupational exposures, have not been conclusively linked to prostate cancer. Benign prostatic hypertrophy arises in cells in a different area of the prostate gland, and a history of BPH does not increase the risk of developing prostate cancer.

Case A 60-year-old man presents with difficulty initiating voiding, incomplete emptying, and increasing urinary frequency over the past few months. He has routine fluid intake. He has no history of stones, cancer, surgery, diabetes, or HIV infection; he takes no medications. His physical exam shows a temperature of 98°F, a blood pressure of 128/78 mm Hg, suprapubic fullness, an enlarged prostate, and no peripheral edema. The remainder of his exam is normal. Urinalysis No protein, no blood, pH 6, unremarkable sediment, no glucose Post-void bladder scan 225 mL Renal ultrasound No hydronephrosis, normal cortex, normal size Serum creatinine 0.9 mg/dL Serum sodium, serum potassium 139 meq/L, 3.9 meq/L Serum bicarbonate 24 meq/L Total white blood cell count 5x103 cells/mm3 Question What is the most likely diagnosis?

Correct answer: Obstructive uropathy Explanation This patient most likely has obstructive uropathy secondary to an enlarged prostate. His prostate is enlarged, and his post-void residual is elevated (>200 mL). Obstructive uropathy can occur from compression of any portion of the urinary tract. Lower tract obstruction includes obstruction of the bladder or urethra and may present with micturition problems such as difficulty initiating voiding, incomplete emptying, urinary frequency, dribbling, etc. Bladder obstruction may occur secondary to bladder stones or prostatic enlargement impeding flow through the urethra. Neurogenic bladder is a functional impairment in bladder emptying that occurs in neurogenic diseases and as a complication of medications. It is diagnosed by cystometry and may be treated by using scheduled voids, self-catheterization, cholinergic agonists, and suprapubic catheterization if other treatments fail. Prolonged acute obstruction and/or chronic obstruction may cause declining glomerular filtration rates, inability to concentrate urine, inability to properly acidify the urine, and dysfunctional sodium and potassium excretion. This patient had a normal urinalysis as well as normal serum sodium, potassium, and bicarbonate levels, suggesting that his obstructive uropathy did not lead to obstructive nephropathy. Infectious cystitis and prostatitis may present with increased urinary frequency. An enlarged prostate may be noted in prostatitis. Both would likely be accompanied by fever, pyuria, and leukocytosis, which are not noted in this vignette. Both cystitis and prostatitis are complications of prostatic enlargement.

A 55-year-old Hispanic woman presents to establish care. She recently went to a health fair, where she had some basic serum chemistries drawn. Her serum creatinine was 1.5 mg/dl. On a questionnaire she completed in your waiting room, she noted that she has no known past medical history, except for occasional muscular aches, for which she takes indomethacin (about 2 or 3 times in the past 8 months). There is no family history of renal disease. Her BP is 142/82 mm Hg, and her body mass index is 31 kg/m2. Question What additional study would be most helpful in determining the cause of her elevated creatinine?

Correct answer: Fasting serum glucose level Explanation The most likely cause of chronic kidney disease in the United States is diabetes, so a history of elevated blood sugars is important to know. This patient is noted to be somewhat overweight, putting her at higher risk for diabetes. A fasting serum glucose level would be useful to rule of diabetes mellitus. Complete blood count may reveal anemia related to chronic kidney disease, but would not be useful in identifying the cause of kidney dysfunction. Chronic urinary infections can lead to scarring of the renal interstitium and to chronic kidney disease. It is important to ask about this risk, particularly if other risk factors for kidney disease are not noted. A urine culture can be used to rule out urinary tract infection. However, in this patient with no signs or symptoms of urinary tract infection, urine culture is not likely to be a useful diagnostic test. Hematuria can be caused by a variety of disorders including glomerular disease, acute and chronic infections, nephrolithiasis, renovascular disease, cystic kidneys, and urogenital cancers. It is prudent to ask about hematuria. Still, these disorders are less common than diabetic and hypertensive nephropathies, so they are not the most likely cause of her elevated creatinine. Renal ultrasound can identify renal scarring, tumors, kidney stones, and other structural abnormalities that could be causing kidney dysfunction, but ultrasound is not a usual first-line screening test.

A 33-year-old woman presents with a 1-day history of burning sensation during urination and foul-smelling urine. She is sexually active with 1 partner and uses a diaphragm for contraception. Fluid intake and blood glucose are normal. Leukocyte esterase dipstick test is positive, and urinalysis reveals 10 WBC/ml and bacteria. Further investigations confirm urinary tract infection (UTI), and you treat it appropriately. During a follow-up visit, she tells you that this is the 3rd episode of UTI in the past few months and asks you what she can do to reduce the risk in future. Question What is one of the steps she can take to reduce the risk of infection?

Correct answer: Frequently void the bladder Explanation Frequent voiding of bladder reduces risk of UTI by flushing out the uropathogens. Cranberry juice, not raspberry juice, is considered an effective prophylaxis for UTI. Current recommendations mention consuming 300 ml of pure unsweetened juice per day. Diaphragm and spermicide use is associated with higher risk of UTI. Women with recurrent UTI should be advised against use of vaginal douches, feminine products, and sprays, especially if they are perfumed. Fluid intake should be increased (at least 2 liters per day) and not decreased to reduce risk of UTI, as it increases urination and may help flush out the bacteria.

Case A 5-year-old boy presents with history of low-grade fever, headache, and intermittent colicky pain in the abdomen, which has been localized mainly around the umbilicus since yesterday. The child has vomited once. His symptoms are also accompanied by a maculopapular rash that is more confluent over the lower extremities and the buttocks. There is no itching. The rash is a purplish-red color. Both knees and ankles are swollen and tender, and there is edema of the hands and feet mainly in the dependent areas. Examination of the cardiovascular, respiratory system, and abdominal examination are essentially normal. Laboratory investigations show: Hb. - 10gm%, WBC. 11,000/cmm Platelet count - 550,000/cmm Serum IGA - 500 mg /dL (normal 14-159 mg/dLfor 2-5 years age group) Urine - Proteinuria++, RBCs++ Stool - RBC+ Question What is the most likely diagnosis?

Correct answer: Henoch-Scholein purpura Explanation The most likely diagnosis is Henoch-Scholein purpura, which is also known as anaphylactoid purpura. It is the most common cause of non-thrombocytopenic purpura in children. Boys are affected twice as frequently as girls. It is a common vasculitis of small vessels, with cutaneous and systemic manifestations. The systems primarily involved are the skin, gastrointestinal tract (GIT), and kidneys. The characteristic manifestation of the disease is the rash, which presents initially as a pink maculopapular rash, but progresses to petechiae and purpura, often referred to as palpable purpura. The rash may continue to appear intermittently for 3 or 4 months, or even up to 1 year. Edema and vasculitis of the GIT may lead to GI hemorrhage, manifesting with colicky pain in abdomen, vomiting, and hematemesis. There may be enlargement of mesenteric lymph nodes. Stool is positive for occult blood. Swelling of knee and ankle joints is frequently seen due to serous effusion. There may be edema of the dependent areas. Renal involvement, which is the most important cause of morbidity and mortality, manifests as hematuria, proteinuria, and hypertension. Central nervous system and cardiac involvement may rarely occur. Laboratory findings include thrombocytosis, leukocytosis, and elevated ESR. Serum IgA levels are elevated. Urine examination shows albuminuria, hematuria, and the presence of white blood cells and casts in the urine. Renal biopsy may show mesangial deposition of IgA. Diagnostic criteria of Kawasaki disease are fever of more than 5 days duration and the presence of at least 4 of the following conditions: (1) Strawberry tongue (protuberance of tongue papillae) suggestive of streptococcal infection (2) Diffuse reddening of the oral and pharyngeal mucosa, dry and cracking lips. (3) Conjunctivitis without any discharge. (4) Edema/erythema of the hands and feet and later desquamation of the skin of the fingers and toes. (5) Polymorphous rash. (6) Cervical lymphadenopathy (at least one lymph node >1.5 cm). These features are not present in the above child. Systemic lupus erythematosis (SLE) is a multisystem disease involving nearly all the organs. It is an autoimmune disorder that causes inflammation of the blood vessels and connective tissue, resulting in multisystem involvement. It is seen more commonly in girls in contrast to Henoch-Schonlein purpura, which is more common in boys. Joints may be merely stiff or there may be active inflammation. Cutaneous manifestations include malar, or butterfly, rash involving the cheeks and nasal bridge. Rash may be photosensitive and may involve all sun exposed areas. This rash is quite different from the rash of Henoch-Schonlein purpura. Hepatosplenomegaly and lymphadenopathy are often present. Cardiac involvement may include pericarditis, valvular thickening, myocarditis, conduction abnormalities, and congestive cardiac failure. Pulmonary involvement includes pulmonary hemorrhage and fibrosis. This is in contrast to the index case. Renal involvement may manifest as hypertension, edema, electrolyte abnormalities, nephrosis, or acute renal failure. Systemic onset juvenile rheumatoid arthritis (JRA) may be characterized by spiking fevers, arthritis, hepatosplenomegaly, lymphadenopathy, and serositis leading to pericardial effusion. Fever is accompanied by a faint transient, evanescent salmon-colored macular rash more commonly over the trunk and proximal limbs. It is non-pruritic and may last for a few hours. Heat, even that of a warm bath, may cause reappearance of the rash. Lab investigation includes raised ESR, leukocytosis, thrombocytosis, and C-reactive proteins (CRP) and anemia of chronic disease. JRA is the most common chronic rheumatologic disease in children, with a minimum duration of 6 weeks. The new nomenclature juvenile idiopathic arthritis (JIA) is being increasingly used to better define various subgroups. Clinical manifestations of Polyarteritis nodosa (PAN) is a necrotizing vasculitis involving small and medium sized arteries. Boys and girls are equally affected. It is believed to be a post-infective autoimmune response in susceptible individuals commonly occurring after upper respiratory infection by group A streptococcal infection, chronic hepatitis B infection, infectious mononucleosis, and tuberculosis. Common features include fever, weight loss, and abdominal pain. Skin manifestations include purpura, edema, and painful nodules along the course of arteries. Cardiac involvement occurs as myocarditis, pericarditis, and arrhythmias. Angiography may show aneurismal dilatation and segmental stenosis.

Case A 65-year-old man presents with chronic low back pain. His wife is with him. A urinalysis is ordered and demonstrates microscopic hematuria. The patient's complete blood count and metabolic profile are within normal limits. A CT urogram study is ordered for further evaluation, and the results are as follows: Noncontrast CT Urogram Exam: The examination demonstrates a large left renal mass, which engulfs the majority of the left kidney. Also present is bulky retroperitoneal adenopathy and expansion of the left renal vein, concerning metastatic adenopathy with tumoral venous extension. Multiple, aggressive-appearing lytic lesions are also seen throughout the lumbar vertebral bodies, sacrum, and iliac crests. Overall, these findings are consistent with metastatic renal cell carcinoma. Upon learning of the news, you first disclose the information to the patient's wife. The patient's wife then begs you not to disclose this information to her husband because she fears that the emotional impact of the news will be "more than he can bear". Question Based upon the clinical scenario, what should be done next?

Correct answer: Fully disclose the patient's condition to the patient and provide treatment options Explanation You should fully disclose the patient's condition to the patient and provide treatment options. Informed consent involves discussion with patients about their diagnosis, treatment options, risks and benefits of treatment, long-term outlook, pain management, and overall prognosis. In clinical practice, it is always the best practice to be honest and truthful with your patients. You should never hide a diagnosis from a patient, no matter who asks that you do so. References AMA Code of Medical Ethics. Ethics in Medicine. Milton D. Heifetz.

A 63-year-old man presents with a 6-month history of symptoms of urinary frequency, hesitancy, and nocturia. In addition, burning dysuria has occurred on 2 occasions, requiring treatment with antibiotics. He has a 1-year history of angina pectoris, for which he takes occasional nitroglycerin. On physical examination, the blood pressure is 130/90 mm Hg; heart rate is 90/min and regular, and an enlarged prostate is palpable per rectum. Laboratory data, including EKG, yields no contraindication to surgery; he is operated upon for a transurethral resection of the prostate. Anesthesia and surgery are uneventful, and blood loss is minimal. 6 hours postoperatively, he experiences a shaking chill, a temperature of 40 degrees C, and his blood pressure is 90/60 mm Hg. Question What is the most likely diagnosis?

Correct answer: Gram-negative bacteremia Explanation The correct response is Gram-negative bacteremia. Some degree of enlargement of the prostate is extremely common from the age of 50 onwards, but this type of enlargement often produces either minor symptoms, or no symptoms at all. However, benign hypertrophy of the gland results in elongation and tortuosity of the prostatic urethra, and the median lobe may become a large, rounded, swelling overlying the posterior aspect of the internal urinary meatus. Here, it can act like a ball valve, producing urinary obstruction. The deranged anatomy in the region of the internal meatus, may allow urine into the prostatic urethra. The urine in this situation sets up a desire to micturate, and this produces one of the most common symptoms of prostatism, namely, frequency. This is particularly worrisome to the patient at night, as it interferes with his sleep. The obstruction, and instrumentation to relieve it, predisposes to urinary infection. The obstruction to the outflow of the bladder may result in renal failure and uremia. Gram-negative enteropathogens are the most common cause of urinary tract infections and intra-abdominal sepsis, especially post-operatively, in the acute abdomen. Septicemia causes high fever, shivering, headache, and rapid breathing; it may progress to delirium, coma, and death. Myocardial infection gives rise to chest pain, which is usually of greater severity and duration than in angina, and is associated with nausea, vomiting, sweating, and extreme distress. The patient may be cold and clammy with tachycardia, hypotension, cyanosis, and mild pyrexia (Postoperative bleeding may lead to hypotension and hypovolemic shock, unless fluid volume is rapidly replenished. Arrhythmias may give rise to tachycardia and bradycardia, which are sometimes felt as palpitations. They may also present with their hemodynamic consequences: dyspnea, angina, collapse, or "funny turns". Corresponding EKG changes are diagnostic. Pneumonia is relatively slow in onset, with symptoms of systemic upset, fever, pleuritic pain, cough, and green sputum (may be scanty at first, or, "rusty" in color, if due to pneumococcal). On examination, there will be signs of consolidation, or just localized crepitations. Tachypnea is a valuable sign, especially in the elderly, in whom there is high index of suspicion.

Case A 55-year-old man presents with painless gross hematuria since last evening. Urine cytology and cystoscopy reveal transitional cell carcinoma of the bladder. He is diabetic and hypertensive on treatment with metformin and atenolol, respectively, for the past 10 years. He has never smoked but consumes alcohol, about 2 pints of beer everyday. He worked as a car mechanic at a garage for 30 years. He recently visited several African countries, and on coming back, had diarrhea and was diagnosed and treated for amebiasis. Question What in his history predisposed him to bladder cancer?

Correct answer: His occupation Explanation His occupation as a mechanic predisposed him to bladder cancer. Occupational exposure to diesel and petroleum products and solvents are considered a risk factor for bladder cancer. Aniline dyes and aromatic amines are responsible for the higher risk. Other occupations exposed to these chemicals include plumbing, truck driving, leather, rubber, and metal work. Organic chemicals and dyes are also known to increase risk of exposure. Therefore, dry cleaners, beauticians, paper industry workers, etc, are also at higher risk. Schistosomiasis, and not amebiasis, is a known risk factor for nontransitional cell bladder cancer. Smoking, and not alcohol consumption, increases the risk of transitional cell bladder carcinoma. Hypertension increases the risk of renal cell carcinoma, not bladder cancer. Metformin is not associated with bladder cancer.

A 2-week-old male infant presents with his father for evaluation of enlarged scrotum. The father states that the scrotum was a little larger in the first few days after birth than it is now, but it has not reduced in size enough to make him feel comfortable that it is normal. Physical examination reveals normally developed penis with abnormally large scrotum that transilluminates on the right side when light is shined on it. Question What is the most likely diagnosis?

Correct answer: Hydrocele Explanation A testicular mass that transilluminates in an infant is most likely a hydrocele. Hydroceles are common in infants and are formed when the processus vaginalis, a tubular extension of the peritoneum, fails to close during development. The processus vaginalis precedes the testes in their descent into the scrotum and then usually closes around the time of birth; however, 80-94% of newborns have a patent processus vaginalis, making hydroceles extremely common. When it does not close, peritoneal fluid collects in the tunica vaginalis, which surrounds the testes, creating enlargement of the scrotum that transilluminates on exam. Inguinal hernias, especially indirect inguinal hernias, develop in a manner similar to hydroceles. The main difference is that the processus vaginalis is patent enough to allow bowel to pass through, not just peritoneal fluid; therefore, it would not transilluminate on exam. Spermatoceles, also known as epididymal cysts, are cysts located at the head of the epididymis. They can present as transilluminating masses on exam; however, they are not nearly as common as hydroceles in infants, and they can be palpated as distinct from the testes on examination. Testicular tumors are uncommon in infants; however, they are the most common solid tumors in males ages 15-35. A tumor would not be expected to transilluminate. Varicoceles are dilated, tortuous veins surrounding the spermatic cord. They are sometimes found in adolescents but are rare in infants.

Your 52-year-old male patient has chronic renal failure on dialysis for 1 year. He is suffering from anemia and hypoproteinemia. He complains of bone pains, which have developed recently. Renal osteodystrophy is diagnosed after appropriate tests and you plan vitamin D therapy. What is the role of the kidneys in the synthesis of vitamin D?

Correct answer: Hydroxylation of 25-hydroxy vitamin D3 at the number 1 position Explanation Activation of Vitamin D is completed by the kidneys when they hydroxylate it at the number 1 position. The 25-hydroxy cholecalciferol is converted to 1, 25 dihydroxycholecalciferol (calcitriol) by the kidneys by 1-alpha-hydroxylase. This activates vitamin D for use in calcium and phosphorus uptake from the intestinal lumen. Hydroxylation of vitamin D3 at the number 25 position is accomplished by the liver. Cholesterol is activated to cholecalciferol by sunlight (UV light) in the skin. Once active, vitamin D3 is placed in the intestinal epithelial cells. Excess can be lost through the lumen. Cholecalciferol cannot be converted to calcitriol without the intermediate hepatic step. Calcitriol is not converted to cholecalciferol. Cholesterol is synthesized by the liver or from dietary sources. Reference: Vitamin d in CKD: A systemic role for selective vitamin D activation Kidney International (2006) 69, 33-43

A 41-year-old woman has been hospitalized for over a week. Her laboratory results reflect an electrolyte abnormality and her EKG demonstrates peaked T waves as well as a widening of the QRS complex. Question These EKG abnormalities are characteristic of what condition?

Correct answer: Hyperkalemia Explanation Hyperkalemia refers to an elevated potassium level. Hyperkalemia can result in peaked T waves on EKG. The T wave is a reflection of ventricular repolarization. There can be widening of the QRS complex as well. The QRS complex is seen when the ventricle depolarizes. Hypercalcemia refers to an elevated calcium level. A shortened QT interval is seen with hypercalcemia. The time from the beginning of the QRS complex to the ending of the T wave is the QT interval. Hypothermia can cause a slower rhythm. Osborne waves can be present. There can be an elevation of the ST segment. The time from the end of the QRS complex to the beginning of the T wave is the ST segment. Hyponatremia refers to a depressed sodium level. Hyponatremia is not associated with EKG changes. Hyperglycemia refers to an elevated blood glucose level. Hyperglycemia is not associated with EKG changes.

The picture below is a view of the inferior aspect of the penis and scrotum of a 2-month-old male infant. The mother brought the infant to a pediatric clinic because, during micturation, urine ran from the opening at the bottom of the midline groove instead of from the tip of the penis. Question What is the most likely diagnosis?

Correct answer: Hypospadias Explanation This is an example of hypospadias. It occurs frequently (about 1/300 male infants) and is caused by inadequate midline fusion of the urogenital folds in male embryos; consequently, there is incomplete formation of the penile urethra. Hypospadias often results from inadequate production of androgens by the testes or inadequate receptors for the hormones. The external urethral orifice may be located anywhere along the ventral midline aspect of the glans penis, shaft of the penis, scrotum, or perineum. Epispadias is an abnormal urethral orifice on the dorsal side of the penis. Cleft scrotum is due to failure of fusion of the labialscrotal swellings. Cryptorchism is either unilateral or bilateral failure of descent of the testes into the scrotum. A bifid penis is often associated with bladder exstrophy.

Case An 83-year-old man presents to his urologist's office with "problems down there." He is a poor historian, but his wife accompanies him and helps with details of the history of present illness. She does not report that he has any voiding issues and reports that he has never had to see a urologist before now. Further genitourinary review of systems questions is suggestive of phimosis. Question What physical examination finding would support this diagnosis?

Correct answer: Inability to retract the foreskin over the glans penis in an uncircumcised male patient Explanation Inability to retract the foreskin over the glans penis in an uncircumcised male patient is the correct answer. Phimosis can be the result of recurrent infections or irritation, advanced age, diabetes, and poor hygiene. Patients can experience painful erections, recurrent balanitis, and voiding difficulties. Treatment can be with topical steroids followed by gradual retraction of the foreskin or circumcision. Entrapment of the foreskin behind the glans penis in an uncircumcised male patient describes a patient with paraphimosis. Typically, this results in retraction of the foreskin for medical or hygiene purposes that is not followed by properly pulling the foreskin back in place over the glans penis. This can be very painful and needs to be manually reduced as soon as possible to prevent necrosis of the glans penis. A dorsal slit or circumcision may be necessary if manual reduction is not possible. Erythema and edema of the glans penis in an uncircumcised male patient most closely describes balanitis and does not address the placement or status of the foreskin, which is the main concern with phimosis. Erythema and edema of the phallus/glans penis in a circumcised male patient are not correct, as patients who are circumcised cannot experience phimosis due to their lack of foreskin.

A 14-year-old boy presents due to embarrassment after an incident in school: while undressing before the class in physical education, the other boys laughed at him because of his "underdevelopment". His personal and family history is non-contributing. Question In males, what is the most common initial sign of puberty?

Correct answer: Increase in size of testes Explanation In boys, age of 13 and a half years is the upper limit of normal onset of puberty (to simplify, 14 years is often used as the upper limit). The initial sign is usually an increase in the size of the testes to more than 2.5 cm in their longest diameter, excluding the epididymis. Testicular size reaches adult size about 6 years after the onset of puberty. Increase in the speed of growth (pubertal growth spurt) is the initial sign of puberty in girls (although breast development is usually the first sign of puberty reported). In boys, a growth spurt usually follows the increase in size of testes. Changes in body composition (lean body mass, skeletal mass and body fat) usually follow changes in testicular size. They happen several years earlier in girls, who also have an earlier growth spurt and weight gain. Critical periods of bone accretion occur during infancy and puberty in both sexes. In puberty, peak of bone mineralization is registered after the peak height velocity. After testicles have enlarged and developed for about 1 year, the length of the penis increases; this is followed by an increase of the breadth of the shaft of the penis and the enlargement of the glans and corpora cavernosa.

An 18-year-old pregnant woman presents with polyuria and increased urinary frequency. She is at 18 weeks gestation and she has no past medical history. She has no suprapubic fullness; her fundal height is appropriate for her weeks of gestation. Diagnostic findings are as below: Urine No cells, no blood, no protein; trace glucose, specific gravity 1.020 Fingerstick glucose 85 mg/dL Renal ultrasound Bilateral mild hydronephrosis, no noted ureteral abnormalities; bladder not distended post-void Serum creatinine 0.5 mg/dL Question What is the most likely explanation for her polyuria?

Correct answer: Increased glomerular filtration Explanation This patient most likely is polyuric due to her increased blood volume and the increased glomerular filtration rate that occurs during pregnancy; both findings are normal. This patient is not isosthenuric (specific gravity of urine = specific gravity of plasma). If she were, she could not concentrate or dilute her urine. Concentrating and diluting abilities are maintained in pregnancy. Isosthenuria occurs when tubules are diseased, such as in diabetic and other forms of chronic kidney disease, and may lead to polyuria, nocturia, and electrolyte imbalances. This patient has trace glucosuria, which is likely due to high glomerular filtrate rates and less efficient tubular glucose absorption. Glucosuria is common in pregnancy and may certainly occur without diabetes. Her fingerstick glucose is not suggestive of diabetes. Physiologic hydronephrosis occurs in pregnancy due to the high hormone levels (estrogen and progesterone) that dilate the ureters. Hydronephrosis may occur with or without obstruction, and obstruction may occur with or without hydronephrosis. Ureteral obstruction should be noted on ultrasound and may cause an increase in serum creatinine, which is not noted here. Obstruction of the bladder or urethra may present with difficulty initiating voiding, incomplete emptying, and urinary frequency, dribbling, etc. Bladder obstruction may occur secondary to bladder stones or prostatic enlargement impeding flow through the urethra. The upper tract (proximal to the urinary bladder) may also become obstructed. Tumors, gastrointestinal processes, and retroperitoneal fibrosis (to name a few causes) may compress and obstruct the ureters. Functional problems, such as diabetes, neurological injuries, and medications, may impair the bladder's ability to empty urine. Neurogenic bladder is diagnosed by cystometry, which demonstrates impaired bladder emptying and low pressures. It is suspected in patients with neuropathies (diabetes), neurological diseases (multiple sclerosis), and in patients who take medications that impair cholinergic stimuli for bladder emptying (cold medicines). Additional causes of polyuria may include urinary tract infections (UTIs), which should be treated so that they do not affect the growing fetus.

A 15-year-old boy is seen by his primary care physician for delayed sexual changes during puberty. Upon physical exam, the physician notes an overall normal appearance, with small testes and some gynecomastia. Behaviorally, the patient seems to be somewhat shy and reserved, with occasional social outbursts. Although he is tall for his age, he is uncoordinated and inactive. Measured testosterone levels are decreased. A karyotype is done, and found to contain an extra chromosome. The most probable diagnosis for this child is which of the following?

Correct answer: Klinefelter syndrome Explanation Fragile X syndrome results from a mutation in the FMR1 gene, located on the long arm of the X chromosome (Xq27). It is characterized by moderate intellectual disability. The phenotypic manifestations of fragile X syndrome vary, but often include developmental delay, hyperactivity, abnormal craniofacials, and macro-orchidism (in post-pubertal males). Greater than 99% of affected individuals have what is known as a "full mutation" in the FMR1 gene. This mutation is caused by an increased number of CGG trinucleotide repeats in the 5' end of the gene (> 230 CGG repeats), which causes aberrant methylation of the gene, and aberrant expression of the gene product. Mothers of affected children that have this full mutation are obligate carriers of a "premutation" in the FMR1 gene. This premutation or "intermediate" allele has between 55 and 230 CGG repeats, and can expand upon transmission to offspring. (The normal allele has between 6 and 54 CGG repeats). These women, and their family members, are at an increased risk to have children affected with fragile X syndrome. Molecular genetic testing is available on a clinical basis to determine the status of the FMR1 gene allele(s). Klinefelter syndrome affects males. It is diagnosed by an abnormal karyotype. Individuals with this syndrome have an extra X chromosome, with the karyotype being 47, XXY (Variations occur, but this karyotype is the most common). The extra X chromosome seems to affect the functioning of the testes and testosterone production. Adolescent boys with this disorder may undergo gynecomastia. Most are tall, but not particularly coordinated. The penis is of normal length; however the testes are small. Treatment with male sex hormones can be helpful. X-linked adrenoleukodystrophy is caused by a mutation in the ALD gene, located on the long arm of the X chromosome (Xq28). The childhood form most commonly presents between the ages of four and eight years. It begins with symptoms of attention deficit and hyperactivity disorder, but gets progressively worse, with symptoms including difficulty with previously mastered subjects such as speech and reading. The affected individual also becomes clumsy and has visual disturbances. Brain MRI is often abnormal, even while symptoms are still mild. The rate of progression of the disorder varies, but leads to death in a matter of years. Along with the childhood form of the disease, there are several other types. Type two usually presents during middle age (possibly as early as in the twenties), and includes leg stiffness and weakness and sexual dysfunction. This, too, is progressive, usually over decades. Type three, which is found in approximately 10% of cases, is characterized by adrenal insufficiency. Presentation can be anywhere between two years of age to adulthood. X-linked mental retardation hypotonic facies syndrome is caused by a mutation in the XNP gene, also located on the X chromosome (Xq13.3). Affected individuals have a distinct phenotype that includes genital abnormalities, a common set of facial features, and severe developmental delays with intellectual disability. All patients have a normal 46, XY karyotype; however their appearance at birth can range anywhere from a male with hypospadius to a normal appearing female. Common craniofacial features include a small head circumference, small triangular nose, tented upper lip, prominent lower lip and open mouth. Short stature is common. Developmentally, milestones are delayed to a marked degree. Hypotonia is commonly present. Interestingly, the mutated gene appears to down regulate expression of the alpha-globin gene, leading to a microcytic and hypochromic anemia in some affected individuals. Individuals with XYY syndrome have an abnormal karyotype - 47, XYY

A 55-year-old woman with no significant medical problems presents with a CC of pink urine, stating, "I think I have blood in my urine." She states she has no pain with urination, but the hematuria is persistent. On questioning, she states that she has had a 1-month history of some progressively worsening left flank pain. The pains are not debilitating, but they are nagging. She admits to a 50 pack-year smoking history, and she states she is currently retired from her job as a teacher. Vital signs: T 98.6°F, BP 118/76 mm Hg, P 78/min, R 14/min. Abdominal exam reveals a left side abdominal mass. Urine dipstick only shows too numerous to count RBCs, and urine cultures are negative. CT scan of the abdomen and pelvis with and without contrast reveals a 4.2 cm solid enhancing lesion in the left renal parenchyma. Question What is the most effective treatment for this patient?

Correct answer: Left nephrectomy Explanation This patient is exhibiting the classic triad of renal cell carcinoma (RCC): Hematuria Flank pain A palpable flank or abdominal mass This is uncommon in patients with renal cell carcinoma (10%) and usually indicates advanced disease. The patient admits to a history of smoking, which is a risk factor for RCC. The acceptable treatment options for RCC include: Surgery, either partial or complete nephrectomy—the only curative treatment. Radiation therapy Immunotherapy Molecular targeted therapy Chemotherapy currently available is not beneficial in RCC. Hormone therapy, including progesterone therapy, has not been shown to be effective in treatment of RCC. Observation is a viable option in elderly patients and poor surgical candidates if the tumor is less than 3 cm in size. These patients are often followed with CT scans every 6 months.

A previously healthy 35-year-old man has had right flank pain radiating to his right groin for the past 3 hours. CT shows a 7 mm stone in his right ureter. What is the best option of treatment in this patient?

Correct answer: Lithotripsy Explanation This patient has a 7mm ureteral stone and will likely require surgical intervention, such as lithotripsy, for this stone to pass. Peitrow and Micali note that 90 to 98% of stones <5 mm are likely to pass on their own, although sometimes >30 days are needed for this to occur. A variety of agents may assist in stone passage. Deflazacort, a steroid, decreases ureter edema and may facilitate stone passage through the ureter. Nifedipine and tamsulosin decrease ureter spasm, facilitating smoother stone passage through the ureter. Analgesics, particularly opioids and non-steroidals, decrease the pain associated with stones lodging in the ureter. In the case of larger stones (>5 mm), passage is unlikely. Surgical interventions for stone removal include lithotripsy, a procedure which uses sound waves to break stones into smaller pieces which can be passed. Other procedures to assist in stone management include ureteral stenting, percutaneous nephrostomy tube placement, open surgical stone removal, and retrograde ureteral stone removal. However, since open surgical removal is an invasive procedure, lithotripsy is the first choice. Lithotripsy involves the usage of shock waves to crush stones in the renal calyx. It may be done as an outpatient procedure. Extracorporeal shock wave lithotripsy involves waves directed from outside; whereas intracorporeal shock wave lithotripsy consists of insertion of a percutaneous nephroscope and then crushing of the stones.

A 2-month-old male infant is being seen for a routine examination by his pediatrician. His mother admits to not following recommendations and has not had him seen by the pediatrician since his hospital discharge. During the genitourinary examination, the pediatrician cannot palpate the testis on either side of the scrotum. The pediatrician is concerned that the infant has bilateral cryptorchidism (undescended testes) but needs to make sure testes are present somewhere above the scrotum. Question What laboratory testing and imaging study combination would the pediatrician order?

Correct answer: Luteinizing hormone, follicle-stimulating hormone, and testosterone levels followed by ultrasonography Explanation Luteinizing hormone (LH), follicle-stimulating hormone (FSH), and testosterone levels followed by ultrasonography is correct. In male infants under the age of 3 months, LH, FSH, and testosterone levels are helpful in determining whether there are testes present. Ultrasonography has a sensitivity of 76%, a specificity of 100%, and an overall accuracy of 84% when diagnosing undescended testes that are nonpalpable on examination. MRI would have also been a good imaging choice, as it has a sensitivity of 86%, a specificity of 79%, and an overall accuracy of 85%. Ultrasonography is both easier and a lower risk to perform on infants and children, however, making it a more popular choice in many cases. CT scan findings in children, when used in the diagnosis of nonpalpable undescended testes, are historically not reliable. The HCG stimulation test is done by administering 2000 IU of HCG daily for 3 days and checking testosterone levels pre- and post-stimulation. This helps to determine the presence or absence of testicular tissue, but this test is reserved for infants older than 3 months. X-ray is not used at all in the diagnosis of cryptorchidism.

A 17-year-old female athlete presents with easy fatigability and weakness. She has been training daily and she has no significant medical history. Her pulse is 55/min, regular, BP 120/80 mm Hg. There are no remarkable findings on a physical examination. Lab reports show the following: Serum Na+ 140 mEq/L Cl- 86 mEq/L K+ 2.3 mEq/L HCO3- 34 mEq/L pH 7.50 Urine Na+ 82 mEq/24 h K+ 168 mEq/24 h Question What is the most likely cause of her condition?

Correct answer: Misuse of diuretics Explanation The patient has hypokalemia and alkalosis. Even though her serum potassium is low, she is still losing potassium in the urine. This is consistent with the action of a diuretic drug. Surreptitious use of diuretics by athletes is a common occurrence for the purpose of weight loss. Abuse of diuretics is also a component of eating disorders. Bulimic vomiting can also cause a hypokalemic alkalosis, but urinary excretion of potassium would not be elevated in that case. Additional signs of induced vomiting such as eroded dental enamel would likely be evident on physical examination. Hyperventilation would lead to a respiratory alkalosis with a compensatory mild metabolic acidosis. In the case of a somatoform disorder, there would not be any abnormal lab values. Anabolic steroids are unlikely to produce a picture of significant serum and urine electrolyte imbalance.

A 55-year-old man is admitted to the hospital with abdominal pain. He has a past medical history of hypertension. A CT scan with contrast reveals the presence of a small bowel obstruction. An NG tube is placed; IV narcotics are given for pain control; and IV fluids are started. A few days into his hospitalization, he develops confusion. Arterial blood gas is drawn, revealing the following: pH -7.55 CO2 - 42 HCO3 - 28 Question What in the patient's history is contributing to his acid base disorder?

Correct answer: NG tube placement Explanation NG tube placement is correct. The patient's ABG values indicate that he is suffering from metabolic alkalosis. Metabolic alkalosis is associated with elevated pH. The pH becomes elevated due to decreased hydrogen ion concentration or increased bicarbonate concentration. NG tube placement causes the loss of HCl from the stomach, and this is the most likely cause of metabolic alkalosis in this patient. Anxiety is incorrect. During anxiety attacks, patients may develop respiratory alkalosis due to hyperventilation. Carbon dioxide levels decrease during hyperventilation due to the patient's rapid breathing. Hypertension is incorrect. Hypertension itself is not associated with metabolic alkalosis. Certain medications, such as diuretics, may lead to metabolic alkalosis due to the associated fluid loss. However, there is no mention in the patient's history of diuretic use. IV narcotic administration is incorrect. The use of IV narcotics may cause CNS depression, which would lead to respiratory acidosis. Respiratory acidosis results from hypoventilation. CT contrast administration is incorrect. CT contrast administration may lead to renal failure in certain individuals. However, this would likely be associated with a metabolic acidosis and not metabolic alkalosis.

A 27-year-old woman presents with a change in her urine color; the change occurred a few days ago and has persisted. Her vital signs are: blood pressure 145/90 mmHg, pulse 82 bpm, respirations 16/min, and temperature 98.6°F. On physical examination, you note lower extremity edema. Urinalysis: Appearance: Pink and Cloudy Glucose: Negative Bilirubin: Negative Ketones: Negative Spec. Gravity: 1.035 Blood: 2+ pH: 6.5 Protein: Trace Urobilinogen: Normal Nitrite: Negative Leuk. Esterase: Negative Microscopic Examination: RBCs: 10 - 20 cells/hpf WBCs: 0 - 2 cells/hpf Question What is the initial diagnosis?

Correct answer: Nephritic syndrome Explanation The clinical picture is suggestive of nephritic syndrome. Nephritic syndrome is classified as a glomerular disease; clinical findings include edema, hypertension, and hematuria. Clinical findings for nephrotic syndrome include large urine protein, peripheral edema, and low serum albumin. Large urinary protein is not present in this patient. Clinical findings in cystitis include the patient being afebrile, irritative voiding symptoms, and positive urine cultures. On urinalysis, leukocyte esterase is usually present with positive or negative nitrites; it has a cloudy appearance and may have a noxious smell. Microscopic exam may reveal WBCs and bacteria. These symptoms are not present in this patient. Findings with pyelonephritis include fever, flank pain, and symptoms similar to cystitis. These findings are not present in this patient. Patients with nephrolithiasis (kidney stones) present with flank pain that may be severe and occur suddenly; the pain may radiate to the anterior abdomen. Nausea and vomiting are also present. Patients tend to move constantly. These symptoms are not present in this patie

A 50-year-old Caucasian man presents to establish care. He has no history of diabetes, hypertension, cardiovascular disease, cerebrovascular disease, peripheral vascular disease, nephrolithiasis, or obstructive uropathy. His only complaint on review of systems is musculoskeletal aches. His parents died in their 80s of unknown causes. He has never smoked, taken illicit drugs, had a transfusion, or had unprotected sex. He worked for years as a handyman. His only medication is ibuprofen, which he has taken at least several times per week for the past 10 years. His exam is normal. You order some basic diagnostic tests, including a basic serum chemistry panel and a urinalysis. The urinalysis shows microscopic hematuria and a few white cells but has no leukocyte esterase or nitrites. His serum creatinine is 1.5mg/dl. His other tests are normal. Question What additional test should you order and why?

Correct answer: Non-contrast CT abdomen or renal ultrasound to evaluate for analgesic nephropathy Explanation A CT of the abdomen should detect reduced renal size and lumpy bumpy contour present in analgesic nephropathy. This patient has consumed non-steroidals frequently and long enough to put him at risk for analgesic nephropathy. In the absence of any other overt cause for kidney disease, this diagnosis should be explored. Chronic analgesic consumption is associated with chronic kidney disease. Analgesic nephropathy is still a relatively uncommon cause of chronic kidney disease; its prevalence is higher in Europe and Australia than in the United States, where it accounts for only 2-4% of the cases of end-stage kidney disease. Some studies have debated causality; however, a New England Journal of Medicine article addressed the risk in patients with chronic consumption and found a 2.5-5 fold increase risk of chronic renal failure in patients who chronically used acetaminophen versus those who did not. Cumulative lifetime use increased the odds of renal failure and results were similar for patients who took aspirin. The highest risk was noted among patients who took >500g/yr acetaminophen (>1.4g/day). Other studies have found increased risk among users of >3-5 gram per lifetime and >6 tablets/day for 3 years. This patient is not at a particularly elevated risk for renovascular disease, given his lack of other vascular disease and lack of smoking. Renovascular disease is an important cause of chronic kidney disease, particularly in elderly populations, and its evaluation might be considered if asymmetric renal size is detected or no other cause for the elevated creatinine is found. Angiograms do pose an iatrogenic risk, and it is debatable if anything could be done to treat renovascular disease if it should be detected. The kidney damage will depend on the duration and degree of vascular impairment. Furthermore, antihypertensives are often quite effective in the management of renovascular hypertension. This patient has no known risk factors for hepatitis and should not be at risk for hepatitis associated causes of kidney disease (glomerular diseases including membranous and membranoproliferative glomerulonephritis and cryoglobinemia). A post-void bladder scan could detect urinary retention. This is a relatively easy diagnostic test. This patient apparently has no symptoms of urinary retention and has a normal physical exam, so urinary retention is a less likely cause. Although the urinalysis showed a few red and white cells, it was leukocyte esterase and nitrite negative. In an asymptomatic patient, an occult urinary tract infection is less likely. Additionally, there are no symptoms of urinary retention to put him at risk for getting a urinary infection. Imaging the kidneys is a better choice. Chronic infections can be a cause of kidney disease by leading to scarring of the renal interstitium.

A 50-year-old Caucasian man with diagnosed peptic ulcer disease (PUD) has nausea and severe vomiting for 2 days. Physical examination shows a patient with confusion and pale skin, tachycardia of 140 bpm, and blood arterial pressure of 90/50mm Hg. Further evaluation reveals hypochloremic, hypokalemic metabolic alkalosis.

Correct answer: Normal Saline Explanation Severe nausea and vomiting in a patient with PUD suggests gastric outlet obstruction (occurs in about 5% of patients). Metabolic alkalosis, hypokalemia, and hypochloremia are determined in laboratory studies, and they should be corrected. In patients with hypokalemic, hypochloremic metabolic alkalosis, the IV fluid of choice would be normal saline (0.9%NaCl). Normal saline contains water and 154 mmol/l of sodium and chloride ions and is useful for replacing gastrointestinal losses. Normal saline is an isotonic solution and thus effectively restores the intravascular fluid volume and the serum osmolality. Chloride is always required in cases in which the hypokalemia is associated with metabolic alkalosis. Potassium replacement is carried out once the volume and sodium deficits have been corrected. Many patients respond to medical management alone, but the problem tends to recur. Endoscopic dilatation or surgical intervention may be required. Solutions of 5% and 25% dextrose contain water and energy in the form of glucose; they are used in hypoglycemic states to provide energy. 0.45%NaCl and blood transfusion are not the IV fluids used in the initial management of hypochloremic, hypokalemic metabolic alkalosis.

Case A 42-year-old man presents with lower extremity swelling. His past medical history and review of symptoms is otherwise negative. The patient looks comfortable, with vitals showing the following: BP 142/91 mm Hg, HR 90 beats/min, RR 16 breaths/min, T 98°F, height 5'9'', and weight 158 lb. His examination is only remarkable for 2+ pitting edema in the lower extremities. The patient is counseled on a low-salt diet. The abnormal laboratory values are as follows: Lab Result Urinalysis 3+ protein, coarse granular casts, 2 - 5 WBCs, 0 - 2 RBCs Serum albumin 2.1 gm/dL Serum creatinine 2.0 mg/dL Serum BUN 18 mg/dL Hemoglobin 12.1 gm/dL Question What should be the next step in the management of this patient?

Correct answer: Order a 24-hour urine to quantitate urine protein Explanation The clinical picture is most consistent with nephrotic syndrome. This syndrome is characterized by proteinuria in excess of 3.5 grams a day per 1.73 m3 body surface area. Other symptoms commonly seen include edema, hypoalbuminemia, and hyperlipidemia. Patients may also exhibit anemia. Usually there is not an active urine sediment. The correct diagnostic test to confirm nephrotic syndrome is a 24-hour urine to quantitate the protein loss. If the diagnosis is confirmed, a renal biopsy may be considered to aid in determining the cause of nephrotic syndrome. However, because it is an invasive test, it should not be performed until the diagnosis is established. The patient has no symptoms of a urinary tract infection and only a small number of WBCs in her urinalysis, so a urine culture and empiric treatment would not be indicated. Since the patient does not have hematuria or pain, a kidney stone is unlikely; therefore, an intravenous pyelogram would not be indicated.

A 17-year-old sexually active boy is seen for the onset of excruciating right testicular pain 3 hours ago. There has been no fever, testicular trauma, or prior episodes of testicular pain. Placing cold packs on the scrotum provided no relief. Examination revealed a Tanner V male with normal vital signs. The right hemiscrotum is red, painful, warm, and swollen. The testicle and surrounding structures could not be palpated due to edema and pain. Elevation of the scrotum did not diminish the pain. Question What is your priority action?

Correct answer: Order an ultrasound to determine testicular blood flow and alert urology Explanation The acute onset of severe testicular pain must be considered to represent testicular torsion until proven otherwise. The short, 4-6 hour window between onset and testicular necrosis makes rapid diagnosis and treatment mandatory. While confirming the diagnosis with the finding of decreased testicular blood flow on ultrasound, urology should perform an immediate detorsion and bilateral orchiopexy (since the "bell clapper" deformity leading to torsion is often present bilaterally). Epididymitis can mimic torsion, but should be considered only after torsion has been ruled out. Although the pain in epididymitis may be relieved by scrotal elevation (as opposed to the usual lack of relief with torsion), this is by no means a universal distinction. Aspiration would be therapeutic for a scrotal hematoma, but would not be undertaken before imaging. In the above scenario, one would not wait for or rely upon urine results to exclude the diagnosis of torsion. Spending valuable time performing a more detailed physical examination is not warranted.

A mother brings her 2-year-old boy for the evaluation of frequent febrile urinary tract infections (UTIs); the infections have been present since the birth. He has just finished a 10-day prescribed trimethoprim/sulfamethoxazole (Bactrim) course, and he now has no problems. The mother thinks that main reason for the frequent UTIs is because a boy is neither interested nor willing to use the toilet; he is almost always wet. Physical examination today is unremarkable. He is not circumcised. You ordered dipstick urine analysis and perform ultrasound that came back normal. Question What will be your next step?

Correct answer: Plan voiding ureterocystogram Explanation Urinary tract infections (UTIs) are common in children and may cause permanent kidney damage. Urinary tract anomalies are risk factors for UTIs, and you should search for them in a boy of this age. While voiding cystourethrography (VCUG) is not recommended routinely after the first UTI, it should be performed if there is a recurrence, particularly a recurrence of febrile UTIs. Kidneys and bladder ultrasound are performed in younger children (3 - 5 years of age) as the initial step to evaluate anatomy, but they cannot relieve vesicoureteral reflux (VUR). Recommendations regarding antimicrobial prophylaxis still lack evidence, both for and against regardless, in this case, your approach should be based on the diagnosis of the presence of eventual urinary tract anomalies. You should tell the patient's mother that circumcision reduces UTIs, especially in high-risk boys; however, it is more important to exclude the presence of anatomical abnormalities, which put this boy at an even bigger risk. Voiding dysfunction is defined as daytime voiding disorders in children who do not have neurologic, anatomic, obstructive, or infectious abnormalities of the urinary tract. It is recommended that toilet training begin when a child is 18 months old and shows the interest. A child's interest usually appears around 24 - 25 months. Daytime dryness is usually achieved by 3 years of age. Dysfunctional voiding can lead to VUR, accidental urinary leakage, and UTIs; however, it is too early to think about the presence of dysfunctional voiding in this child. Both radionuclide cystography and voiding cystourethrography are used in detecting and grading vesicoureteral reflux. While VCUG is suggested for both girls and boys, radionuclide cystography is suggested only for girls because voiding cystourethrography is needed for adequate anatomic imaging of the urethra and bladder in boys. References

A 42-year-old woman presents for an evaluation of back pain. She states that she has a history of recurrent UTIs; she notes that on occasion her urine appears red. Upon physical examination, she demonstrates CVA tenderness. Her vital signs include blood pressure of 145/90 mm Hg, respiratory rate of 16 breaths/min, and heart rate of 72 bpm. Urinalysis shows leukocyte esterase 1+, protein trace, and blood 3+. The microscopic examination reveals 5-10 WBCs/hpf and 20-30 RBCs/hpf. An abdominal ultrasound demonstrates multiple, bilateral fluid collections within the kidneys. Question What, if any, is the most likely underlying renal pathology?

Correct answer: Polycystic kidney disease Explanation The clinical picture is suggestive of acute pyelonephritis in the setting of polycystic kidney disease. Abdominal or flank pain, blood in the urine, recurrent UTIs, and hypertension are common symptoms of polycystic kidney disease. Cystic lesions on abdominal ultrasound are a hallmark of the disorder. Cystic lesions would not be present in a patient with normal kidneys. A solid renal mass is most often seen in the context of renal cell carcinoma. Goodpasture syndrome is defined as a combination of glomerulonephritis and pulmonary hemorrhages. The disease is usually preceded by an upper respiratory tract infection. Patients usually present with dyspnea and hemoptysis. Medullary sponge kidney is a common and benign disorder presenting at birth; it is not usually diagnosed until the 30s-40s. Common findings include gross hematuria, recurrent UTIs, and renal stones. The kidneys will have irregular enlargements of the medullary and interpapillary collecting ducts, giving a "swiss cheese" appearance. This is not the finding present on this patient's abdominal ultrasound.

A 64-year-old man who was recently diagnosed with BPH presents with suspected acute renal failure. He provides a urine specimen that demonstrates high urine osmolality, low sodium urine, and high BUN-to-creatinine ratio. The urine microscopic examination reveals few red blood cells and white blood cells with no casts. Question What is the most likely explanation?

Correct answer: Postrenal cause Explanation The clinical picture is suggestive of a postrenal cause. Common etiology includes obstruction of the urinary tract. Initial urine findings include high urine osmolality, low sodium urine, and a high BUN-to-creatinine level. The microscopic examination may be normal, but it also may contain red blood cells, white blood cells, or crystals. A prerenal cause may have hyaline casts present, a Una < 20 mEQ/L and an FENA < 1%. Glomerulonephritis would have dysmorphic red blood cells and red blood cell casts on microscopic examination. Acute tubular necrosis would have pigmented granular casts and renal tubular epithelial cells on microscopic examination. Interstitial nephritis would have pyuria (including eosinophiluria), white blood cell casts, and white blood cells on microscopic examination. Patients may also present with fever and a transient maculopapular rash.

A 62-year-old man is hospitalized because of a 1-week history of extreme malaise and painful skin changes. It started as "just a rash" but developed into clusters of clear vesicles; new vesicles appear while old ones dry and crust over. His past medical history is significant for type 2 diabetes mellitus, for which he is on a controlled diet. You find the rash distributed over his body and you make a diagnosis of disseminated herpes zoster. Acyclovir IV is introduced together with the supportive therapy, but after 2 days, the patient starts experiencing nausea and swelling; his urine output dramatically decreased and urinalysis revealed gross hematuria, with other laboratory studies showing elevated BUN and creatinine. Question What is the most probable mechanism of renal failure in this patient?

Correct answer: Renal tubular obstruction Explanation Acyclovir precipitates in renal tubules because it is poorly soluble in urine. In that way, it causes the obstruction of renal tubules and acute renal failure. Endothelial injury is characterized by reduced vasodilation, a proinflammatory state, and prothrombic properties; it may be associated with hypertension and diabetes, particularly in type 2 diabetes with insulin resistance. But in a patient with acute kidney failure who has been exposed to the high dose of parenteral therapy with a poorly soluble nephrotoxic drug, endothelial injury should not be your initial choice. Renal tubular cell dysfunction caused by a hypersensitivity reaction to drugs or by infection will cause acute interstitial nephritis. It is often associated with obstruction or reflux, so you can include this on your list of differential diagnoses, but the development of acute renal failure during the therapy with acyclovir makes renal tubular obstruction more likely. In acute interstitial nephritis, renal tubular cells dysfunction is caused primarily by a hypersensitivity reaction. When caused by an allergic reaction, the symptoms of acute tubulointerstitial nephritis are fever, rash, and enlarged kidneys. Besides, acyclovir-induced crystalluria that causes mechanical tubular obstruction is the better option in this case. Kidney infection with acute renal failure is not a probable diagnosis in a patient that has no back pain and no signs of a urinary tract infection.

A 63-year-old woman presents to you with a 5-year history of stage 3 chronic kidney disease. She states that she has not been very good about following her provider's orders, and wants to know what things she can do to help her condition. Question What is the appropriate dietary management for this patient?

Correct answer: Salt, potassium, protein, and phosphorous restriction Explanation The correct answer is the restriction of salt, protein, phosphorus, and potassium. Some studies have shown that protein restriction will slow the progression to end-stage renal disease. Overload of sodium and water can lead to congestive heart failure and edema. Phosphorus and magnesium restriction is needed, as hyperphosphatemia and hypermagnesemia can be seen in chronic renal failure; this is due to decreased excretion of phosphate and magnesium. The other answers are not correct, as potassium supplementation could cause a hyperkalemic state; it should be avoided in chronic renal failure unless otherwise indicated. Patients will stage 3 kidney disease should follow a potassium-restricted diet. Phosphorus, magnesium, and protein should be restricted as indicated above.

A 65-year-old man presents with flank pain, blood in his urine, and an unexplained weight loss. His past medical history is significant for numerous infections, kidney stones, cigarette use, and alcohol use. On physical exam, there is a palpable abdominal mass, as well as a low-grade fever. Diagnostic tests determine that he has cancer. Question What puts this patient at risk for the development of his particular type of cancer?

Correct answer: Smoking Explanation This patient has renal cell carcinoma. Smoking is a risk factor for many neoplasms, including renal cell carcinoma. There is a classic triad associated with renal cell carcinoma: hematuria, flank pain, and a palpable abdominal mass. The classic triad is not seen in most patients, however. Other presenting symptoms include a fever and weight loss. Cells from the proximal convoluted tubule are the most common cells from which renal carcinoma arises. There is an increased incidence of renal cell carcinoma with von Hippel-Lindau disease. Escherichia coli infection, urolithiasis, and interstitial nephritis are not known risk factors. Schistosoma haematobium infection is associated with bladder tumors, not renal cell carcinoma.

Case A 45-year-old premenopausal woman presents with urinary leaking; the leaking occurred after frequent coughing due to upper respiratory infection. She denies the urge to void; there has been no increase in urinary frequency, nocturia, or sense of incomplete voiding. She has no chronic medical problems, and her only medication is a multivitamin. Her BMI is 22 kg/m2, and her temperature is 98 degrees Fahrenheit. She has no grossly visible urethral strictures, no bladder distension, and has a normal pelvic examination. Diagnostics include: Post void residual 25 ml Urinalysis Specific gravity 1.015, pH 6, no cells, no glucose, no blood, no casts, no leukocyte esterase or nitrate Voiding diary No nighttime voids 5 voluntary voids and 3 leaks with coughing Total output 2.5 liters a day Question What is the most likely diagnosis?

Correct answer: Stress incontinence Explanation This woman most likely has stress incontinence. Stress incontinence is characterized by the involuntary leaking of urine during stress or increases in abdominal and bladder pressure, such as coughing and sneezing. Bladder pressure at these times exceeds urethral pressure, allowing urine to leak through the urethra. Stress incontinence is more common in women than men. Obesity, pregnancy, and vaginal births may increase the risk for stress incontinence. In such cases, the pelvic floor muscles may be insufficiently strong to support the urethra and overcome pressure of urine flowing from the bladder. Treatments for stress incontinence include pelvic floor exercises. By repeatedly contracting and relaxing the vagina and pelvic floor, leaking may decrease. In this patient's case, treating her cough with cough suppressants may additionally help with the urine leaks. In obese patients, 5 - 10% weight loss may also improve symptoms. Pessaries may be inserted into the vagina to increase urethral support. Urethral support can also be increased surgically by inserting a fascial sling or vaginal tape to support the urethra. Duloxetine, a serotonin and noradrenaline reuptake inhibitor, may also improve symptoms of urinary stress incontinence. In cases of urge incontinence, patients typically have involuntary leaks, increased urinary frequency, and nocturnal incontinence; they occur either during or just after the sensation of needing to void. Bladder detrusor muscles may have variable activity. Treatments include scheduled voiding and anticholinergic medications (oxybutynin, etc.). Mixed incontinence refers to the presence of symptoms of both stress and urge incontinence. It may be seen in 1/3 of patients. Overflow incontinence refers to urinary leaks that occur due to an obstruction of urine flow. Post-void residuals are typically elevated; normal post-void residuals in the absence of retention are less than 200 ml. Initially, patients may experience dribbling after voids, straining, the sensation of a full bladder, and a constant urge to void. Prostatic hypertrophy, atonic bladders, etc., can impede urine flow. Once urine volume exceeds bladder capacity, it may spill out, causing a leak. Overflow incontinence may be distinguished from urge incontinence by urodynamic testing, and it may be treated with terazosin and finasteride. Incontinence may be a symptom of a urinary tract infection. In this case, the lack of fever, urinary white cells, urinary nitrate, and urinary leukocyte esterase make this diagnosis unlikely.

A 55-year-old postmenopausal woman presents with urinary leaking during coughing and sneezing. She denies the urge to void; there has been no increase in urinary frequency, no nocturia, and no sense of incomplete voiding. She had 2 vaginal deliveries over 20 years ago. She has no chronic medical problems, and her only medications are a multivitamin and calcium + vitamin D supplements. Her Body Mass Index is 22 kg/m2, and her temperature is 98 degrees Fahrenheit. She has no grossly visible urethral strictures, no bladder distension, and a normal pelvic examination. Diagnostics include: Post void residual 25 ml Urinalysis Specific gravity 1.015, pH 6, no cells, no glucose, no blood, no casts, no leukocyte esterase or nitrate Voiding diary No nighttime voids 5 voluntary voids and 3 leaks with coughing 2.5 liters a day total Serum calcium 9 mg/dl Question How should you initially treat her symptoms?

Correct answer: Suggest pelvic toning exercises Explanation This patient has symptoms of stress incontinence and should be initially prescribed pelvic toning exercises. Stress incontinence is characterized by the involuntary leaking of urine during stress or increases in abdominal and bladder pressure such as coughing and sneezing. Bladder pressure at these times exceeds urethral pressure, allowing urine to leak through the urethra. Stress incontinence is more common in women than men. Obesity, pregnancy, and vaginal births may increase the risk for stress incontinence. In such cases, the pelvic floor muscles may be insufficiently strong to support the urethra and overcome pressure of urine flowing from the bladder. Treatments for stress incontinence include pelvic floor exercises. By repeatedly contracting and relaxing the vagina and pelvic floor 30 - 50 times/day, leaking may decrease. In obese patients, 5 - 10% weight loss may also improve symptoms. This patient has an acceptable body mass index of 22 (18.5-24.9 kg/m2), so weight loss won't necessarily help her symptoms. Pessaries may be inserted into the vagina to increase urethral support and help with symptoms. Urethral support can also be increased surgically by inserting a fascial sling or vaginal tape to support the urethra. Since surgical interventions are invasive and may involve bleeding and infection risks, medical therapies should be attempted first. Increasing the time interval between voids is not suggested. She will increase the volume of urine retained in the bladder and put herself at risk for a larger volume leak. In urge incontinence, decreasing the time between voids (e.g., scheduling voids every 2 hours) is suggested in order to avoid leaks. This patient does not have urge symptoms, such as increased frequency, nocturnal incontinence, or the extreme sensation of a need to void. There are no FDA approved medications for the treatment of stress incontinence. Anticholinergic medications block muscarinic receptors in the smooth muscle of the bladder and thus inhibit detrusor contraction. These medications are associated with moderate improvements in urgency, frequency, and urgency incontinence episodes.(11) Duloxetine, a serotonin and noradrenaline reuptake inhibitor, not fluoxetine, may be used in the treatment of stress incontinence. In small studies, Duloxetine may also improve symptoms of stress incontinence. β-3 Agonists are also available for treating urgency incontinence. Stimulation of the β-3 pathway promotes smooth muscle relaxation of the bladder to increase urine storage. There is only one medication, Mirabegron in this class, and it has shown efficacy in alleviation of urinary incontinence symptoms.(11)

A 15-year-old boy is seen in your office at 11:30 AM with a complaint of left scrotal pain and swelling; it started at 7 AM that same day when he woke up. He recalls no trauma. When questioned, he says that he has never had intercourse. He has been feeling nauseated, and he vomited once. Physical examination demonstrates a well-nourished, well-developed boy, appearing moderately uncomfortable. Vital signs are normal, with the exception of a temperature of 37.9°C orally. Pain assessment score (Wong-Baker scale) is 6/10. He is Tanner Stage III puberty. The remaining physical examination is normal, except for the following findings: the left testicle is approximately 1.5 times the size of the right testicle. The skin is diffusely erythematous. Due to tenderness when touched, it is difficult to palpate the scrotum. Cremasteric reflex is absent. There are small soft pea-sized lymph nodes in both inguinal areas. Penis is circumcised and appears normal. Scrotal ultrasonography with Doppler ultrasound demonstrates decreased blood flow to the testis. Question What is the most likely diagnosis?

Correct answer: Testicular torsion Explanation Testicular torsion in the adolescent boy is a urologic emergency, is the most common cause of acute scrotal swelling and pain, and is the most common cause of testicular loss. Torsion occurs 1 in 4000 and occurs most commonly on the left side in the United States. The cause is a congenital anomaly that occurs in approximately 12% of boys/men, in which the tunica vaginalis is attached too high, allowing the testicle to rotate freely on the spermatic cord and vascular pedicle in the tunica vaginalis. Approximately 40% of boys/men have the anomaly bilaterally. Testicular torsion usually occurs in ages 12-18 with the peak at 14. It may occur up to age 30, and it is found in infants and occasionally neonates at the time of birth. Up to 50% of patients may have had prior episodes of mild intermittent testicular pain that has resolved spontaneously due to intermittent torsion and spontaneous derotation Associated symptoms may include nausea and vomiting (20%), fever (16%), abdominal pain (20-30%), and urinary frequency (4%). Physical examination may demonstrate a horizontal position of the testis, and it may be elevated compared to the uninvolved side. The cremasteric reflex is usually absent, but its presence does not rule out testicular torsion. Elevation of the scrotum does not relieve the pain. The diagnosis is clinical. Because it consumes precious time, ultrasound examination of the testis with color flow Doppler should only be ordered when the diagnosis is uncertain, and it can determine if there is blood flow to the testis. The studies are 86% sensitive and 100% specific in making the diagnosis if the only criterion is decreased blood flow. Radionuclide scans are 90-100% accurate in identifying decreased blood flow. Rapid diagnosis is critical; if surgical intervention is provided within 6 hours of onset, the salvage rate for the testis is 80-100%; after 6 hours, the salvage rate is approximately 0%. Acute idiopathic scrotal edema is uncommon but presents acutely at age 6, on average. 90% of patients have a unilateral presentation. The scrotal skin is red and tender, but the testis appears to be normal. The redness tends to extend off the scrotum onto the perineum or onto the penis. This tends to resolve spontaneously in 48-72 hours and leaves no sequelae. Doppler ultrasound, if done, demonstrates good blood flow to the testis with peritesticular edema and fluid in the scrotal wall. Laboratory examination is normal except for occasional eosinophilia. Acute epididymitis and/or orchitis is not a common pediatric diagnosis. It was first described in 1956. The onset tends to be more gradual, generally over a few days, with fever and dysuria. Elevation of the scrotum may reduce discomfort. The cause may be viral (adenovirus, mumps, Epstein-Barr virus) or bacterial. Bacterial infection is often associated with structural changes in the urinary tract. Urinalysis and urine culture may be helpful in establishing the diagnosis. Typical treatment is with rest, analgesia, and antibiotics if there is concern about a bacterial etiology. If a bacterial cause is identified, urinary tract imaging should be performed. There have been rare reports of acute epididymitis progressing to testicular infarction. Torsion of the appendix testis may present similarly to testicular torsion. Tenderness is usually localized to the upper portion of the testis; typically, a blue dot is seen on the scrotal skin resulting from the venous congestion in the appendix testis. This is a self-limited condition and does not require surgical intervention. There are 5 appendages to the testis, all of which serve no function. If one twists or infarcts, symptoms result. Pain is less intense than with testicular torsion, and the cremasteric reflex is usually present. Varicocele occurs in 10-15% of males, 16% of adolescents, and 20-40% of men evaluated for infertility. First described in adolescents in 1885, the most common age of presentation is adolescence and early adulthood. They are caused by incompetent or absent valves of the spermatic veins, resulting in dilatation of the veins of the pampiniform plexus. Rarely are they caused by compression of the renal vein by a tumor, an aberrant renal artery, an obstructed renal vein. Doppler ultrasonography can demonstrate retrograde blood flow. They are most common on the left side, are usually asymptomatic, but may present with vague scrotal discomfort and swelling. Of those with symptoms, 2% have intratesticular varicocele and these are more common on the right side. The typical physical finding is the bag of worms within the scrotal sac. They may be missed on physical examination in the supine position, so the patient should be examined in a standing position. Patients should be referred to urologists for further evaluation and to discuss options for treatment; it sometimes requires surgery.

A 70-year-old man presents with a 3-month history of urinary leaking. He notes that he suddenly feels the need to urinate. He notes symptoms when at rest and when asleep; he has not had any symptoms when coughing or bearing down. He has not experienced urinary dribbling, dysuria, foul smelling urine, or fever. He has daily bowel movements. He has no other known medical problems, and he takes no medications. His temperature is 98 degrees Fahrenheit. His bladder is not distended, and he walks without difficulty. His post void residual is 30 ml. His rectal examination is normal, and his urinalysis is unremarkable. Question What is the most likely diagnosis?

Correct answer: Urge incontinence Explanation This patient most likely has urge incontinence. In urge incontinence, patients typically have involuntary leaks, increased urinary frequency, and nocturnal incontinence either during or just after the sensation of needing to void. Symptoms are not exacerbated by increased abdominal pressure or the stress of coughing/sneezing. Bladder detrusor muscles may be overactive, leading to the unexpected release of urine. Treatments include scheduled voiding and anticholinergic medications (oxybutynin, etc.). Stress incontinence is characterized by the involuntary leaking of urine during stress or increases in abdominal and bladder pressure, such as coughing and sneezing. Bladder pressure at these times exceeds urethral pressure, allowing urine to leak through the urethra. Urinary tract deficits are found commonly in older patients; both men and women have decreased bladder sensation, decreased contractility, and involuntary bladder contractions, which predispose them to incontinence. Obesity, pregnancy, and vaginal births may increase the risk for stress incontinence. In such cases, the pelvic floor muscles may be insufficiently strong to support the urethra and overcome pressure of urine flowing from the bladder. Mixed incontinence refers to the presence of symptoms of both stress and urge incontinence. It may be seen in 1/3 of patients. Overflow incontinence refers to urinary leaks that occur due to an obstruction of urine flow. In the absence of urinary retention, post void residuals are typically elevated; normal post-void residuals in the absence of retention are less than 200 ml. His 30 ml post-void urine volume is not consistent with urinary retention. Initially, patients with urinary retention may experience dribbling after voids, straining, the sensation of a full bladder, and a constant urge to void. Prostatic hypertrophy, atonic bladders, etc. can impede urine flow. Once urine volume exceeds bladder capacity, it may spill out, causing a leak. Overflow incontinence may be distinguished from urge incontinence by urodynamic testing, and it may be treated with terazosin and finasteride. Many medical problems and medications can contribute to incontinence. Delirium, restricted mobility, urinary infection, fecal impaction, polyuria, and medications that decrease urethral pressure (e.g., alpha blockers, neuroleptics, and benzodiazepines) or increased bladder pressure (e.g. anticholinergics, beta blockers, anti-Parkinson's medications, and bethanechol) may contribute to incontinence. The normal rectal examination and the history of regular bowel movements make fecal impaction a less likely diagnosis in this case. References

A previously healthy 8-year-old African American boy presented with a 3-day history of worsening fatigue and generalized edema. You saw him about a week ago when he had symptoms of a cold, for which you advised only supportive therapy. He appears alert and cooperative. His vitals are normal (temperature 37°C, pulse 90/min, respiratory rate 20/min, and blood pressure 100/70 mm Hg). Physical examination reveals the presence of generalized edema, and the rest of the examination is within normal limits. His laboratory values are below: Test Finding Normal (for age) Units Urine protein dipstick 4+ 0 Urine protein 40 0-20 mg/dL Specific gravity 1030 1008-1020 Urine Protein/creatinine ratio 2 <0.2 Per gr creatinine Serum protein 2.9 5.9-8 g/dL Cholesterol 299 112-247 mg/dL Urea nitrogen 10 17-Jul mg/dL Creatinine 0.5 0.3-0.9 mg/dL The rest of his laboratory results are not contributing. You plan to start therapy with prednisone. Question What parameter would be most important to follow and evaluate the effect of therapy in order to prevent chronic kidney disease?

Correct answer: Urine protein Explanation Your patient most likely has nephrotic syndrome. The diagnostic criteria for the nephrotic syndrome are generalized edema, hypoproteinemia with disproportionately low albumin level (<3 grams/dL), urine protein to urine creatinine ratio in excess of 2 mg/dL in a first morning void or a 24-hour urine protein loss that exceeds 50 mg/kg or 40 mg/m2, and hypercholesterolemia (>200 mg/dL). He is normotensive, and the effect of therapy on proteinuria will be the best indicator of therapeutic efficacy. Serum protein will probably normalize when the patient stops losing proteins. Your patient is normotensive. If he develops hypertension, then normalization of blood pressure would be an additional important factor that should be considered in the decision about the therapy. Edema is important for the evaluation of a need for diuretics, but it is not a direct parameter to be considered for the progression of chronic kidney disease. Small studies with statins in nephrotic syndrome in children demonstrated their efficacy in lowering hyperlipidemia, but there were no changes in proteinuria, hypoalbuminemia, or progression of renal disease. Lipid abnormalities will resolve when the nephrotic syndrome is in remission.

A 62-year-old man presents with persistent hematuria. He denies any dysuria, urgency, or frequency. The 1st episode of hematuria was discovered 4 months ago on a routine urinalysis for a workplace physical. A repeat urinalysis 6 weeks later again shows hematuria, confirmed by microscopic evaluation. The remainder of his urinalysis is within normal limits. His past medical history is remarkable for COPD and obesity. He has smoked 1.5 ppd x 45 years. He uses inhaled medications for his COPD and has NKDA. His physical exam is significant for a temperature of 100.5° F. Abdominal exam elicits mild tenderness in the left upper quadrant, but no masses are palpable (although his exam is limited by his obesity). Chest and abdominal CTs with contrast show a normal chest and a left renal mass enhanced by radiocontrast, suggestive of renal cell carcinoma (RCC). No metastatic disease was noted on imaging. A renal biopsy is pending. Question Assuming the biopsy supports the diagnosis of renal cell carcinoma, what referral would be most appropriate for this patient?

Correct answer: Urologic surgeon for radical nephrectomy Explanation The most appropriate intervention for this patient is referral to a urologic surgeon for radical nephrectomy, which is the standard treatment for localized RCC. The nephrectomy serves to diagnose, stage, and treat the cancer. A referral to a medical oncologist for traditional cytotoxic chemotherapy is not recommended, as RCC is refractory to traditional chemotherapies. Immunotherapy (interleukin or interferon) and antiangiogenic agents are considered a reasonable addition to nephrectomy in patients with metastatic RCC, but there is no current evidence suggesting this patient has metastatic disease. RCC is considered a radiation-resistant tumor, so external beam radiation is rarely used as a primary treatment. Radiation may have a role in palliative treatment in special circumstances. Referral to a urologist for a retrograde pyelogram would not address the RCC. This type of imaging allows visualization of the bladder, ureters, and pelvicalyceal collecting system by administering contrast through a catheter to flow up toward the kidneys. This type of imaging may be helpful in the diagnosis of urethral strictures, trauma, and reflux, but adds no additional information and certainly no treatment value for this patient with RCC. A referral to hospice for palliative care only suggests there are no reasonable treatments for this patient and that he is expected to die within 6 months; however, 5-year survival rates are around 66% for stage I RCC.

A 43-year-old woman presents with a 1-day history of burning sensation during urination and foul-smelling urine. She is sexually active with 1 partner and uses a diaphragm for contraception. She is hypertensive on treatment with thiazides. Fluid intake and blood glucose are normal. Leukocyte esterase dipstick test is positive, and urinalysis reveals 10 WBC/mL and bacteria. You suspect a urinary tract infection (UTI) and treat it appropriately after a culture and sensitivity test. During a follow-up visit, she tells you that this is the 3rd episode of UTI in the past few months and asks you whether she has any risk factor that could be responsible for the recurrence. Question What will you tell her?

Correct answer: Using diaphragm for contraception Explanation Diaphragm and spermicidal use are associated with higher risk of UTI. A recurrent UTI is considered to be 3 or more episodes of infection per year. The woman should be advised to avoid diaphragm and spermicide use and void soon after intercourse. A Large amount of fluid intake is generally recommended along with frequent emptying of the bladder to flush out the uropathogens. Vaginal douches, feminine products, and sprays should be avoided. Thiazides are diuretics that promote urination and do not increase the risk of UTI. If these behavioral modifications fail, low-dose antibiotics such as trimethoprim/sulfamethoxozole or a fluoroquinolone may be prescribed.

A 4-year-old uncircumcised boy presents with a 2-day history of penile pain. The patient is afebrile, and vital signs are stable. On genital examination, his foreskin is retracted proximally and the glans is edematous and cold. You are unable to reduce the proximal foreskin distally over the glans penis; it is strongly suspected that arterial flow is compromised. The only urologist available will arrive in 1.5 hours. Question What is the most appropriate treatment for this patient?

Correct answer: Vertical incision of the constricting band Explanation This patient presents with paraphimosis, which is an inability to reduce the proximal edematous foreskin distally over the glans penis. If arterial compromise is suspected, the most appropriate treatment is to make a vertical incision of the constricting band. Subcutaneous epinephrine, subcutaneous terbutaline, or local ice application will not correct paraphimosis; they could worsen the vascular compromise. Circumferential incision of the constricting band could result in amputation.

Case A 3-year-old girl is brought to the clinic by her mother, who tells you that the child has not been eating well over the past month and has developed swelling in the abdomen. On exam, the child has a smooth abdominal mass that is the size of a baseball on the left side. Vital signs reveal a blood pressure of 134/82 mm Hg, temperature of 99.8°F, and respirations of 16 breaths per minute. Urinalysis shows only 1+ red blood cells, and CBC and CMP are within normal limits. Question What is the most likely diagnosis?

Correct answer: Wilms tumor Explanation Wilms tumors account for 95% of all urinary tract malignancies in childhood. The median age of diagnosis is 3. These tumors are usually a solitary, unilateral mass. Children typically present with an enlarging smooth abdominal mass confined to one side of the abdomen. Other associated symptoms include abdominal pain, fever, hypertension (60%), and hematuria (25%). Multicystic dysplastic kidney disease is more common on the left side and is more common in boys. 60% of unilateral multicystic dysplastic kidneys involute in the first 3 years of life; instead of getting bigger to reveal a palpable mass, these typically get smaller over time in children. Splenomegaly in children is usually indicative of hepatic or hematologic disease. This child's CBC and CMP are within normal limits, making this diagnosis unlikely. Lymphoma often presents with painless, firm lymphadenopathy typically confined to 1 or 2 lymph node areas (usually the supraclavicular and cervical nodes). Mediastinal lymphadenopathy is also a common presentation. It can manifest with coughing and shortness of breath. Abdominal masses are not a common presentation of lymphoma, and CBC is typically abnormal in these patients. Renal cell carcinoma accounts for only 3% of childhood urinary tract malignancies. Wilms tumor is much more likely to be the diagnosis in a child.

Case A 59-year-old patient with end-stage renal disease presents via ambulance to the emergency department. Initially complaining of severe weakness, he quickly developed respiratory failure, for which he was intubated shortly after arrival. Stat chemistry labs reveal the following: Na 131 mEq/L; K 6.1 mEq/L; Cl 98 mEq/L; CO3- 19 mEq/L; BUN 42 mEq/L; Cr 6.6 mEq/L. Question Which of the following electrocardiographic (ECG) findings is most likely to be present?

hide Correct answer: Peaked T waves Explanation The patient is presenting with hyperkalemia. This is relatively common in patients with end-stage renal disease, especially among those who are noncompliant with medications and/or dialysis therapy. Signs and symptoms of hyperkalemia include weakness, which may escalate to muscle paralysis and respiratory failure. A number of characteristic electrocardiographic (ECG) findings are present in the setting of hyperkalemia. Tall, so-called "peaked" T waves are a classic early finding. As potassium levels rise, there is flattening of P waves, prolongation of the PR interval, and widening of the QRS complex. Untreated, a sinus wave pattern may develop followed by life-threatening dysrrhythmias and/or cardiac arrest. Reference: 2005 American Heart Association Guidelines for Cardiopulmonary Resuscitation and Emergency Cardiovascular Care. Part X. Life-threatening electrolyte abnormalities. Circulation. 2005;IV-121-IV-125.

Case A 10-year-old boy is brought to the clinic by his mother. She noted that his face is swollen, and he told her that his urine was cloudy and reddish. He has a history of falling and abrading the skin of his right thigh 2 weeks ago. The next day, the skin became red, hot, and tender; the infection was treated with a topical antibiotic ointment. The cellulitis gradually healed. You suspect acute glomerulonephritis. Question What organism is the most likely cause of the disease?

orrect answer: Streptococcus pyogenes (group A beta-hemolytic) Explanation Streptococcus pyogenes (group A beta-hemolytic) cause 3 types of diseases: pyogenic diseases, such as pharyngitis and cellulitis toxigenic diseases, such as scarlet fever and toxic shock syndrome immunologic diseases, such as rheumatic fever and acute glomerulonephritis. Glomerulonephritis occurs especially following skin infections. Streptococcus pneumoniae are gram-positive lancet-shaped cocci arranged in pairs (diplococci) or short chains. On blood agar, they produce alpha-hemolysis. Virulence factors of Pneumococci are polysaccharide capsules. Pneumococci cause pneumonia, bacteremia, meningitis, and infections of the upper respiratory tract, such as otitis and sinusitis. Mortality rate is high in elderly, immunocompromised (especially splenectomized), and/or debilitated patients. They should be immunized with the polyvalent polysaccharide vaccine. Peptostreptococci grow under anaerobic or microaerophilic conditions and produce variable hemolysis. Peptostreptococci are members of the normal flora of the gut and female genital tract and participate in mixed anaerobic infections of the abdomen, pelvis, lungs, and brain. Streptococcus agalactiae (group B streptococcus) colonize the genital tract of some women and can cause neonatal meningitis and sepsis. They are usually bacitracin-resistant. Enterococcus faecalis (group D streptococcus), formerly known as Streptococcus faecalis, are part of the normal flora in the gut. They can cause urinary, biliary, and cardiovascular infections.


संबंधित स्टडी सेट्स

Promise and a Blessing New Testament Survey

View Set

Chapter 16 Quiz: Sustainable Marketing: Social Responsibility and Ethics

View Set

Nursing Health Assessment Chapter 10

View Set